Mock Exam Practice Flashcards

1
Q

A 43-year-old man has a 10-year history of generalized anxiety disorder (GAD). He has been unable to tolerate paroxetine, escitalopram, or venlafaxine. His medical history is also significant for type 2 diabetes with painful peripheral neuropathy, dyslipidemia, and hypertension. He takes metformin, lisinopril, rosuvastatin, empagliflozin, and alprazolam. Which medication is best for long-term control of this patient’s anxiety symptoms?

A	Buspirone.
B	Duloxetine.
C	Pregabalin.
D	Sertraline.
A

C - Pregablin. Is a 1st line treatment for GAD and is good for patients who can’t tolerate SSRIs or SNRIs. Duloxetine is wrong because patient has a history of intolerance for Several SSRIs

How well did you know this?
1
Not at all
2
3
4
5
Perfectly
2
Q

A 27-year-old woman with Crohn’s disease presents to the infusion center pharmacy with a new prescription for vedolizumab infusion. She has no other significant medical history. She states that she has tried several other medications in the past with some success; most recently, she has been treated with infliximab infusion but has had reduced efficacy with the past several infusions. Which education point is most important to share with your patient?

A	Vedolizumab is an integrin inhibitor associated with a risk of rare but fatal progressive multifocal leukoencephalopathy (PML).
B	Biologic agents increase the risk of developing congestive heart failure and should be avoided in young patients.
C	Because you had loss of efficacy to infliximab, vedolizumab is not likely to be effective for you.
D	Vedolizumab is administered by intravenous infusion, requiring you to come to the infusion center for each dose.
A

D - Because it’s a MAB

How well did you know this?
1
Not at all
2
3
4
5
Perfectly
3
Q

J.A. is a 64-year-old man (height 70 inches, weight 90 kg) who is currently undergoing aortic valve replacement with a mechanical heart valve. His medical history includes hypertension, dyslipidemia, and generalized anxiety. His current medications include carvedilol 25 mg twice daily, lisinopril 10 mg once daily, atorvastatin 20 mg once daily, and escitalopram 10 mg daily. His vital signs and laboratory values are all within normal limits. Which is the most appropriate antithrombotic regimen for J.A.?

A	Apixaban 5 mg twice daily.
B	Adjusted-dose warfarin to an INR of 2.5–3.5 with aspirin 81 mg daily.
C	Aspirin 325 mg daily.
D	Adjusted-dose warfarin to an INR of 2.0–3.0 with aspirin 81 mg daily.
A

D - A and C are wrong because they are CI’d in patients with valvular disease. B is for mitral valves not aortic.

How well did you know this?
1
Not at all
2
3
4
5
Perfectly
4
Q

A randomized controlled trial compared a new anticoagulant (drug A) with warfarin for the treatment of venous thromboembolism. One of the safety end points was the incidence of clinically significant bleeding, which was 2.3% in the drug A group compared with 4.1% in the warfarin group. The 95% CI for the difference was -1.9% to 2.4%. Which conclusion is most appropriate?

A	Drug A was superior to warfarin with respect to bleeding complications.
B	Superiority of drug A over warfarin could not be established.
C	Warfarin was inferior to drug A with respect to bleeding complications.
D	No conclusion can be drawn because p values are unavailable.
A

B - The CI includes 0, thus no statistically significant difference.

How well did you know this?
1
Not at all
2
3
4
5
Perfectly
5
Q

A 67-year-old woman presents to the ED with septic shock, presumably caused by a UTI. Her medical history is significant for hypertension, hyperlipidemia, and recurrent Escherichia coli UTIs. Pertinent vital signs include blood pressure 82/44 mm Hg, heart rate 106 beats/minute, respiratory rate 28 breaths/minute, and temperature 103.6ºF (39.8ºC). Pertinent laboratory values include WBC 24 × 103 cells/mm3, Na 148 mEq/L, K 3.8 mEq/L, Cl 110 mEq/L, HCO3 19 mEq/L, SCr 1.9 mg/dL, and lactate 4.1 mmol/L. On physical examination, the patient is confused and lethargic with a Glasgow Coma Scale score of 12, and she weighs 68 kg. She is given 2 L of lactated Ringer solution, cultures are sent, and broad-spectrum antibiotics are being administered.

Question 5
An arterial blood gas shows pH 7.28, Paco2 32 mm Hg, and Pao2 56 mm Hg. Which most accurately describes her acid-base status?

A	Anion gap metabolic acidosis.
B	Non-anion gap metabolic acidosis.
C	Respiratory acidosis.
D	Respiratory alkalosis.
A

A - patient has a low PH and therefore has an acidosis (D is incorrect). Anion gap is 148 - (110+19) = 19 which is above 12 (the upper limit of normal) and therefore shows that anions are in the blood (B is incorrect). Paco2 is low indicating respiratory compensation for the primary metabolic disturbance (c is incorrect).

How well did you know this?
1
Not at all
2
3
4
5
Perfectly
6
Q

A 67-year-old woman presents to the ED with septic shock, presumably caused by a UTI. Her medical history is significant for hypertension, hyperlipidemia, and recurrent Escherichia coli UTIs. Pertinent vital signs include blood pressure 82/44 mm Hg, heart rate 106 beats/minute, respiratory rate 28 breaths/minute, and temperature 103.6°F (39.8°C). Pertinent laboratory values include WBC 24 × 103 cells/mm3, Na 148 mEq/L, K 3.8 mEq/L, Cl 110 mEq/L, HCO3 19 mEq/L, SCr 1.9 mg/dL, and lactate 4.1 mmol/L. On physical examination, the patient is confused and lethargic with a Glasgow Coma Scale score of 12, and she weighs 68 kg. She is given 2 L of lactated Ringer solution, cultures are sent, and broad-spectrum antibiotics are being administered.

Question 6
After administration of crystalloids, the patient’s blood pressure is 86/46 mm Hg. Which is the most appropriate intervention to implement next?

A	Initiate dobutamine 2 mcg/kg/minute because she has cardiogenic shock.
B	Initiate vasopressin 0.03 units/minute because she has distributive shock.
C	Initiate norepinephrine 0.05 mcg/kg/minute because she has distributive shock.
D	No additional intervention is needed because the patient improved with fluid administration.
A

C - initiate norepinephrine 0.05 mcg/kg/minute because she has distributive shock, is correct. The patient is in septic shock and remains hypotensive despite adequate fluid administration (Answer D is incorrect). Septic shock is a type of distributive shock, and dobutamine, used for cardiogenic shock, is not indicated at this time (Answer A is incorrect). Although vasopressin could be added to norepinephrine in septic shock, it is not recommended as the initial vasopressor of choice (Answer B is incorrect).

How well did you know this?
1
Not at all
2
3
4
5
Perfectly
7
Q

A 67-year-old woman presents to the ED with septic shock, presumably caused by a UTI. Her medical history is significant for hypertension, hyperlipidemia, and recurrent Escherichia coli UTIs. Pertinent vital signs include blood pressure 82/44 mm Hg, heart rate 106 beats/minute, respiratory rate 28 breaths/minute, and temperature 103.6ºF (39.8ºC). Pertinent laboratory values include WBC 24 × 103 cells/mm3, Na 148 mEq/L, K 3.8 mEq/L, Cl 110 mEq/L, HCO3 19 mEq/L, SCr 1.9 mg/dL, and lactate 4.1 mmol/L. On physical examination, the patient is confused and lethargic with a Glasgow Coma Scale score of 12, and she weighs 68 kg. She is given 2 L of lactated Ringer solution, cultures are sent, and broad-spectrum antibiotics are being administered.

Question 7
In which scenario would hydrocortisone most be indicated for this patient?

A	Result from a corticotropin stimulation test yielding a cortisol change of 7 mcg/mL.
B	Norepinephrine 0.4 mcg/kg/minute, vasopressin 0.03 units/minute, and blood pressure 88/42 mm Hg.
C	Norepinephrine 0.05 mcg/kg/minute and blood pressure 94/52 mm Hg.
D	Hydrocortisone not indicated in septic shock.
A

B - norepinephrine 0.4 mcg/kg/minute, vasopressin 0.03 units/minute, and blood pressure 88/42 mm Hg, is correct. The patient remains hypotensive despite increased doses of vasopressor therapy (Answer D is incorrect). The Surviving Sepsis Campaign guidelines recommend against using hydrocortisone in patients for whom hemodynamic stability was restored after adequate fluid resuscitation and vasopressor therapy (Answer C is incorrect). The corticotropin stimulation test is not recommended to identify patients who should receive hydrocortisone (Answer A is incorrect).

How well did you know this?
1
Not at all
2
3
4
5
Perfectly
8
Q

R.M. is a 68-year-old woman (height 65 inches, weight 65 kg) with newly diagnosed atrial fibrillation. Her medical history includes dyslipidemia, hypertension, peptic ulcer disease 2 years ago, chronic kidney disease, and hypothyroidism. Her current medications include metoprolol tartrate 50 mg twice daily, atorvastatin 20 mg once daily, chlorthalidone 25 mg once daily, dronedarone 400 mg twice daily, omeprazole 20 mg once daily, and levothyroxine 0.25 mg once daily. Her laboratory values include K 4.2 mEq/L, blood glucose 98 mg/dL, and calculated CrCl 40 mL/minute/1.73 m2. Her vital signs today include heart rate 78 beats/minute and blood pressure 128/78 mm Hg.

Question 8
Which is the most appropriate regimen for reducing R.M.’s risk of stroke?

A	Dabigatran 75 mg twice daily.
B	Rivaroxaban 20 mg once daily.
C	Apixaban 2.5 mg twice daily.
D	Edoxaban 60 mg once daily.
A

A - Patients with a CrCl of less than 50 mL/minute/1.73 m2 should have their rivaroxaban dose decreased from 20 mg daily to 15 mg daily, making Answer B incorrect. The apixaban dose is only decreased to two of three criteria (SCr greater than 1.5 mg/dL, age 80 or older, or weight 60 kg or less). Although no SCr value is given, neither the patient’s age nor her weight would qualify for a dose reduction of apixaban, making Answer C incorrect. Answer D is incorrect because the edoxaban dose should be reduced with moderate renal insufficiency or with the use of dronedarone. The 75-mg twice-daily dabigatran dose is used with a CrCl of 15–30 mL/minute/1.73 m2 and with a CrCl of 30–50 mL/minute/1.73 m2 with the use with dronedarone or ketoconazole. Because this patient has moderate renal insufficiency and is taking dronedarone, Answer A is correct.

How well did you know this?
1
Not at all
2
3
4
5
Perfectly
9
Q

R.M. is a 68-year-old woman (height 65 inches, weight 65 kg) with newly diagnosed atrial fibrillation. Her medical history includes dyslipidemia, hypertension, peptic ulcer disease 2 years ago, chronic kidney disease, and hypothyroidism. Her current medications include metoprolol tartrate 50 mg twice daily, atorvastatin 20 mg once daily, chlorthalidone 25 mg once daily, dronedarone 400 mg twice daily, omeprazole 20 mg once daily, and levothyroxine 0.25 mg once daily. Her laboratory values include K 4.2 mEq/L, blood glucose 98 mg/dL, and calculated CrCl 40 mL/minute/1.73 m2. Her vital signs today include heart rate 78 beats/minute and blood pressure 128/78 mm Hg.

Question 9
R.M. was initiated on apixaban, and 6 months later, she is in a motor vehicle accident with major life-threatening bleeding. Which is the most appropriate treatment for her bleeding episode?

A	Idarucizumab 5 g intravenously.
B	8 units of fresh frozen plasma with vitamin K 10 mg intravenously.
C	Andexanet 400-mg intravenous bolus followed by 4 mg/minute for 2 hours.
D	4-factor prothrombin complex concentrate 100 units/kg.
A

C - Andexanet alfa is the reversal agent for apixaban

How well did you know this?
1
Not at all
2
3
4
5
Perfectly
10
Q

A new anticoagulant was evaluated in a clinical trial to determine whether it provided efficacy similar to warfarin for reducing the risk of stroke in patients with atrial fibrillation. The study was designed to be a randomized, double-blind, noninferiority trial set to enroll 12,000 patients. Noninferiority was considered proven if there was no more than a 15% excess of events in the new anticoagulant arm of the study. The final results of the trial showed a hazard ratio of 0.96 and a 95% confidence interval of 0.80–1.18. Which is the most accurate interpretation of this clinical trial?

A	New anticoagulant was noninferior to warfarin for reducing stroke.
B	New anticoagulant was equivalent to warfarin for reducing stroke.
C	New anticoagulant was not noninferior to warfarin for reducing stroke.
D	New anticoagulant showed a 4% absolute reduction in stroke.
A

C - A noninferiority trial is designed to determine whether a treatment is not clinically inferior to an existing therapy. This trial allowed for up to a 15% excess of events compared with warfarin therapy, which meant the upper bounds of the 95% confidence interval could not exceed 1.15. In this study, the upper end of the confidence interval was 1.18, which exceeded the study’s preset margin of noninferiority. Even though the hazard ratio was less than 1, the new anticoagulant did not prove to be noninferior to warfarin therapy, making Answer C correct and Answer A incorrect. Answer B is incorrect because a hazard ratio crossing 1 in a noninferiority trial does not prove equivalence of the two agents studied. The absolute end point event rates are not provided; therefore, the absolute difference cannot be calculated, making Answer D incorrect.

How well did you know this?
1
Not at all
2
3
4
5
Perfectly
11
Q

A 42-year-old man presents to the physician with shortness of breath and difficulty breathing over the past 6 months. His medical history is significant for hypertension. He denies any history of cigarette smoking or significant exposure. His family history is significant for chronic obstructive pulmonary disease (COPD) in his father (diagnosed at age 40) and severe asthma in a sister. He describes his symptoms as being worse at night than during the day, and they seem to worsen when he has spent time outside. He has tried his sister’s albuterol inhaler without improvement in symptoms. He denies any acutely worsening symptoms that have led to medical treatment other than “being tired of having a hard time breathing.” Spirometry testing reveals a forced expiratory volume in 1 second (FEV1) of 75% of predicted and an FEV1/forced vital capacity of 68%, with no response to a bronchodilator during testing. Which initial treatment approach is most appropriate for this patient?

A	Inhaled corticosteroid–containing treatment is essential because the patient presents with clinical features of asthma.
B	Inhaled corticosteroid–containing treatment is essential in addition to a long-acting bronchodilator according to the GOLD recommendations because the patient presents with clinical features of both asthma and COPD.
C	The patient presents with clinical features of acute bronchitis and should not be treated at this time.
D	The patient presents with clinical features of COPD and should be treated according to the GOLD recommendations.
A

B - The patient presents with clinical features of both asthma and COPD. The patient’s age, family history of COPD, and spirometry testing indicate COPD. However, his lack of exposure to risk factors, family history of asthma, and worsening of symptoms at nighttime and after exposure to allergens indicate asthma. Given that his symptoms are consistent with both asthma and COPD, the patient should be initiated on inhaled corticosteroid–containing treatment in addition to COPD treatments according to the GOLD recommendations, making Answer B correct and Answers A, C, and D incorrect

How well did you know this?
1
Not at all
2
3
4
5
Perfectly
12
Q

M.C. is a 42-year-old man who stepped on a nail a month ago. The nail penetrated his shoe and went deep into his left foot. He cleaned the puncture area well and applied antibiotic cream to the area for about a week. He now presents with pain and swelling in the left foot. An MRI of the foot reveals osteomyelitis with bone erosion. A bone biopsy with tissue culture is planned but not completed at this time. Which is the best empiric antibiotic regimen for M.C.?

A	Nafcillin for 2 weeks, followed by dicloxacillin to complete a 4-week course.
B	Cefepime for 6 weeks.
C	Vancomycin plus metronidazole for 6 weeks.
D	Levofloxacin for 2 weeks.
A

B - Although nafcillin is commonly used first line for osteomyelitis, Pseudomonas aeruginosa is often involved in foot puncture osteomyelitis. Therefore, a broader-spectrum antibiotic is required. In addition, oral antibiotics should only be used if certain characteristics are present (Answer A is incorrect). Cefepime is the best option for this patient. Cefepime has good activity against Pseudomonas and gram-positive organisms, both of which are possible causative organisms in this case. In addition, the length of therapy is appropriate (4–6 weeks) (Answer B is correct). Vancomycin and metronidazole would not have activity against Pseudomonas (Answer C is incorrect). Levofloxacin might not achieve adequate concentrations at the infection site to kill gram-positive organisms, and the length of therapy for this option is too short (Answer D is incorrect).

How well did you know this?
1
Not at all
2
3
4
5
Perfectly
13
Q

In a study reporting the occurrence of patient symptoms, participants were asked to rate the severity of their symptoms on a scale from 0 to 5 (0 = no symptoms, 5 = unbearable symptoms). Which measure of central tendency is best to describe patients’ responses to this question?

A	Mean.
B	Interquartile range.
C	Median.
D	Standard deviation (SD).
A

C - Ordered or Likert-type scales typically are not continuous. For example, the interval between 1 and 2 is not necessarily the same as that between 2 and 3. Although there is some controversy about a true Likert scale, using means and SDs to describe the central tendency and variability of these types of data are among the most common errors made in the literature (Answer A is incorrect). Answers B and D are incorrect because they are measures of spread or variability, not point estimates of the central tendency. Answer C, median, provides the most appropriate description of the central tendency of these types of data.

How well did you know this?
1
Not at all
2
3
4
5
Perfectly
14
Q

A 61-year-old woman with chronic obstructive pulmonary disease (COPD) was recently admitted to the hospital because of a moderate exacerbation. This is the patient’s third exacerbation in the past year. She is a current smoker (42 pack-years) and is not interested in quitting within the next 6 months. She verbalizes appropriate doses and frequencies of all inhalers, including umeclidinium/vilanterol once daily and albuterol metered dose inhaler (MDI) as needed. A medication adherence report shows proportion of days covered (PDC) as 99% and 98% for umeclidinium/vilanterol and albuterol MDI, respectively. Which educational strategy is most appropriate to help the patient improve COPD control and prevent exacerbations?

A	Ask the patient to demonstrate inhaler administration technique.
B	Provide education on nicotine replacement therapy to assist with a quit attempt.
C	Recommend COPD pharmacotherapy adjustments and assess patient understanding using the teach-back method.
D	Reinforce the dosing and frequency of current inhalers.
A

A - Studies have shown a significant relationship between poor inhaler use and worsening symptom control in patients with COPD. Regular education on inhaler technique is critically important and should always be assessed before assuming current therapy is inadequate (Answer A is correct; Answer C is incorrect). Although the patient would benefit from tobacco cessation, she is currently in the precontemplation stage. Given the patient’s current stage of change, recommending nicotine replacement therapy would not be appropriate at this time, making Answer B incorrect. The patient should be provided information, with permission, about the health consequences of smoking with the goal of moving her to the contemplation stage. Although reinforcement of dosing and frequency of inhaler use would be appropriate, the patient’s self-report of administering doses as prescribed in addition to high PDC does not support this as the main concern at this time (Answer D is incorrect).

How well did you know this?
1
Not at all
2
3
4
5
Perfectly
15
Q

A.R. is a 62-year-old white man (height 68 inches, weight 88 kg) with a history of hypertension, a non–ST-segment elevation myocardial infarction 8 months ago with two drug-eluting stents placed, chronic kidney disease (baseline SCr 2.7 mg/dL), New York Heart Association (NYHA) class II heart failure (HF) with a left ventricular ejection fraction of 30%, and chronic lower back pain. His current medications include enalapril 10 mg twice daily, furosemide 40 mg daily, atenolol 50 mg daily, aspirin 81 mg daily, clopidogrel 75 mg daily, atorvastatin 40 mg daily, and naproxen 550 mg daily. He is following up in the clinic today for management of his HF regimen. He has no new concerns. On examination, his breath sounds are clear, he has no evidence of edema, his blood pressure is 128/76 mm Hg, and his heart rate is 78 beats/minute. His laboratory values include Na 141 mEq/L, K 4.2 mEq/L, BUN 44 mg/dL, and SCr 2.7 mg/dL.

Question 15
Which additional medication changes would be best to make to A.R.’s regimen?

A	Discontinue clopidogrel.
B	Change naproxen to acetaminophen.
C	Change atorvastatin to simvastatin 40 mg daily.
D	Add losartan 50 mg daily.
A

B - A.R. recently had a myocardial infarction (MI) and received drug-eluting stents, so both clopidogrel (Answer A) and a high-intensity statin (Answer C) should be continued. Although losartan is a reasonable alternative to angiotensin-converting enzyme (ACE) inhibitor therapy in heart failure with reduced ejection fraction (HFrEF), angiotensin receptor blockers generally should not be used in combination with ACE inhibitors because of the increased risk of kidney injury and hyperkalemia (Answer D is incorrect). Naproxen is an NSAID, which can promote sodium and water retention, blunt diuretic response, and increase the risk of cardiovascular events and mortality in a patient with a history of HF and MI. Therefore, naproxen should be discontinued and changed to acetaminophen, which is considered safe in HF (Answer B is correct).

How well did you know this?
1
Not at all
2
3
4
5
Perfectly
16
Q

A.R. is a 62-year-old white man (height 68 inches, weight 88 kg) with a history of hypertension, a non–ST-segment elevation myocardial infarction 8 months ago with two drug-eluting stents placed, chronic kidney disease (baseline SCr 2.7 mg/dL), New York Heart Association (NYHA) class II heart failure (HF) with a left ventricular ejection fraction of 30%, and chronic lower back pain. His current medications include enalapril 10 mg twice daily, furosemide 40 mg daily, atenolol 50 mg daily, aspirin 81 mg daily, clopidogrel 75 mg daily, atorvastatin 40 mg daily, and naproxen 550 mg daily. He is following up in the clinic today for management of his HF regimen. He has no new concerns. On examination, his breath sounds are clear, he has no evidence of edema, his blood pressure is 128/76 mm Hg, and his heart rate is 78 beats/minute. His laboratory values include Na 141 mEq/L, K 4.2 mEq/L, BUN 44 mg/dL, and SCr 2.7 mg/dL.

Question 16
Which is the best recommendation for management of A.R.’s HF regimen?

A	Increase enalapril to 20 mg twice daily.
B	Add hydralazine/isosorbide dinitrate 1 tablet three times daily.
C	Add spironolactone 12.5 mg daily.
D	Change atenolol to metoprolol succinate.
A

D - Answer A is incorrect; this patient is already taking the target dose of enalapril and has achieved a goal blood pressure, so further increases are unnecessary. Answer B is incorrect because hydralazine/isosorbide dinitrate is recommended in patients self-described as African American who are already receiving optimal doses of angiotensin-converting enzyme (ACE) inhibitors and ß-blockers. Although spironolactone is a reasonable add-on therapy in patients with NYHA class II–IV heart failure with reduced ejection fraction (HFrEF) receiving ACE inhibitors and ß-blockers, this patient’s SCr is greater than 2.5 mg/dL, so aldosterone receptor antagonists should be avoided because of the risk of hyperkalemia (Answer C is incorrect). ß-Blockers are recommended in all patients with HFrEF in addition to ACE inhibitors unless contraindicated. However, only bisoprolol, carvedilol, and metoprolol succinate have been shown to improve mortality in HFrEF, making Answer D correct.

How well did you know this?
1
Not at all
2
3
4
5
Perfectly
17
Q

K.P. is a 39-year-old woman with constipation-predominant irritable bowel syndrome (IBS-C). She also has a history of heartburn, for which she is prescribed famotidine 20 mg twice daily, but has difficulty remembering the evening dose. K.P. recently had a reduction in stool frequency despite using psyllium 1 scoop daily and pegylated interferon 17 g daily. She reports that her last bowel movement was 3 days ago. She also has concerns of abdominal discomfort and bloating. Which is the best recommendation for K.P. at this time?

A	Increase the psyllium dose to 1 scoop twice daily.
B	Add plecanatide 3 mg daily.
C	Add tegaserod 6 mg twice daily.
D	Add lubiprostone 8 mcg twice daily.
A

B - This patient needs additional therapy for IBS-C, for which secretagogues such as lubiprostone, linaclotide, and plecanatide are effective. She reports difficulty remembering her second daily dose of famotidine; thus, lubiprostone, which is administered twice daily, might also be difficult for her to take consistently (Answer D is incorrect). A medication dosed once per day such as plecanatide would be a better option for her (Answer B is correct). Increasing the psyllium dose could exacerbate her abdominal discomfort and bloating (Answer A is incorrect). Tegaserod is only available on a limited basis for emergency use because of its risk of cardiovascular adverse effects (Answer C is incorrect).

How well did you know this?
1
Not at all
2
3
4
5
Perfectly
18
Q

A 28-year-old man with schizophrenia has been stable on clozapine for 1 year. He is brought to the ED after a seizure. He is somnolent and unable to walk or stand. He has a smoking history of 1 pack/day for 8 years. He started varenicline 2 weeks ago to help him stop smoking. His last cigarette was 1 week ago. He drinks 1–2 cups of coffee daily and had 1 beer last night. Which is the most likely cause of this patient’s current clinical picture?

A	Caffeine intake.
B	Combining alcohol with varenicline.
C	Combining clozapine and varenicline.
D	Smoking cessation
A

D - Clozapine is metabolized by CYP1A2 and Smoking is an inducer, when the patient stopped smoking his clozapine concentrations increased.

How well did you know this?
1
Not at all
2
3
4
5
Perfectly
19
Q

A 42-year-old woman wants to stop smoking. She has a 22 pack-year history and currently smokes 1 pack/day. She has her first cigarette immediately upon awakening. Her medical history includes anorexia nervosa, schizophrenia, nasal polyps, and asthma. Her current medications include risperidone long-acting injectable every 2 weeks, budesonide 160 mg/formoterol fumarate 4.5 mg 2 puffs twice daily, and albuterol HFA every 4–6 hours as needed (using twice weekly). This is her first quit attempt, and she is interested in a highly effective product. Which treatment would be best to recommend for her?

A	Bupropion.
B	Nicotine gum.
C	Nicotine nasal spray.
D	Varenicline.
A

D - Varenicline is the best option for this woman (Answer D is correct). The latest data analyses suggest that varenicline is not as likely to cause neuropsychiatric symptoms as once thought, and a review of existing trials shows that patients with schizophrenia are not more likely to develop neuropsychiatric symptoms. The advantages of abstinence from smoking are believed to outweigh any risk of developing psychiatric symptoms. Bupropion is contraindicated in patients with eating disorders, and this patient has a history of anorexia nervosa (Answer A is incorrect). Nicotine gum monotherapy is less likely to be effective for smoking cessation than if it were combined with nicotine patches or other modalities (Answer B is incorrect). Nicotine nasal spray should be avoided in patients with nasal polyps (Answer C is incorrect).

How well did you know this?
1
Not at all
2
3
4
5
Perfectly
20
Q

S.K. is an 18-year-old woman brought to your ED for a seizure that has lasted 10 minutes. She has no history of epilepsy and does not take any medications. Which is best for treatment of S.K.’s seizure?

A	No treatment.
B	Levetiracetam.
C	Lorazepam.
D	Valproate.
A

C - Status epilepticus is defined as a prolonged seizure lasting more than 5 minutes. Because this seizure has lasted for 10 minutes, treatment is required, making Answer A incorrect. Levetiracetam (Answer B) can be used if prior treatments fail, but not in the initial treatment of status epilepticus. Similarly, valproate (Answer D) can be used in refractory status epilepticus. In addition, valproate is not ideal for a woman who might be pregnant. Lorazepam (Answer C) is used as initial treatment in status epilepticus to control seizures.

How well did you know this?
1
Not at all
2
3
4
5
Perfectly
21
Q

The Centers for Medicare & Medicaid Services (CMS) implements quality initiatives to ensure quality health care for patients receiving Medicare. This has produced the National Hospital Inpatient Quality Measures, which are aligned with The Joint Commission. Which best depicts the disease state that is part of these quality measures for 30-day risk-standardized mortality?

A	Hip replacement surgery.
B	Hypertensive crises.
C	Stroke.
D	Atrial fibrillation.
A
C - Acute Myocardial Infarction
Heart Failure
Pneumonia
Chronic Obstructive Pulmonary Disease (COPD)
Coronary Artery Bypass Graft (CABG)
Stroke

are the measures. Hip/knee and readmission is the 30 day readmission measure

How well did you know this?
1
Not at all
2
3
4
5
Perfectly
22
Q

Which of the following infections is a notifiable disease and should be reported to the Centers for Disease Control and Prevention when diagnosed?

A	ESBL producing Klebsiella pneumoniae.
B	Vancomycin resistant Enterococcus faecium.
C	Listeria monocytogenes.
D	Clostridioides difficile.
A

C - CDC notifiable diseases are considered of significant public health importance and help the CDC effectively follow trends in infections or disease outbreaks. Many of the more common bacterial infections are not considered “notifiable.” Although ESBL producing organisms are concerning from an antibiotic resistance perspective, they are not notifiable (Answer A is incorrect). Vancomycin intermediate or resistant Staphylococcus aureus is notifiable but not VRE (Answer B is incorrect). C. difficile infections are important to track from a local health system perspective, but when diagnosed the CDC does not need to be notified (Answer D is incorrect). Since L. monocytogenes infections can originate from contaminated foods, is it important for the CDC to track these infections to potentially recall any contaminated products and prevent a widespread outbreak.

How well did you know this?
1
Not at all
2
3
4
5
Perfectly
23
Q

A 48-year-old woman is brought to your institution by ambulance after being found unconscious at home. She was intubated in the field, and laboratory values obtained in the ambulance show a sodium concentration of 183 mg/dL. She seized en route and received 2 mg of intravenous lorazepam. When the family arrives, you learn that she has a history of diabetes insipidus and was recently changed from intranasal to oral desmopressin. Which is the best initial step in this patient’s treatment?

A	Sodium chloride 0.9%.
B	Sodium chloride 0.45%.
C	Sodium chloride 0.9% plus desmopressin 2 mcg intravenously.
D	Sodium chloride 0.45% plus desmopressin 0.1 mg orally.
A

C - Given this patient’s history of diabetes insipidus, she needs desmopressin as part of her treatment of hypernatremia. Because Answers A and B provide only fluid and not desmopressin, they are both incorrect. The patient is intubated and was found unconscious, so she should receive desmopressin intravenously rather than orally. In addition, the sodium content of sodium chloride 0.45% would decrease the patient’s sodium too quickly (Answer D). Sodium chloride 0.9% and intravenous desmopressin is the best choice for this patient (Answer C)

How well did you know this?
1
Not at all
2
3
4
5
Perfectly
24
Q

A patient with chronic kidney disease category G3a is scheduled for a cardiac catheterization for evaluation of possible coronary artery disease. Which treatment is best for preventing contrast-associated nephropathy?

A	Acetylcysteine orally.
B	0.9% sodium chloride intravenously.
C	Furosemide intravenously.
D	Sodium bicarbonate intravenously.
A

B - The best treatment for preventing contrast-associated nephropathy is volume expansion with intravenous saline (Answer B is correct). Although diuretics may increase urinary output, there is no evidence that they decrease the risk of nephropathy, and they may worsen nephropathy if volume status is worsened (Answer C is incorrect). Although early data analyses suggested a benefit from intravenous sodium bicarbonate, more recent information has shown no benefit (Answer D is incorrect). Although acetylcysteine has been used because it is safe and inexpensive, current data analyses suggest this agent does not prevent contrast-associated acute kidney injury (Answer A is incorrect).

How well did you know this?
1
Not at all
2
3
4
5
Perfectly
25
Q

B.L. is a 68-year-old woman (height 64 inches, weight 58 kg) with a medical history of hypertension and heart failure (left ventricular ejection fraction 35%). Her current medications include lisinopril 20 mg daily and metoprolol succinate 50 mg daily. She presents to the clinic today with concerns about increased shortness of breath when walking and a “fluttering in my chest.” Her blood pressure today is 134/82 mm Hg and heart rate is 98 beats/minute. The patient has no other concerns. On examination, her breath sounds are clear, and she has no pitting edema. An ECG reveals atrial fibrillation (AF). Her laboratory results are all within normal limits, including SCr 1.2 mg/dL and BNP 84 pg/mL. Which is the best pharmacologic approach for managing B.L.’s symptoms today?

A	Increase metoprolol succinate to 100 mg daily.
B	Add diltiazem CD 240 mg daily.
C	Add dronedarone 400 mg twice daily.
D	No changes are needed; B.L.’s heart rate is at goal.
A

A - B.L. has been given a diagnosis of AF and is symptomatic, so her heart rate goal is less than 80 beats/minute. A heart rate goal of less than 110 beats/minute would be reasonable only if B.L. were asymptomatic, making Answer D incorrect. An initial rate control strategy is preferred. Furthermore, dronedarone would be an inappropriate antiarrhythmic agent for this patient, given her history of heart failure with reduced ejection fraction (HFrEF) (Answer C). Both metoprolol succinate and diltiazem could improve the patient’s heart rate, but diltiazem is not recommended in patients with HFrEF because of its negative inotropic properties when used in HFrEF (Answer B). Increasing the dose of metoprolol succinate should help improve B.L.’s heart rate, making Answer A correct. In addition, metoprolol succinate is not yet at the target dose for her HFrEF and should be titrated to maximally tolerated doses or a goal of 200 mg daily.

How well did you know this?
1
Not at all
2
3
4
5
Perfectly
26
Q

A 25-year-old woman with a history of recurrent major depressive disorder has been admitted to the inpatient psychiatric unit after being treated for an overdose of acetaminophen and alprazolam (not her prescription). She has obesity and insomnia. She had anorgasmia while taking fluoxetine and worsened insomnia while taking venlafaxine. Which medication would be most appropriate to initiate in this patient?

A	Bupropion.
B	Nortriptyline.
C	Paroxetine.
D	Vilazodone.
A

D - The antidepressant prescribed for this patient should be relatively safe in overdose. Tricyclic antidepressants such as nortriptyline can be fatal if too much is taken. Adverse effects include torsades de pointes and seizures, and overdoses are often fatal (Answer B is incorrect). Paroxetine is associated with a high rate of sexual dysfunction and can cause weight gain (Answer C is incorrect). Although bupropion does not cause sexual dysfunction, it can cause seizures in a dose-dependent fashion. This makes bupropion a poor choice in a patient with suicidal ideation, and it should be avoided in patients with seizure disorder. Bupropion is also activating and could worsen the patient’s insomnia (Answer A is incorrect). Vilazodone has a lower incidence of sexual dysfunction and is relatively safe in overdose, making it the best choice (Answer D is correct).

How well did you know this?
1
Not at all
2
3
4
5
Perfectly
27
Q

Enteral nutrition is being initiated in a 72-year-old man (height 69 inches, weight 72 kg) in your ICU after a stroke. He was well nourished before admission, and his laboratory values are within normal limits. Which would be the most appropriate goals in this patient?

A	1650 kcal plus 72 g of protein.
B	1725 kcal plus 144 g of protein.
C	2000 kcal plus 101 g of protein.
D	2200 kcal plus 86 g of protein.
A

C - The patient requires 25–30 kcal/kg/day and 1.3–1.5 g/kg/day of protein

How well did you know this?
1
Not at all
2
3
4
5
Perfectly
28
Q

M.H., an 83-year-old woman, presents to her ambulatory care clinic accompanied by her daughter, who reports she no longer feels comfortable leaving her mother alone because of her mother’s “increasing forgetfulness.” The patient reports increasing pain at her knee joints, contributing to difficulty walking. The patient’s medical history is significant for type 2 diabetes, hypertension, coronary artery disease, congestive heart failure, and osteoarthritis. She takes the following medications: aspirin 81 mg daily, atorvastatin 20 mg once daily, lisinopril 20 mg daily, furosemide 20 mg daily, potassium chloride 20 mEq daily, carvedilol 12.5 mg twice daily, metformin 1000 mg twice daily, and glipizide extended release 5 mg daily. Her Mini-Mental State Examination (MMSE) score is 20/30, and her Geriatric Depression Scale score is 3/15. Blood tests obtained last week showed a normal basic metabolic panel, except for a fasting plasma glucose of 65 mg/dL. Her A1C is 7.6%. A urinalysis is negative. No nutritional deficiencies are noted. The patient’s blood pressure is 130/80 mm Hg and heart rate is 60 beats/minute. She is given a diagnosis of Alzheimer disease (AD), and initiation of a cholinesterase inhibitor is considered.

Question 29
Although initiating a cholinesterase inhibitor for M.H. might improve her cognition, which adverse effect would it most likely cause?

A	Vivid dreams.
B	Depression.
C	Hallucinations.
D	Diarrhea.
A

D - Cholinesterase inhibitors prevent the breakdown of acetylcholine, resulting in increased cholinergic activity. Cholinesterase inhibitors in older adults with dementia can cause or worsen diarrhea because of increased cholinergic activity (Answer D is correct). Vivid dreams, depression, and hallucinations are other possible adverse effects to consider monitoring for when initiating cholinesterase inhibitor therapy but are less likely to occur than GI adverse effects (Answers A, B, and C are incorrect).

How well did you know this?
1
Not at all
2
3
4
5
Perfectly
29
Q

M.H., an 83-year-old woman, presents to her ambulatory care clinic accompanied by her daughter, who reports she no longer feels comfortable leaving her mother alone because of her mother’s “increasing forgetfulness.” The patient reports increasing pain at her knee joints, contributing to difficulty walking. The patient’s medical history is significant for type 2 diabetes, hypertension, coronary artery disease, congestive heart failure, and osteoarthritis. She takes the following medications: aspirin 81 mg daily, atorvastatin 20 mg once daily, lisinopril 20 mg daily, furosemide 20 mg daily, potassium chloride 20 mEq daily, carvedilol 12.5 mg twice daily, metformin 1000 mg twice daily, and glipizide extended release 5 mg daily. Her Mini-Mental State Examination (MMSE) score is 20/30, and her Geriatric Depression Scale score is 3/15. Blood tests obtained last week showed a normal basic metabolic panel, except for a fasting plasma glucose of 65 mg/dL. Her A1C is 7.6%. A urinalysis is negative. No nutritional deficiencies are noted. The patient’s blood pressure is 130/80 mm Hg and heart rate is 60 beats/minute. She is given a diagnosis of Alzheimer disease (AD), and initiation of a cholinesterase inhibitor is considered.

Question 30
Which patient parameter most justifies initiating a cholinesterase inhibitor for M.H. right now?

A	MMSE score 20/30.
B	Geriatric Depression Scale score 3/15.
C	Increasing forgetfulness reported by the patient’s daughter.
D	A1C greater than 6.5%.
A

A - For mild to moderate AD, a cholinesterase inhibitor is initially the most appropriate option for slowing cognitive decline; this patient’s MMSE score of 20/30 and corresponding diagnosis of mild dementia warrant initiating cholinesterase inhibitor therapy (Answer A is correct). The Geriatric Depression Scale score does not correspond with justification of use of cholinesterase inhibitor therapy (Answer B is incorrect). Further exploration of the daughter’s reports of her mother’s increasing forgetfulness would help classify the severity of her mother’s cognitive impairment and AD diagnosis but, alone, would not be adequate to justify the need for a cholinesterase inhibitor (Answer C is incorrect). Finally, A1C is related to the patient’s diagnosis of diabetes and is not relevant to the question regarding initiation of a cholinesterase inhibitor (Answer D is incorrect).

If MMSE is lower than 24 it means decline in cognitive function.

How well did you know this?
1
Not at all
2
3
4
5
Perfectly
30
Q

M.H., an 83-year-old woman, presents to her ambulatory care clinic accompanied by her daughter, who reports she no longer feels comfortable leaving her mother alone because of her mother’s “increasing forgetfulness.” The patient reports increasing pain at her knee joints, contributing to difficulty walking. The patient’s medical history is significant for type 2 diabetes, hypertension, coronary artery disease, congestive heart failure, and osteoarthritis. She takes the following medications: aspirin 81 mg daily, atorvastatin 20 mg once daily, lisinopril 20 mg daily, furosemide 20 mg daily, potassium chloride 20 mEq daily, carvedilol 12.5 mg twice daily, metformin 1000 mg twice daily, and glipizide extended release 5 mg daily. Her Mini-Mental State Examination (MMSE) score is 20/30, and her Geriatric Depression Scale score is 3/15. Blood tests obtained last week showed a normal basic metabolic panel, except for a fasting plasma glucose of 65 mg/dL. Her A1C is 7.6%. A urinalysis is negative. No nutritional deficiencies are noted. The patient’s blood pressure is 130/80 mm Hg and heart rate is 60 beats/minute. She is given a diagnosis of Alzheimer disease (AD), and initiation of a cholinesterase inhibitor is considered.

Question 31
Which would be the most appropriate treatment for M.H.’s OA knee pain?

A	Ibuprofen 200 mg four times daily.
B	Diclofenac gel 1% applied four times daily to the affected joints.
C	Tramadol 50 mg three times daily as needed for pain.
D	Acetaminophen 650 mg three times daily.
A

B - An initial trial of a topical NSAID is reasonable for patients with OA pain because the efficacy of topical NSAIDs is similar to systemic NSAIDs without most of the adverse effects (Answer B is correct). Ibuprofen would be an alternative when topical NSAIDs have failed (Answer A is incorrect). Given the patient’s history of coronary artery disease and heart failure, as well as the fact that she takes aspirin daily, using ibuprofen would also be discouraged. As-needed tramadol should be used cautiously in older patients and only when NSAIDs are contraindicated or ineffective and the pain is severe (Answer C is incorrect). Acetaminophen at doses of less than 3 g/day is a potential alternative, but many studies have shown minimal efficacy, and long-term use requires monitoring for hepatotoxicity (Answer D is incorrect).

How well did you know this?
1
Not at all
2
3
4
5
Perfectly
31
Q

A patient with type 2 diabetes was recently initiated on a basal-bolus insulin regimen. The current basal dose is insulin glargine 20 units once daily. The current bolus dose is insulin aspart 7 units before breakfast, lunch, and dinner. When implementing a correctional insulin dose strategy to minimize hyperglycemic excursions, which is the best estimate, in milligrams per deciliter, of how much 1 unit of insulin aspart would reduce this patient’s plasma glucose concentration?

A	11.
B	22.
C	31.
D	44.
A

D - To estimate how much in milligrams per deciliter 1 unit of a rapid-acting insulin would decrease an individual patient’s glucose concentration, the rule of 1800 applies. This patient’s total daily insulin needs, adding the current basal and bolus insulin regimen, calculates as 41. Dividing the total daily insulin needs into 1800 is 44. Hence, 1 unit of insulin aspart would be expected to decrease a glucose concentration by 44 mg/dL (Answer D is correct). Answers A, B, and C would decrease glucose concentrations by 163, 82, and 58 mg/dL, respectively.

How well did you know this?
1
Not at all
2
3
4
5
Perfectly
32
Q

A 78-year-old man with chronic obstructive pulmonary disease (COPD) has been having worsening symptoms of cough, sputum production, and shortness of breath over the past 2 months. He has had two documented exacerbations in the past 9 months, with the most recent requiring hospitalization 1 month ago. He currently has moderate symptoms with most activities. His CAT (COPD Assessment Test) score is 14 today. His most recent spirometry reveals a forced expiratory volume in 1 second (FEV1) of 48% of predicted. At last check, his eosinophil count was 175 cells/microliter. His current treatment for COPD is indacaterol/glycopyrrolate 27.5/15.6 mcg 1 capsule inhaled twice daily and albuterol 100 mcg 1 puff four times daily as needed for shortness of breath/wheezing.

Question 33
Two months ago, the patient began the treatment regimen recommended, and his symptoms have improved. Today, he presents to the ED with concerns of increased trouble breathing, cough, and sputum production with a purulent color. On examination, his respiratory rate is 22 breaths/minute, SaO2 is 89%, and CrCl is 45 mL/minute/1.73 m2. He is alert and oriented × 3. After receiving supplemental oxygen and albuterol 2.5 mg by nebulizer, his shortness of breath is improving. Which is the most appropriate regimen to initiate today?

A	Prednisone 40 mg orally once daily for 5 days.
B	Prednisone 40 mg orally once daily for 14 days.
C	Prednisone 40 mg orally once daily for 5 days and antibiotic guided by sputum culture results.
D	Prednisone 40 mg orally once daily for 14 days and antibiotic guided by sputum culture results.
A

C - The patient is currently experiencing a severe COPD exacerbation, given his respiratory rate and FEV1. He is being treated in the ED appropriately. Given the presence of the three cardinal symptoms (dyspnea, increased sputum volume and increased sputum purulence antibiotics are indicated (Answers A and B are incorrect). Systemic corticosteroids given for 5 days are noninferior to longer 14-day courses; therefore, a shorter duration should be used (Answers B and D are incorrect). Given that his last exacerbation requiring hospitalization was 3 months ago and he has severe COPD (FEV1 less than 50%), he is at risk of Pseudomonas. A sputum culture should be obtained to determine the presence of gram-negative bacteria or resistant pathogens to guide antibiotic treatment decisions (Answer C is correct).

How well did you know this?
1
Not at all
2
3
4
5
Perfectly
33
Q

A 78-year-old man with chronic obstructive pulmonary disease (COPD) has been having worsening symptoms of cough, sputum production, and shortness of breath over the past 2 months. He has had two documented exacerbations in the past 9 months, with the most recent requiring hospitalization 1 month ago. He currently has moderate symptoms with most activities. His CAT (COPD Assessment Test) score is 14 today. His most recent spirometry reveals a forced expiratory volume in 1 second (FEV1) of 48% of predicted. At last check, his eosinophil count was 175 cells/microliter. His current treatment for COPD is indacaterol/glycopyrrolate 27.5/15.6 mcg 1 capsule inhaled twice daily and albuterol 100 mcg 1 puff four times daily as needed for shortness of breath/wheezing.

Question 34
Which is the most appropriate maintenance medication adjustment to initiate today?

A	Add roflumilast 500 mcg by mouth once daily.
B	Add tiotropium 2.5 mcg 2 puffs once daily.
C	Replace with fluticasone propionate/salmeterol 113/14 mcg 1 puff twice daily.
D	Replace with fluticasone furoate/umeclidinium/vilanterol 100/62.5/25 mcg 1 puff once daily.
A

Given the slow worsening of the patient’s symptoms over the past 2 months, treatment intensification should be recommended. According to the GOLD guidelines, therapy adjustments should be based on the predominant trait of dyspnea or exacerbations. Given this patient’s exacerbation history and recent hospitalization, the exacerbation pathway should be followed. Therapy adjustments should be made on the basis of current maintenance therapy. Because this patient is currently taking a combination long-acting muscarinic antagonist (LAMA)/long-acting ß2-agonist (LABA) and has an eosinophil count greater than 100 cells/microliter, adding an inhaled corticosteroid (ICS) is recommended (triple therapy with a LAMA/LABA/ICS), making Answer D correct. Answer A is incorrect because roflumilast should only be considered with an FEV1 of less than 50%, chronic bronchitis, and eosinophils less than 100 cells/microliter. Answer B is incorrect because this would be stepping down treatment, given that the patient is currently taking LAMA/LABA therapy. Answer C is incorrect because this option does not include a LAMA

How well did you know this?
1
Not at all
2
3
4
5
Perfectly
34
Q

A 3-year-old boy presented to your pediatric clinic yesterday with fever, decreased enteral intake, irritability, and tugging at his ears. He was given a diagnosis of acute otitis media (AOM) in both ears. A delayed antibiotic-prescribing strategy was decided on, and the patient was sent home with instructions for ibuprofen as needed. The boy’s father has brought him back to your clinic today because he seems more restless, and the fever is inadequately controlled with ibuprofen alone. The patient has a history of receiving amoxicillin for otitis media about 6 months ago. Which recommendation is best to treat this patient?

A	Administer cefdinir oral solution at 14 mg/kg/day for 7 days.
B	Administer ceftriaxone 50 mg/kg intravenously for 3 days.
C	Administer amoxicillin oral solution at 90 mg/kg/day for 7 days.
D	Continue to watch and wait until the 48–72 hours have passed.
A

C - Delayed antibiotic prescribing is acceptable for children older than 6 months. For children older than 2 years, this approach is recommended only if the symptoms are mild and there is no otorrhea. This approach is also acceptable for bilateral AOM if the child is older than 2 years. For patients in whom delayed antibiotic prescribing is initiated, an antibiotic should be prescribed if symptoms worsen or if there is no improvement in 48–72 hours. In this case, the symptoms appear to be worse, so further treatment should not be delayed (Answer D is incorrect). Cefdinir and ceftriaxone may be considered in patients whose first-line treatment regimens have failed, but this patient has not yet been initiated on antibiotics (Answers A and B are incorrect). Cefdinir would have been reasonable, however, if the patient had a true allergy to penicillins. The American Academy of Pediatrics recommends high-dose amoxicillin (80–100 mg/kg/day) as first-line therapy for AOM (Answer C is correct).

How well did you know this?
1
Not at all
2
3
4
5
Perfectly
35
Q

A 55-year-old man with a history of hypertension, dyslipidemia, and coronary artery disease had a drug-eluting stent placed 6 months ago. He presents today with a non–ST-segment elevation myocardial infarction (NSTEMI) and a high thrombolysis in myocardial infarction (TIMI) risk score. Inpatient therapies include aspirin 81 mg orally daily, ticagrelor 180 mg (single oral dose), and an unfractionated heparin intravenous infusion. After 12 hours, there is resolution of ECG changes, but intermittent chest pain remains. Coronary angiography reveals worrisome three-vessel disease. He is scheduled for urgent coronary artery bypass grafting (CABG) surgery. Which is the best management strategy, given his recent antiplatelet administration?

A	Discontinue ticagrelor; go ahead with surgery after 24 hours; continue aspirin.
B	Recommend that surgery be postponed for 7 days; discontinue ticagrelor and aspirin.
C	Recommend that surgery be postponed for 5 days; discontinue ticagrelor and continue aspirin.
D	Go ahead with surgery, and continue both ticagrelor 90 mg twice daily and aspirin
A

A - This patient has intermittent angina and presents with a high TIMI risk score, making him at high risk of a recurrent myocardial infarction at this time. The U.S. guidelines for treating patients undergoing urgent CABG surgery recommend discontinuing clopidogrel or ticagrelor for at least 24 hours in the setting of urgent CABG. For elective CABG, clopidogrel and ticagrelor should be discontinued for at least 5 days and prasugrel for at least 7 days before surgery. In addition, this patient had a drug-eluting stent placed 6 months ago. Guidelines recommend continuing a P2Y12 inhibitor for patients who had a stent placed less than 12 months ago; for patients at high risk of ischemic events, guidelines recommend continuing aspirin (Answer A is correct). Answers B and C because this is not an elective procedure. Answer B is also incorrect because aspirin should be continued. Answer D is incorrect because the P2Y12 inhibitor should be discontinued for at least 24 hours to minimize the risk of bleeding from CABG.

How well did you know this?
1
Not at all
2
3
4
5
Perfectly
36
Q

M.K. is a 59-year-old white woman with a medical history of hypertension, type 2 diabetes, and stable ischemic heart disease. Her current medications include aspirin 81 mg daily, metformin 500 mg twice daily, rosuvastatin 20 mg daily, amlodipine 5 mg daily, and metoprolol tartrate 25 mg twice daily. She presents to the clinic today with concerns about continued chest pain when she works in her garden or walks her dogs. The pain is predictable and subsides after a few minutes of rest. She has no other concerns. Her clinic vital signs include blood pressure 92/60 mm Hg and heart rate 59 beats/minute. Her laboratory results are all within normal limits, including SCr of 0.9 mg/dL. Which medication adjustments, if any, would be most appropriate to make at this visit for M.K.?

A	Increase metoprolol tartrate to 50 mg twice daily.
B	Increase amlodipine to 10 mg once daily.
C	Add ranolazine 500 mg twice daily.
D	No medication changes are warranted.
A

C - Relieves chest pain without lowering BP or HR.

How well did you know this?
1
Not at all
2
3
4
5
Perfectly
37
Q

In analyzing several clinical pharmacokinetic software programs for your institution, you find two with most of the characteristics meeting your criteria. The main difference between the two programs is that only one can perform Bayesian pharmacokinetics. Which is the biggest advantage of analyzing your clinical data using Bayesian pharmacokinetics?

A	When patient-specific data are limited, this method will place less emphasis on the population parameters.
B	With only a few serum concentrations, the calculated pharmacokinetic parameters will generally be more accurate.
C	The results of your analysis can be represented better graphically, including confidence intervals for each of the data points.
D	This method can effectively adjust for errors in documentation of the timing of drug concentration samples.
A

B - The biggest advantage of Bayesian pharmacokinetics is that, using population data, estimation of individual pharmacokinetic parameters is better with limited patient-specific data (Answer B is correct). This is accomplished by placing more emphasis on the population parameters when the patient-specific data are limited (Answer A is incorrect). Bayesian pharmacokinetics does not affect graphic output and cannot correct for inaccuracies in documentation of drug concentration sampling (Answers C and D are incorrect).

How well did you know this?
1
Not at all
2
3
4
5
Perfectly
38
Q

M.F. is a 36-year-old woman (height 66 inches, weight 56 kg) who presents to the ED with increased redness, swelling, and pain in her left leg. Duplex ultrasonography reveals a deep venous thrombosis (DVT). M.F. was hospitalized 2 weeks ago for pneumonia. She currently also has hypertension and smokes 1 pack/day of cigarettes. Her current medications include lisinopril 10 mg daily, hydrochlorothiazide 25 mg daily, and oral birth control pills. Her laboratory test results are all within normal limits. She plans to quit smoking and change her form of birth control.

Question 39
Which best depicts how long M.F. should receive anticoagulation therapy?

A	1 month.
B	3 months.
C	6 months.
D	Indefinitely.
A

B - All of the patient’s risk factors for venous thromboembolism (VTE) (recent hospitalization, smoking, oral birth control) are reversible. In patients with an identifiable and/or reversible cause of VTE, treatment recommendations are for 3 months of anticoagulant therapy, making Answer B correct. One month is only recommended for a superficial VTE, making Answer A incorrect. Answers C and D are incorrect because they would be considered in patients with idiopathic VTE.

How well did you know this?
1
Not at all
2
3
4
5
Perfectly
39
Q

M.F. is a 36-year-old woman (height 66 inches, weight 56 kg) who presents to the ED with increased redness, swelling, and pain in her left leg. Duplex ultrasonography reveals a deep venous thrombosis (DVT). M.F. was hospitalized 2 weeks ago for pneumonia. She currently also has hypertension and smokes 1 pack/day of cigarettes. Her current medications include lisinopril 10 mg daily, hydrochlorothiazide 25 mg daily, and oral birth control pills. Her laboratory test results are all within normal limits. She plans to quit smoking and change her form of birth control.

Question 40
Which is the most appropriate regimen for treating M.F.’s DVT?

A	Enoxaparin 60 mg subcutaneously twice daily for 5 days, followed by edoxaban 60 mg once daily.
B	Unfractionated heparin 3400-unit bolus, followed by 670 units per hour with warfarin 5 mg daily.
C	Rivaroxaban 15 mg twice daily for 21 days, followed by 20 mg daily.
D	Apixaban 5 mg twice daily for 7 days, followed by 2.5 mg daily.
A

C - Although edoxaban does require at least 5 days of initial injectable anticoagulant therapy and enoxaparin 60 mg once daily is the correct dose, the edoxaban dose should be reduced to 30 mg once daily instead of 60 mg because the patient weighs less than 60 kg, making Answer A incorrect. Answer B is incorrect because venous thromboembolism (VTE) treatment with a direct oral anticoagulant is recommended over a traditional injectable followed by warfarin. Answer B is also incorrect because the dosing of unfractionated heparin is the correct dosing for an acute coronary syndrome, but not for VTE. Although apixaban can be used as pure oral therapy for VTE, the dosing is 10 mg twice daily for 7 days, followed by 5 mg twice daily. Therefore, Answer D is incorrect because of the incorrect dosing regimen. Answer C is correct for using rivaroxaban in the treatment of VTE.

How well did you know this?
1
Not at all
2
3
4
5
Perfectly
40
Q

A 72-year-old man is admitted to the medical ICU for post-cardiac arrest care with therapeutic hypothermia. His body temperature was maintained at 91.4ºF (33ºC) for 24 hours, and the health care team decides it is time to rewarm slowly at 0.5ºC every hour. Which is most important to consider during the rewarming phase?

A	Frequent laboratory monitoring is necessary to guide potassium supplementation, given the risk of hypokalemia.
B	Vecuronium continuous infusion should be administered to prevent shivering.
C	Phenytoin should be administered for seizure prophylaxis.
D	Frequent blood glucose monitoring is necessary, given the risk of hypoglycemia.
A

D - Answer D, frequent blood glucose monitoring is necessary, given the risk of hypoglycemia, is correct. The cooling phase of therapeutic hypothermia may cause an intracellular shift of potassium, leading to a decreased serum potassium concentration on laboratory results. During the rewarming phase, an extracellular shift occurs, placing the patient at risk of hyperkalemia (Answer A is incorrect). Paralytics are not mandatory for shivering prevention during the rewarming phase and should be avoided, if possible (Answer B is incorrect). The rewarming phase of targeted temperature management alone does not place the patient at elevated risk of seizures, so prophylaxis with phenytoin is not required (Answer C is incorrect).

How well did you know this?
1
Not at all
2
3
4
5
Perfectly
41
Q

Differences in CYP inhibition may exist within the same class of therapeutic agents. Which class is best matched with the drug in that class that has the least CYP inhibition?

A	Fluoroquinolones – ciprofloxacin.
B	Macrolides – clarithromycin.
C	Calcium channel blockers – diltiazem.
D	Histamine-2 blockers – famotidine.
A

D - Ciprofloxacin has significant CYP inhibition. The correct drug would be levofloxacin or moxifloxacin (Answer A is incorrect). Clarithromycin also has significant CYP inhibition. The correct drug would be azithromycin (Answer B is incorrect). Diltiazem has significant CYP inhibition. The correct drug would be any of the dihydropyridine calcium channel blockers (Answer C is incorrect). Famotidine does not inhibit CYP, whereas cimetidine does (Answer D is correct).`

How well did you know this?
1
Not at all
2
3
4
5
Perfectly
42
Q

Which step would most assist in clearly defining a clinical question using the PICO model?

A	Compare databases to search.
B	Determine the preferred study design.
C	Evaluate the quality of evidence using the GRADE system.
D	Identify the intervention to be considered.
A

D - The PICO model is used to frame a clinical question in order to identify clinically relevant evidence in the literature. The “P” stands for patients or problems, in which the most important characteristics of the patient are described; “I” stands for the intervention to be considered (Answer D is correct); “C” stands for comparison or alternatives being considered compared with the intervention (Answer A is incorrect); and “O” represents the outcome of focus. A preferred study design may be identified according to the formulated clinical question using the PICO model, but the PICO model is not involved in defining the question itself, making Answer B incorrect. Although the GRADE system requires clearly defined clinical questions using PICO to evaluate quality of evidence, it is not included as part of the PICO model (Answer C is incorrect).

How well did you know this?
1
Not at all
2
3
4
5
Perfectly
43
Q

A health system is planning to expand its transitions of care program using the plan-do-study-act (PDSA) model. Which statement is most accurate when implementing the PDSA model for quality improvement?

A	Process outcomes and measures should be defined during the “do” phase. 
B	The PDSA cycle should be applied using a repetitive approach to allow for retesting and refinement.
C	The PDSA model should be implemented during large-scale quality improvement phases.
D	Unexpected observations should be documented during the “study” phase.
A

B - The PDSA model is a “test-and-learn-approach” to allow for refinement and continuous quality improvement through repetitive cycles, making Answer B correct. The PDSA cycle allows for controlled change experiments on a small scale before expanding to larger-scale projects (Answer C is incorrect). Process outcomes and measures should be defined during the “plan” phase, whereas active implementation in addition to documentation of the process, including barriers and unexpected observations, takes place during the “do” phase (Answers A and D are incorrect).

How well did you know this?
1
Not at all
2
3
4
5
Perfectly
44
Q

A 74-year-old woman with a history of hypertension and diabetes is admitted to the hospital with a 3-day history of nausea, vomiting, and diarrhea. Her baseline BUN and SCr are 15 mg/dL and 1.0 mg/dL, respectively. Her home medications include hydrochlorothiazide 12.5 mg by mouth daily, lisinopril 20 mg by mouth daily, and metformin 500 mg by mouth twice daily. She has had a diminished appetite and has been unable to drink fluids. Her blood pressure on admission is 102/64 mm Hg with a heart rate of 104 beats/minute (sitting) that decreases to 80/40 mm Hg on standing. Laboratory test results show serum sodium 136 mEq/L, serum potassium 4.2 mEq/L, BUN 48 mg/dL, SCr 2.2 mg/dL, and WBC 16.5 × 103 cells/mm3. A urinary catheter is placed, and a small amount of concentrated urine is obtained. Her urinary sodium is 12 mEq/L and urinary creatinine is 24.2 mg/dL. Which is the best assessment of this patient’s renal disease?

A	Stage 1 prerenal acute kidney injury (AKI).
B	Stage 2 prerenal AKI.
C	Stage 1 intrinsic AKI.
D	Stage 2 intrinsic AKI.
A

B - This patient has evidence of AKI, given the increase in BUN and SCr within 24 hours. Her AKI is prerenal, given her history of fluid loss from vomiting and diarrhea with poor oral intake of fluids. Her low blood pressure with orthostatic hypotension is consistent with inadequate vascular volume. The BUN/SCr ratio (greater than 20:1) is consistent with prerenal azotemia, whereas it would be expected to be normal (10–15:1) in a patient with intrinsic AKI. The fractional excretion of sodium (FENa) of less than 1% is consistent with prerenal AKI. Using the following equation to calculate FENa [(urinary sodium/serum sodium)/(urinary creatinine/SCr)] × 100 = [(12/136)/(24.2/2.2)] × 100 yields 0.8%. This reflects increased sodium reabsorption by the kidneys. Intrinsic AKI from either acute interstitial nephritis or acute tubular necrosis would be characterized by sodium wasting, with a high urinary sodium concentration and high FENa. All of these suggest that Answers C and D are incorrect. This AKI is classified as stage 2 because the SCr has increased by 2.0–2.9 times baseline (2.2 times) (Answer B is correct), whereas it would need to be 1.5–1.9 times baseline to be classified as stage 1 (Answer A is incorrect).

How well did you know this?
1
Not at all
2
3
4
5
Perfectly
45
Q

A 74-year-old man presents to the clinic for a follow-up of benign prostatic hyperplasia and hypertension. His urinary symptoms are well controlled, blood pressure is 150/84 mm Hg, and heart rate is 60 beats/minute. On digital rectal examination, his prostate is smooth but enlarged. He is currently treated for hypertension with lisinopril 20 mg and atenolol 25 mg daily and, for benign prostatic hyperplasia, with tamsulosin 0.4 mg once daily with dinner. Which is currently the most appropriate course of action?

A	Initiate finasteride 5 mg once daily.
B	Change tamsulosin to alfuzosin 10 mg once daily.
C	Increase atenolol to 50 mg daily.
D	Change tamsulosin to terazosin 5 mg daily.
A

D - In this patient with comorbid conditions of hypertension and benign prostatic hyperplasia, the choice of α-blocker is based on the adverse effect profiles. Changing tamsulosin to terazosin would improve the patient’s blood pressure control, but merely changing to an alternative selective α-blocker would not likely address the comorbid hypertension (Answer D is correct; Answer B is incorrect). Increasing the atenolol dose would not be appropriate, given the patient’s heart rate of 60 beats/minute (Answer C is incorrect). Initiating finasteride would not be indicated without first evaluating a prostate-specific antigen concentration (Answer A is incorrect).

How well did you know this?
1
Not at all
2
3
4
5
Perfectly
46
Q

A 54-year-old man with stage IIIa non–small cell lung cancer comes to the clinic today to start cycle 1 of chemotherapy with paclitaxel 45 mg/m2 and carboplatin dosed at AUC 2 given intravenously every week in combination with concurrent radiation. According to the most recent National Comprehensive Cancer Network (NCCN) guideline update, which is the most appropriate antiemetic regimen for nausea/vomiting prophylaxis for this patient to receive on treatment day 1?

A	Ondansetron plus dexamethasone.
B	Fosaprepitant plus palonosetron.
C	Fosaprepitant plus prochlorperazine plus dexamethasone.
D	Palonosetron plus olanzapine plus lorazepam.
A

A - With the recent NCCN guideline update classifying carboplatin-containing regimens as highly emetogenic if the carboplatin dose has an AUC of 4 or greater and moderately emetogenic when the carboplatin dose has an AUC less than 4, this patient’s regimen would fall into the moderately emetogenic category (paclitaxel is low risk for emetogenicity). This would require standard treatment with a serotonin-3 antagonist and steroid, or the olanzapine regimen of olanzapine, palonosetron, and dexamethasone (Answer A is correct). A neurokinin-1 antagonist can potentially be added on in patients receiving moderately emetogenic regimens if they previously had treatment failure with a serotonin-3 antagonist and steroid or if they have significant risk factors, which this patient does not have noted (Answer B is incorrect). Prochlorperazine is not recommended by the NCCN guidelines as part of the day 1 prophylactic regimen and is instead recommended as an option for breakthrough nausea/vomiting (Answer C is incorrect). Lorazepam is recommended for anticipatory nausea and vomiting, which this patient does not have (Answer D is incorrect).

How well did you know this?
1
Not at all
2
3
4
5
Perfectly
47
Q

A new drug is being studied for managing acute heart failure. The impact of this new drug on the primary end point of death from cardiovascular causes was assessed by survival analysis using a Kaplan-Meier analysis. Which best describes the benefits of using survival analysis?

A	All subjects required to enter the study at the same time.
B	Allows evaluation of the time course of treatment effect.
C	Does not allow data censoring because of loss of follow-up.
D	Only allows for assessment of mortality.
A

B - Survival analysis provides information relative to the temporal course of treatment effect (Answer B is correct), does not require subjects to enter the study at the same time (Answer A is incorrect), and does allow data censoring, one of its benefits (Answer C is incorrect). Answer D is incorrect because a survival analysis can be used for many different events such as time to disease progression or adverse events.

How well did you know this?
1
Not at all
2
3
4
5
Perfectly
48
Q

R.D. is an 87-year-old woman who was given a diagnosis of Alzheimer disease a year ago. After the diagnosis, she moved in with her daughter. Her daughter states that lately she wanders around the house continuously. She often changes clothes, cries out, and asks repetitive questions. She has a history of hypothyroidism, breast cancer, osteoarthritis in her right knee, and hypertension. Her current medication regimen includes donepezil 10 mg daily, which she has taken for the past 6 months; levothyroxine 50 mcg daily; hydrochlorothiazide 25 mg daily; and acetaminophen 650 mg three times daily. Which would be most appropriate for managing R.D.’s new behavioral symptoms?

A	Initiate olanzapine 5 mg daily.
B	Initiate risperidone 0.5 mg twice daily.
C	Change the donepezil dosage to 23 mg once daily.
D	Change acetaminophen to a topical or systemic NSA
A

D - No evidence currently supports increasing the donepezil dose to 23 mg to manage the behavioral symptoms of dementia (Answer C is incorrect). Off-label use of atypical antipsychotic medications in patients with behavioral symptoms of dementia should be reserved for patients who pose a danger to themselves or others or who have hallucinations or delusions that are stressful to them (Answers A and B are incorrect). Changing acetaminophen to an NSAID to treat possible pain that could be causing the patient’s behavior should be tried before trying more aggressive strategies (Answer D is correct).

How well did you know this?
1
Not at all
2
3
4
5
Perfectly
49
Q

Your health system has identified the opportunity to improve immunizations for pregnant people admitted for delivery. Standardizing the administration of which immunization upon hospital admission would have the greatest impact on achieving Healthy People 2030 objectives?

A	Hepatitis A.
B	Human papillomavirus (HPV).
C	Pneumococcal polysaccharide (PPSV23).
D	Tetanus, diphtheria, and pertussis (Tdap).
A

D - Healthy People 2030 has several objectives related to decreasing the prevalence of infectious disease through increased immunization rates, including pertussis in infants, HPV in adolescents, and acute hepatitis. When looking at the recommendations for use of certain vaccines during pregnancy, hepatitis A is only recommended during pregnancy if the patient is at risk of an infection or a severe outcome from infection while pregnant (Answer A is incorrect). The HPV is not recommended for use during pregnancy, and if the vaccine is to be administered upon hospital admission, the recipient would still be pregnant (Answer B is incorrect). A dose of PPSV23 is only recommended for those with certain conditions before age 65; therefore, routine administration is not recommended (Answer C is incorrect). The use of Tdap is recommended during each pregnancy to reduce the risk of pertussis in the infant; therefore, standardization of this vaccine upon hospital admission would likely have the largest impact on reaching the Healthy People 2030 goals (Answer D is correct).

How well did you know this?
1
Not at all
2
3
4
5
Perfectly
50
Q

Which benefit of automated dispensing cabinet technologies utilized by hospital systems best helps optimize pharmacy technician workload?

A	Enhanced security of medications.
B	Increased measures to avert medication diversion.
C	Reduction in medication errors.
D	Enhanced inventory control.
A

D - Each option is a benefit of automatic dispensing cabinet technologies. Enhanced security of medications will help avert medication diversion, but these will not best optimize technician workload (Answers A and B are incorrect). Reduction in medication errors is a major benefit of automated dispensing cabinets, but this is primarily helpful to improve patient safety (Answer C is incorrect). Enhanced inventory control such as expansion of high-use medications and reduction of infrequently used medications decrease the volume and frequency of stocking and destocking medications, ultimately increasing efficiency of technician work (Answer D is correct).

How well did you know this?
1
Not at all
2
3
4
5
Perfectly
51
Q

Results from a randomized controlled trial showed a difference in 90-day mortality between the intervention group and the control group as 11% (p=0.03). Which statement is most accurate?

A	The trial did not have adequate power.
B	The chance of a type II error was 3 in 100.
C	The chance of a type I error was 3 in 100.
D	There was no statistically significant difference between groups.
A

C - The typical a priori α (type I) rate is 5% (i.e., when the study was designed, the error rate was designed to be 5% or less). The actual type I error rate reported was 0.03 (3%), the p value (Answer C is correct). This difference was considered statistically significant because the p value was less than 0.05 (Answer D is incorrect). The study did have enough power because a statistically significant difference was observed (Answer A is incorrect). Similarly, a type II error was not made because this error has to do with not finding a difference when one truly exists (Answer B is incorrect).

How well did you know this?
1
Not at all
2
3
4
5
Perfectly
52
Q

A new oral anticoagulant was studied to evaluate its ability to prevent venous thromboembolism (VTE) in medically ill patients. In this randomized, double-blind, double-dummy trial, patients were randomized to receive enoxaparin 40 mg once daily during the hospitalization, with placebo also during the hospitalization and for 30 days after discharge, or the new oral anticoagulant during the hospitalization and for 30 days after discharge with placebo injections during the hospital stay. The primary efficacy end point was the rate of VTE at 30 days after discharge, and the primary safety end point was major bleeding. At the end of the trial, VTE had occurred less often with the use of extended prophylaxis with the new oral anticoagulant than with enoxaparin during hospitalization only (3.9% vs. 5.7%; RR 0.68 [95% CI, 0.53–0.88]). Major bleeding with extended prophylaxis was 0.7% compared with 0.5% with enoxaparin (RR 1.48; 95% CI, 0.77–2.84). Which is the most accurate interpretation of these data?

A	Extended prophylaxis significantly reduced VTE and significantly increased major bleeding compared with enoxaparin.
B	Extended prophylaxis significantly reduced VTE without significantly increasing major bleeding compared with enoxaparin.
C	Extended prophylaxis increased major bleeding more than it reduced VTE compared with enoxaparin.
D	The reduction in VTE with extended prophylaxis was balanced against the increased risk of bleeding compared with enoxaparin.
A

B - The RR for the reduction in VTE with extended prophylaxis with the new oral anticoagulant compared with enoxaparin was less than 1, and the CI did not include 1, making the reduction in VTE statistically significant. Although the RR for major bleeding was greater than 1 for extended prophylaxis compared with enoxaparin, the CI included 1, making the difference not statistically significant. Therefore, Answer B is correct and Answer A is incorrect. The absolute reduction in VTE with extended prophylaxis compared with enoxaparin was 1.8, and the absolute increase in major bleeding was 0.2. Therefore, the increase in major bleeding was not balanced or more than the reduction in VTE with extended prophylaxis compared with enoxaparin, making Answers C and D incorrect.

How well did you know this?
1
Not at all
2
3
4
5
Perfectly
53
Q

C.T. is a 40-year-old woman who presents to the hospital with right leg pain and swelling. The erythema and swelling extend from her ankle to 2 inches below her knee. There is no purulent drainage and no leading edge. She also has chills. Her medical history includes asthma and obesity (diabetes in her father). She has a history of rash associated with penicillin use. Which is the best treatment for C.T.?

A	Piperacillin/tazobactam 4.5 g intravenously every 8 hours plus vancomycin 15 mg/kg intravenously every 12 hours.
B	Ceftaroline 600 mg intravenously every 12 hours.
C	Doxycycline 100 mg intravenously every 12 hours.
D	Cefazolin 2 g intravenously every 8 hours.
A

D - This infection does not appear to be necrotizing fasciitis. Therefore, piperacillin/tazobactam plus vancomycin would provide too-broad antibacterial activity. There is no need for the gram-negative and anaerobic activity that piperacillin/tazobactam adds over vancomycin alone. Moreover, the patient is allergic to penicillins (Answer A is incorrect). This appears to be a cellulitis, but there is no indication from the patient’s symptoms and history that methicillin-resistant Staphylococcus aureus (MRSA) is involved. Therefore, ceftaroline is not necessary because its advantage over cefazolin is primarily in the addition of MRSA activity (Answer B is incorrect). Doxycycline has activity against Staphylococcus, including MRSA, but its streptococcal activity is too poor to use alone empirically in this situation, and there is no indication that MRSA is causing the infection (Answer C is incorrect). Cefazolin is the best choice in this situation because of its excellent activity against Staphylococcus (non-MRSA) and Streptococcus, the two most-suspected organisms causing cellulitis. Because the patient only describes her penicillin allergy as a rash, it is safe to try a cephalosporin at this time (Answer D is correct).

How well did you know this?
1
Not at all
2
3
4
5
Perfectly
54
Q

Which is the most appropriate candidate to receive the tetanus, diphtheria, pertussis (Tdap) vaccine?

A	64-year-old man who received his last Tdap vaccine at 53 years of age.
B	32-year-old pregnant woman at 12 weeks' gestation.
C	14-year-old female adolescent requiring wound management, who received a Tdap vaccine at 12 years of age.
D	18-year-old man entering college, who received his first Tdap vaccine at 11 years of age.
A

A - In 2019, the ACIP updated its recommendations to allow Tdap administration when previously only tetanus, diphtheria (Td) was recommended, including decennial Td booster doses, tetanus prophylaxis for wound management, and catch-up immunizations for those 7 years and older with an incomplete or unknown vaccination history. Answer A is correct because this patient is currently overdue for his decennial booster dose. Although pregnant women should receive one dose of Tdap during each pregnancy, it should ideally be administered at 27–36 weeks’ gestation, making Answer B incorrect. Answer C is incorrect because Tdap or Td is indicated for wound management when more than 5 years have passed since the previous tetanus toxoid–containing vaccine dose. Answer D is incorrect because this patient is currently up to date with a routine adolescent Tdap dose and would only require a booster every 10 years.

How well did you know this?
1
Not at all
2
3
4
5
Perfectly
55
Q

A.J. is a 34-year-old man who has been HIV positive for the past 3 years. His current viral load is 200,000 copies/mL, and his CD4 count is 360 cells/mm3. Which is the best initial action to take for A.J.?

A	Continue to monitor viral load and CD4 count, but no therapy is needed at this time.
B	Initiate tenofovir disoproxil fumarate 300 mg orally daily and emtricitabine 200 mg orally daily because his CD4 count is still above 200 cells/mm3.
C	Initiate dolutegravir 50 mg orally daily, emtricitabine 200 mg orally daily, and tenofovir disoproxil fumarate 300 mg orally daily.
D	Initiate darunavir/cobicistat 800/150 mg orally daily, abacavir 600 mg orally daily, and lamivudine 300 mg orally daily.
A

C - Current guidelines recommend that all patients with HIV infection be treated with highly active antiretroviral therapy, regardless of viral load or CD4 counts. Therefore, therapy should be initiated (Answer A is incorrect). Therapy should include a “recommended regimen for most patients.” Tenofovir disoproxil fumarate and emtricitabine without a third agent is not a recommended regimen (Answer B is incorrect). Darunavir/cobicistat with abacavir and lamivudine is recommended in certain clinical situations and is not considered a first-line regimen (Answer D is incorrect). Dolutegravir with emtricitabine/tenofovir disoproxil fumarate is correct. All of the current “recommended regimens” have an integrase inhibitor combined with two reverse transcriptase inhibitors (Answer C is correct).

How well did you know this?
1
Not at all
2
3
4
5
Perfectly
56
Q

Which of the following best depicts the organism that can be identified in a pharmacy through a Clinical Laboratory Improvement Amendments of 1988 (CLIA)–waived point-of-care test?

A	Helicobacter pylori.
B	Staphylococcus aureus.
C	Hepatitis B virus.
D	Pseudomonas aeruginosa.
A

A - H. pylori is the only one of the listed organisms that can be identified through a CLIA-waived point-of-care test, making Answer A correct. Although hepatitis C virus can be identified through detected antibodies by a point-of-care test, this is not true for hepatitis B virus. Therefore, Answer C is incorrect. There are no CLIA-waived tests for S. aureus or P. aeruginosa, making Answers B and D incorrect.

Adenovirus, GAS, H.pulori, HCV, HIV, mononucleosis, Flu A and B, Borerrelia burgdoferi, RSV, Treponema pallidum, Trichomonas vaginalis

How well did you know this?
1
Not at all
2
3
4
5
Perfectly
57
Q

A 62-year-old man with a history of hypertension, type 2 diabetes, and dyslipidemia takes metformin 500 mg twice daily and atorvastatin 20 mg daily. His blood pressure at home and in the clinic averages 144/86 mm Hg. His SCr, urine albumin/creatinine, LDL, and A1C are 0.7 mg/dL, 89 mg/g, 68 mg/dL, and 6.9%, respectively. All laboratory measures are consistent with the values obtained 3 months ago. All additional laboratory values are within normal limits. Which recommendation is most appropriate for this patient at today’s visit?

A	No change in therapy is needed.
B	Initiate enalapril.
C	Increase metformin.
D	Increase atorvastatin.
A

B - Initiating angiotensin-converting enzyme inhibitor therapy (Answer B) is the most appropriate answer because the patient’s blood pressure and urine albumin/creatinine remain elevated. Answer A is incorrect; doing nothing would place the patient at increased risk of damage from uncontrolled blood pressure. Answer C is incorrect because the patient’s A1C is adequately controlled. Increasing atorvastatin, Answer D, is also incorrect because the patient does not have a history of atherosclerotic cardiovascular disease and is appropriately receiving a moderate-intensity statin. In addition, insufficient information is provided to suggest the patient’s current atorvastatin dose is incorrect.

How well did you know this?
1
Not at all
2
3
4
5
Perfectly
58
Q

A health system has been notified of a drug shortage. Which is the first step when developing a drug shortage management plan?

A	Assess the clinical impact of the drug shortage.
B	Determine the amount of product on hand.
C	Identify any medication safety risks associated with the shortage.
D	Plan for necessary changes in workflow.
A

B - There is a stepwise approach in developing a drug shortage management plan. All of the answer choices are appropriate steps; however, the first step is determining the amount of product on hand to approximate how long the product will last, in addition to when the next delivery will arrive, if known (Answer B is correct). Then, the clinical impact of the drug shortage is determined, which includes how essential the medication is with respect to saving lives or curing a disease, whether any alternatives are available, and which patients and clinicians will be affected by the shortage (Answer A). Next, safety risks associated with any alternative drugs or changes in formulation or strength are identified (Answer C). Overall workflow changes including storage and technology are addressed (Answer D). Finally, strategies on how the remaining product will be used are established

How well did you know this?
1
Not at all
2
3
4
5
Perfectly
59
Q

A 62-year-old woman with a history of diabetes and recent trauma is admitted to the hospital because of cellulitis/osteomyelitis caused by Staphylococcus aureus. Empirically, she is initiated on vancomycin at 1000 mg intravenously daily. You are consulted to monitor therapy. Which monitoring strategy is best?

A	Obtain a peak concentration only to ensure concentrations reach at least 40 mg/L for adequate tissue perfusion and optimal efficacy.
B	Obtain a trough concentration only to ensure concentrations decrease below 15 mg/L during the dosing interval to avoid nephrotoxicity.
C	Obtain a trough concentration and potentially one other concentration to target an AUC/MIC ratio of 400–600.
D	Obtain peak and trough concentrations to optimize efficacy while minimizing toxicity.
A

C - Because vancomycin follows time-dependent killing, obtaining and assessing peak concentrations is not beneficial from an efficacy standpoint. Moreover, nephrotoxicity is most associated with higher trough concentrations, eliminating the need to monitor peak concentrations (Answers A and D are incorrect). Traditionally, trough concentrations have been monitored to maintain concentrations above the MIC of the organism while minimizing the incidence of nephrotoxicity. However, more recent data analyses support that achieving an AUC/MIC ratio of 400–600 both maximizes efficacy and minimizes toxicity (Answer C is correct). Calculations of AUC can be completed with one concentration and a Bayesian software program or two concentrations during the same dosing interval (Answer B is incorrect).

How well did you know this?
1
Not at all
2
3
4
5
Perfectly
60
Q

Which reference is best to find the most up-to-date information on a new drug for the treatment of heart failure with preserved ejection fraction?

A	Journal of the American College of Cardiology.
B	American Hospital Formulary Service.
C	UpToDate.
D	Braunwald’s Heart Disease: A Textbook of Cardiovascular Medici
A

A - Answers B, C, and D all represent tertiary references, which are not the best references for investigating new information because they may have a several-year lag in the data provided. Despite titles like “UpToDate” or specific cardiology texts, Answers B, C, and D are incorrect. The Journal of the American College of Cardiology is the official journal of the American College of Cardiology, which is one of the leading cardiology organizations in the world. Answer A represents a primary literature source, which would provide the latest data in a well-respected cardiology journal. Therefore, Answer A is correct.

Primary - original research, immediate results, original research published in peer reviewed journals, dissertations, technical reports, conference proceedings

Secondary - Summarizes and synthesizes primary literature, literature review articles, books

Tertiary - summaries or condensed versions, references to primary or secondary sources, good place to look up facts or general overview. Textbooks, dictionaries, encyclopedias, handbooks

How well did you know this?
1
Not at all
2
3
4
5
Perfectly
61
Q

You are reviewing a manuscript that reports the results from a study of the effect of pharmacist involvement on a general medical team. The authors find that teams that include pharmacists make fewer errors, and the estimates are presented as the mean number of errors plus or minus the standard error of the mean (SEM). In your review, you comment on the authors’ choice of the SEM as their measure of variability. Which best describes the authors’ choice?

A	Appropriate because it estimates the spread of the data.
B	Inappropriate because it estimates the variability of the individual observations, not of the means estimated from repeated iterations of the study.
C	Appropriate because it overestimates the variability of the data.
D	Inappropriate because it estimates the variability of the means estimated from repeated iterations of the study, not of the individual observations.
A

D - Answer D is correct because SEM does not measure the spread (Answer A is incorrect) or the variability (Answers B and C are incorrect) of data. The SEM is estimated by dividing the standard deviation by the square root of the sample size. The SEM alone does not measure the variability of data and often underestimates the variability of a data set (again making Answer C incorrect). That leaves Answer D, which correctly describes the purpose of the SEM.

How well did you know this?
1
Not at all
2
3
4
5
Perfectly
62
Q

M.F. is a 45-year-old man with recently diagnosed Parkinson disease. Currently, his symptoms consist of a tremor in his right hand. Other than taking hydrochlorothiazide for hypertension, he is in good health. At this time, which would be the best treatment for M.F.?

A	Carbidopa/levodopa.
B	Ropinirole.
C	Entacapone.
D	Trihexyphenidyl.
A

D - In Parkinson disease, the more effective drugs should be reserved for later in treatment as the disease progresses. Ropinirole (Answer B) and carbidopa/levodopa (Answer A) are highly effective for Parkinson disease and treat many of the symptoms in addition to tremor. Most treatment schemes recommend waiting until the Parkinson symptoms progress. Trihexyphenidyl (Answer D) is effective when the patient is only symptomatic for a tremor. Entacapone (Answer C) helps extend the effectiveness of carbidopa/levodopa through a dosing interval but is not highly effective on its own.

How well did you know this?
1
Not at all
2
3
4
5
Perfectly
63
Q

The relationship between a newly marketed drug and standard of care for impact on asthma exacerbations leading to hospitalizations was studied in a prospective cohort study. The results of the study showed a risk ratio (RR) of 2.0 with a 95% CI of 1.5–2.5.

Question 65
Which one of the following statements is most consistent with these results?

A	The new drug is associated with no change in exacerbations.
B	The new drug is associated with a decrease in exacerbations.
C	The new drug is associated with an increase in exacerbations.
D	More information is needed to decide whether the new drug is associated with any changes in exacerbations.
A

C - The key to this question is the 95% CI. When interpreting an RR, an equal risk would be reported as an RR of 1. For an RR greater than 1, as in this case, the risk of developing an exacerbation would be higher in the patients receiving the new drug than in the patients not receiving this drug. Furthermore, because the CI does not cross or include 1.0, the point estimate in this study (2.0) is statistically significant (Answer C). Both Answer A (CI would need to include 1) and Answer B (RR would need to be less than 1 and CI could not include 1 for this to be correct) are incorrect. Answer D is incorrect because the information given is adequate to answer this question.

How well did you know this?
1
Not at all
2
3
4
5
Perfectly
64
Q

The relationship between a newly marketed drug and standard of care for impact on asthma exacerbations leading to hospitalizations was studied in a prospective cohort study. The results of the study showed a risk ratio (RR) of 2.0 with a 95% CI of 1.5–2.5.

Question 66
Given the information provided, which statement best represents the statistical significance of the reported RR?

A	Because the p value was not presented, it cannot be determined whether there is a statistically significant difference between the groups.
B	The RR is statistically significant at p<0.05.
C	The RR is statistically significant at p<0.01.
D	The RR is not statistically significant.
A

B - The CI does not cross or include 1.0, indicating that the point estimate in this study (2.0) is statistically significant (Answer B is correct). Answer A is incorrect because enough information is present to answer this question, and a p value is not required. According to the question, the RR and CI information allows the appropriate answer to this question. The p value of less than 0.05 corresponds to an α value less than 0.05 and is thus statistically significant. This p value also corresponds with the 95% CI. For Answer C to be correct, the 99% CI would need to be determined and the same evaluation of the CI made. Furthermore, because the CI does not cross or include 1.0, the point estimate in this study (2.0) is statistically significant (Answer D is incorrect).

How well did you know this?
1
Not at all
2
3
4
5
Perfectly
65
Q

T.T. is a nurse who works at your institution. While obtaining blood from a patient known to be HIV positive, she accidentally stuck herself. The needle went through her glove and broke the skin on her hand. The patient’s most recent CD4 count was 220 cells/mm3, and her most recent viral load was 50,000 copies/mL. Which therapy is best for T.T.?

A	No drug therapy is needed – just disinfect the area well.
B	Offer emtricitabine 200 mg orally daily for 2 weeks.
C	Offer emtricitabine 200 mg and tenofovir alafenamide 25 mg orally daily for 3 months.
D	Offer emtricitabine 200 mg orally daily, tenofovir disoproxil fumarate 300 mg orally daily, and raltegravir 400 mg orally twice daily for 4 weeks.
A

D - Because this was a significant needlestick from a known HIV-positive patient, T.T. should be offered drug therapy (Answer A is incorrect). Although it is generally recommended that postexposure prophylaxis last 4 weeks (not 2 weeks), emtricitabine monotherapy is inappropriate for occupational postexposure prophylaxis (Answer B is incorrect). Emtricitabine plus tenofovir alafenamide is indicated for preexposure prophylaxis, but a regimen with only two agents is not recommended for postexposure prophylaxis. Moreover, 3 months is too long a duration (Answer C is incorrect). The most appropriate prophylactic regimen is raltegravir with tenofovir disoproxil fumarate/emtricitabine for 4 weeks (Answer D is correct).

How well did you know this?
1
Not at all
2
3
4
5
Perfectly
66
Q

R.N. is a 68-year-old man with a history of hypertension and atrial fibrillation who is receiving digoxin. Verapamil is initiated for further ventricular rate control. Which best predicts what will happen to his digoxin concentration if verapamil is initiated?

A	Will increase significantly because more digoxin will be absorbed.
B	Will decrease significantly because more digoxin will be removed during the first pass through the liver.
C	Will decrease significantly because more digoxin will be secreted in the kidneys.
D	Will increase significantly because less digoxin will distribute into peripheral tissues.
A

A - Digoxin concentrations increase significantly when verapamil is added . This is because verapamil blocks P-glycoprotein, which normally decreases the amount of digoxin that is absorbed in the GI tract (by pumping it back into the GI lumen) (Answer A is correct). Verapamil also blocks P-glycoprotein in the kidneys, which decreases renal secretion of digoxin, also increasing its concentration (Answer C is incorrect). These effects are not the result of any effect within the liver (Answer B is incorrect). Verapamil does not affect the distribution of digoxin into peripheral tissues (Answer D is incorrect).

How well did you know this?
1
Not at all
2
3
4
5
Perfectly
67
Q

A recent randomized controlled trial of a new drug was designed with 85% power to detect a 20% difference in the primary end point of blood pressure reduction. The trial targeted 500 subjects for enrollment on the basis of the sample size calculation. The secondary end point, occurrence of adverse drug events, was also studied. The power associated with this secondary end point in detecting a 6% difference in the occurrence of adverse drug events was 80%. Because of the increased costs associated with the trial, the investigators could only enroll 400 subjects. In the discussion section of the paper describing the study findings, the authors commented that because of lower sample size, the power associated with detecting a 20% difference in blood pressure was still an acceptable 80%. Which is the most critical problem caused by the reduced sample size?

A	Randomization procedure was suboptimal.
B	Study was underpowered to detect a difference in the primary end point (blood pressure reduction).
C	Study was underpowered to detect rare adverse events.
D	Study was underpowered to detect a difference in the secondary end point (occurrence of adverse drug events).
A

D - In the discussion section, the authors correctly pointed out that even though they could not recruit the appropriate number of subjects, 80% power was obtained, which is considered an acceptable level of power in the biomedical literature for the primary end point. Answer D is correct because the authors could not maintain adequate power for comparison of the secondary end point. Because the original power for that comparison was 80%, it would have fallen below the 80% level typically considered optimal. No information is presented to conclude that the randomization was suboptimal (Answer A is incorrect). Answer B is incorrect because the power is adequate. Answer C is incorrect because this size of trial could not identify rare adverse events, regardless of subject enrollment.

How well did you know this?
1
Not at all
2
3
4
5
Perfectly
68
Q

The results of a randomized controlled trial between two drug treatments for depression show no difference in clinical response between the two treatment groups (p=0.25). Clinical response was the primary end point of the study. The study authors report that they subsequently analyzed subgroups on the basis of the following: subjects with past episodes of depressive episodes, subjects with a family history of depression, and subjects who were women. The authors found a significant difference in response between the two treatments (p<0.01) for men versus women. Which statement is most accurate regarding the results of the subgroup analysis?

A	The finding of a significant difference between men and women in this study should be tested in a prospective study.
B	Post hoc subgroup analyses are generally useful for showing differences between patient subgroups.
C	Statistical adjustments are not needed when comparing the results of each subgroup.
D	Men and women are expected to respond differently to depression pharmacotherapy.
A

A - Subgroup analyses are vulnerable to many problems in interpretation (Answer B is incorrect). In general, subgroup analyses not specified at the beginning of the study are likely to have issues of power and the absence of correction to the statistical analysis to account for multiple subgroup comparisons (Answer C is incorrect). Findings from post hoc analyses are best viewed as hypothesis generating (Answer A is correct), similar to other observational studies, not as answers to the questions themselves (Answer D is incorrect). There are some exceptions to this discussion.

How well did you know this?
1
Not at all
2
3
4
5
Perfectly
69
Q

A.J. is a 45-year-old woman with multiple sclerosis (MS). She has been treated with glatiramer and ß-interferon, which have slowed the disease progression. In the clinic today, she notes increased difficulty walking. Her walking speed has slowed greatly. She asks whether anything can help with her walking. Which medication would best help with walking?

A	Amantadine.
B	Fingolimod.
C	Natalizumab.
D	Dalfampridine.
A

D - Fingolimod (Answer B) and natalizumab (Answer C) are disease-modifying treatments for MS; however, although these agents slow disease progression in MS, they do not help with acute treatment of symptoms. Amantadine (Answer A) relieves fatigue symptoms associated with MS, but not difficulties with ambulation. Dalfampridine (Answer D) is specifically indicated for difficulty walking with MS.

How well did you know this?
1
Not at all
2
3
4
5
Perfectly
70
Q

A 65-year-old man undergoes coronary angioplasty and has two drug-eluting stents placed. The current plan is to initiate aspirin and clopidogrel. Pharmacogenetic testing shows that he is a poor metabolizer of CYP2C19. According to the Clinical Pharmacogenetics Implementation Consortium (CPIC), which is the best recommendation concerning his antiplatelet therapy?

A	The clopidogrel dose should be decreased to avoid potential toxicity.
B	The clopidogrel dose should be increased to prevent stent thrombosis.
C	Clopidogrel should be dosed the same, but aspirin should only be used at a dose of 81 mg.
D	Another antiplatelet should be used in place of clopidogrel.
A

D - Clopidogrel is a prodrug that is metabolized by CYP2C19 into its active metabolite. Poor metabolizers of clopidogrel do not produce as much active metabolite, rendering clopidogrel less effective. Lowering the dose would only further decrease its effectiveness (Answer A is incorrect). Raising the dose to overcome this effect has not been completely evaluated and is currently not recommended (because the exact required dose increase is not known) (Answer B is incorrect). The CPIC currently recommends that poor metabolizers of CYP2C19 receive an alternative antiplatelet (Answer D is correct; Answer C is incorrect).

71
Q

The pharmacy and therapeutics committee is scrutinizing a formulary drug because of case reports of a rare, but serious drug-induced toxicity not observed before marketing. The FDA is reevaluating the drug’s safety, but this reevaluation will likely take time. Which study design would be most suitable to address this problem?

A	Randomized controlled crossover clinical trial.
B	Case-control study.
C	Prospective cohort study.
D	Meta-analysis.
A

B - A case-control study would be the most appropriate study design in this situation because this type of study would investigate a relatively common exposure (drug is on the market and is routinely being used clinically) and a relatively rare/uncommon outcome (Answer B is correct). Answer A would not be ethically possible because we know there is a published rare, but serious toxicity. In addition, Answers A and C are incorrect because these designs would take considerable time to conduct. Answer D is incorrect because this design would require previously published studies to conduct.

72
Q

Which characteristic most increases the readability of patient education materials?

A	Use of symbols instead of words.
B	Written at a fifth-grading reading level.
C	Long lists of statistics.
D	Expressing risk as a percentage.
A

B - Symbols are not universally recognized and understood across populations; therefore, use of symbols and images is not recommended to replace words (Answer A is incorrect). The average adult in the United States is able to read at an eighth-grade reading level; therefore, to be most readable, it is recommended to keep the reading level of patient materials at a fifth-grade level (Answer B is correct). Use of long lists and many statistics can be confusing and difficult to interpret and is not a recommended best practice for patient education materials (Answer C is incorrect). When expressing risks or benefits, these should be presented as absolutes (whole numbers) rather than percentages – so 1 in 100 will experience this, rather than 1% (Answer D is incorrect).

73
Q

According to the Grading of Recommendations Assessment, Development and Evaluation (GRADE) system, which scenario best fits a strong recommendation, high quality of evidence designation?

A	Two randomized trials with large effect.
B	Six observational studies with minimal bias.
C	Four randomized trials with publication bias.
D	Four observational studies and two randomized trials with inconsistency and imprecision.
A

A - According to the GRADE system, a recommendation is designated as high quality of evidence when randomized trials have a large magnitude of effect with no plausible confounders (Answer A is correct). Observational studies are initially designated as low quality of evidence; however, they may be increased to moderate with minimal bias (Answer B is incorrect). Randomized trials are initially designated as high-quality evidence; however, they may be lowered to moderate quality of evidence with publication bias, inconsistency, and/or imprecision (Answers C and D are incorrect).

74
Q

An 82-year-old community-dwelling woman with a history of Parkinson disease, hypertension, and urinary incontinence presents to her primary care clinic accompanied by her daughter. She walks slowly with a cane and has stooped posture. Her medication profile shows carbidopa/levodopa, pramipexole, selegiline, tolterodine, diazepam, metoprolol, and hydrochlorothiazide. Her daughter reports that she has been more lethargic and unsteady walking over the past 3 days. Her blood pressure is 135/72 mm Hg and heart rate is 76 beats/minute. Urinalysis is negative, and thyroid-stimulating hormone is within the reference range.

Question 76
Which best represents the single most important intervention in reducing this patient’s current risk of falling?

A	Reassess her medication regimen for Parkinson disease.
B	Consider discontinuing diazepam.
C	Suggest she see a physical therapist to institute a therapeutic exercise program.
D	Evaluate orthostatic blood pressure measurements.
A

B - For this patient, all of the choices could reasonably reduce her risk of falling. However, diazepam is a long-acting benzodiazepine that can accumulate in older patients, resulting in excessive lethargy, sedation, and unsteady gait, which may currently be most likely contributing to her fall risk (Answer B is correct). Patients with Parkinson disease may be at increased risk of falling, physical therapy may improve gait and balance, and monitoring blood pressure to avoid hypotension with the use of antihypertensives are all reasonable, but reevaluating the use of diazepam is currently the single most important intervention for reducing her risk of falls (Answers A, C, and D are incorrect).

75
Q

An 82-year-old community-dwelling woman with a history of Parkinson disease, hypertension, and urinary incontinence presents to her primary care clinic accompanied by her daughter. She walks slowly with a cane and has stooped posture. Her medication profile shows carbidopa/levodopa, pramipexole, selegiline, tolterodine, diazepam, metoprolol, and hydrochlorothiazide. Her daughter reports that she has been more lethargic and unsteady walking over the past 3 days. Her blood pressure is 135/72 mm Hg and heart rate is 76 beats/minute. Urinalysis is negative, and thyroid-stimulating hormone is within the reference range.

Question 77
Which age-related change in pharmacokinetics is most likely increasing the negative impact of diazepam on her energy level and gait?

A	Delayed gastric emptying.
B	Decreased renal excretion.
C	Slowed metabolism in the liver.
D	Decreased volume of distribution.
A

C - Diazepam tends to accumulate with reduced capacity for elimination, resulting in excessive sedation and an increased risk of falls in older patients. In this case, changes in hepatic metabolism through phase I (oxidation) are diminished; thus, this patient may be having altered diazepam metabolism resulting in increased lethargy (Answer C is correct). In older adults, the volume of distribution of lipid-soluble drugs such as diazepam is increased, not decreased (Answer D is incorrect). Delayed gastric emptying is unlikely to be problematic, given no indication of this problem in the patient’s medical history (Answer A is incorrect). Decreased renal excretion is likely but is probably not a significant contributor in this patient, given that diazepam is eliminated hepatically, with only inactive conjugated metabolites eliminated by the kidney (Answer B is incorrect).

76
Q

A 62-year-old man with a history of heart failure and hyperthyroidism is admitted to the ICU with severe community-acquired pneumonia. Six hours after admission, he develops acute respiratory failure and hypotension, necessitating intubation. He is sedated on fentanyl 150 mcg/hour and propofol 15 mcg/kg/minute. His Richmond Agitation-Sedation Scale (RASS) score is +2, Critical-Care Pain Observation Tool (CPOT) score is 2, and confusion assessment method for the ICU is negative. Laboratory values include WBC 19 × 103 cells/mm3, Hgb 12.3 g/dL, Hct 36%, Plt 210,000/mm3, BUN 12 mg/dL, SCr 1.1 mg/dL, glucose 102 mg/dL, Na 144 mEq/L, Cl 109 mEq/L, K 4.1 mEq/L, and HCO3 24 mEq/L.

Question 78
Which therapy is most appropriate for preventing stress ulcers in this patient?

A	Administer sucralfate 1 g four times daily by nasogastric tube.
B	Administer magnesium hydroxide 30 mL four times daily by nasogastric tube.
C	Administer famotidine 20 mg intravenously twice daily.
D	Stress ulcer prophylaxis is not indicated.
A

C - Answer C, administer famotidine 20 mg intravenously twice daily, is correct. The patient has one of the major risk factors for respiratory failure necessitating mechanical ventilation, warranting prophylactic therapy for stress ulcers (Answer D is incorrect). Antacids and sucralfate are not recommended for prevention, and sucralfate is inferior to histamine-2 blockers (Answers A and B are incorrect).

77
Q

A 62-year-old man with a history of heart failure and hyperthyroidism is admitted to the ICU with severe community-acquired pneumonia. Six hours after admission, he develops acute respiratory failure and hypotension, necessitating intubation. He is sedated on fentanyl 150 mcg/hour and propofol 15 mcg/kg/minute. His Richmond Agitation-Sedation Scale (RASS) score is +2, Critical-Care Pain Observation Tool (CPOT) score is 2, and confusion assessment method for the ICU is negative. Laboratory values include WBC 19 × 103 cells/mm3, Hgb 12.3 g/dL, Hct 36%, Plt 210,000/mm3, BUN 12 mg/dL, SCr 1.1 mg/dL, glucose 102 mg/dL, Na 144 mEq/L, Cl 109 mEq/L, K 4.1 mEq/L, and HCO3 24 mEq/L.

Question 79
Which statement regarding this patient’s sedation is most appropriate?

A	Propofol should be increased to achieve a goal RASS of 0 to -1.
B	Fentanyl should be increased to achieve a goal CPOT of 0.
C	Initiate haloperidol to treat delirium and achieve a goal RASS of 0 to -1.
D	Initiate adjunctive ketamine to reduce fentanyl and propofol requirements.
A

A - Answer A, propofol should be increased to achieve a goal RASS of 0 to -1, is correct. For this patient, agitation is the main issue requiring intervention. His delirium test was negative, so haloperidol is not warranted (Answer C is incorrect). The role of ketamine in ICU patients has not yet been fully elucidated. Thus, even though ketamine may play a role, in this case, the patient’s sedative requirements are not causing any adverse effects, so adding a third agent at this time is unnecessary (Answer D is incorrect). His CPOT is 2, which is at goal of less than 3, indicating his pain is controlled with the current fentanyl infusion dose (Answer B is incorrect).

78
Q

Which is the most appropriate therapy to reduce the risk of delayed cerebral ischemia and improve neurologic outcomes after aneurysmal subarachnoid hemorrhage?

A	Clevidipine continuous infusion titrated to maintain systolic blood pressure less than 160 mm Hg.
B	Nimodipine 60 mg every 4 hours for 21 days.
C	Norepinephrine continuous infusion titrated to maintain systolic blood pressure greater than 160 mm Hg.
D	Amlodipine 10 mg every 24 hours for 21 days.
A

B - Answer B, nimodipine 60 mg every 4 hours for 21 days, is correct. Nimodipine should be administered in all patients with aneurysmal subarachnoid hemorrhage for 21 days to reduce the risk of delayed cerebral ischemia and improve neurologic outcomes. Blood pressure control is important, and guidelines recommend using a titratable agent such as clevidipine to maintain a goal systolic blood pressure less than 160 mm Hg (Answer C is incorrect). The value of calcium antagonists other than nimodipine for vasospasm remains uncertain (Answers A and D are incorrect).

79
Q

A 62-year-old man is being treated for stage II bladder cancer with dose-dense MVAC (methotrexate, vinblastine, doxorubicin, and cisplatin) with colony-stimulating factor support (pegfilgrastim). The patient also receives fosaprepitant, dexamethasone, and ondansetron before each cycle for nausea/vomiting prevention. After cycle 3, the patient calls the clinic reporting significant bone pain in his legs. Which agent most likely caused this patient’s bone pain adverse event?

A	Ondansetron.
B	Pegfilgrastim.
C	Fosaprepitant.
D	Cisplatin.
A

B - When colony-stimulating factors (i.e., pegfilgrastim) begin to take effect, there is a potential for bone pain as the bone marrow increases activity (Answer B is correct). Ondansetron and other serotonin-3 antagonists do not usually cause bone pain. Instead, ondansetron is more likely to cause headaches, constipation, and QTc prolongation (Answer A is incorrect). Fosaprepitant is likely to cause hiccups and diarrhea, but not bone pain (Answer C is incorrect). Cisplatin is a cytotoxic chemotherapy agent that is more likely to cause myelosuppression, peripheral neuropathy, nausea/vomiting, renal insufficiency, and ototoxicity than bone pain (Answer D is incorrect).

80
Q

Which phrase is most effective in initiating the teach-back approach during a counseling session on an insulin pen?

A	Do you understand how to use your new insulin pen?
B	During this session, I will provide step-by-step instructions on how to use your new insulin pen.
C	Just to make sure we are on the same page, did this education session improve your understanding of how to use your new insulin pen?
D	I want to make sure that I explained things clearly. Can you explain to me how you plan to take your insulin using your new pen device?
A

D - teach back method with open ended questions

81
Q

J.C. is a 52-year-old woman who presents to the clinic with concerns of recent episodes of what she believes may be seizures. During these episodes, her mouth makes chewing motions, and she has repetitive movements of her right arm. J.C. is initiated on phenytoin, and you learn that she has hypertension, anemia, and chronic renal failure (hemodialysis three times weekly). She is receiving phenytoin 600 mg orally daily in the evening, and a trough phenytoin concentration is 18.4 mg/L. Which is the best assessment of her phenytoin therapy?

A	Her concentration is fine, but her dose should be changed to 300 mg in the morning and 300 mg in the evening.
B	Her concentration is too high, and her dose should be lowered to 500 mg orally once daily.
C	Her concentration is fine, but she should take her dose only on days of hemodialysis (after the dialysis session).
D	Her concentration is too high, and her dose should be lowered. She should not receive more than 400 mg in a single dose.
A

D - Although the patient’s phenytoin concentration is in the therapeutic range (10–20 mg/L), her concentration must be adjusted for her chronic renal failure . Because she has chronic renal failure and is receiving hemodialysis, her corrected concentration will be well above the therapeutic range (Answers A and C are incorrect). In addition, phenytoin has nonlinear absorption, and patients should not receive single doses of more than 400 mg (Answer D is correct; Answer B is incorrect). Phenytoin is not eliminated by hemodialysis, and dosing should be daily, even in patients with chronic renal failure receiving dialysis (Answer C is incorrect).

82
Q

B.R. is a 62-year-old woman (height 66 inches, weight 91 kg, BMI 32.3 kg/m2) with type 2 diabetes and a history of seizures. Her diabetes is reasonably controlled, with her last A1C being 6.9%. She has concerns of numbness, burning, and tingling in both feet. On examination, she has decreased sensation to the top of her ankles and is given a diagnosis of diabetic neuropathy. Which treatment would be best for B.R.?

    A	Valproate.
B	Nortriptyline.
C	Pregabalin.
D	Tramadol.
A

C - Valproate (Answer A) might be reasonable, given the patient’s history of seizures; however, it causes weight gain, which could compromise her diabetes control. In a patient with a history of seizures, tramadol (Answer D) is not advised because of its potential to cause seizures, and it is not a recommended treatment for diabetic neuropathy. Nortriptyline (Answer B) would be a possibility, but recent guidelines recommend that it not be used initially because of its adverse effects and limited supporting data. Recent guidelines consider pregabalin (Answer C) a first-line drug for diabetic neuropathy.

83
Q

L.H., a 74-year-old woman (height 66 inches, weight 82 kg) whose medical history is significant for rheumatoid arthritis (RA), type 2 diabetes, gastroesophageal reflux disease, and hypothyroidism, presents to the clinic with inflammation of the joints of the hands and stiffness lasting 1–2 hours in the morning. She is a current smoker. Her medications include pantoprazole 40 mg daily, metformin 850 mg twice daily, levothyroxine 100 mcg daily, folic acid 1 mg daily, methotrexate 12.5 mg weekly, naproxen 500 mg twice daily, calcium 600 mg twice daily, and vitamin D 1000 units twice daily. Her laboratory tests show a negative rheumatoid factor but positive anti–cyclic citrullinated peptides. The physician determines this is a flare of moderate disease. Three months later at L.H.’s follow-up appointment, her physician decides to change her RA regimen and prescribe a biologic agent. Which additional assessment is most indicated right now?

A	Skin cancer screening.
B	DEXA (dual-energy x-ray absorptiometry) scan.
C	PPD (purified protein derivative) skin test.
D	Colonoscopy.
A

C - Infection risk is elevated in patients receiving biologic disease-modifying antirheumatic drugs (DMARDs) for RA. In considering modifying the therapy to include a biologic agent, this patient should be evaluated for tuberculosis (Answer C is correct). Patients with RA are more likely to develop malignancy and osteoporosis than patients without these conditions; however, screening for skin cancer or colon cancer by colonoscopy or evaluating bone density for osteoporosis is not indicated when initiating a biologic DMARD (Answers A, B, and D are incorrect).

84
Q

B.J. is a 73-year-old man recently given a diagnosis of atrial fibrillation. He has hypertension, which is treated with enalapril. In addition, he has type 2 diabetes. Which stroke prevention treatment would best be initiated in B.J.?

A	Apixaban.
B	No treatment.
C	Clopidogrel.
D	Ticagrelor.
A

A - With atrial fibrillation, the CHA2DS2-VASc is used to determine the need for anticoagulation. This patient’s score is 3, indicating the need for an oral anticoagulant; therefore, Answer B is not an option. Clopidogrel (Answer C) can be used in secondary prevention of stroke caused by factors other than atrial fibrillation. Ticagrelor (Answer D) has not been shown effective in stroke prophylaxis. Apixaban (Answer A) is an oral anticoagulant with good efficacy in atrial fibrillation.

85
Q

A 64-year-old woman is seen by her primary care physician for an annual follow-up of her chronic conditions. She received a dose of herpes zoster live-attenuated vaccine (ZVL) at her visit 3 years ago and had a fully resolved episode of shingles 6 months ago. Which is the best recommendation to provide the patient today regarding the zoster vaccine?

A	No additional doses are needed.
B	One dose of recombinant zoster vaccine (RZV) is necessary to complete the series today.
C	Two doses of RZV are recommended 4 weeks apart.
D	Two doses of RZV are recommended 2 months apart.
A

D - According to the CDC guidelines, the RZV two-dose series is recommended for all adults 50 and older. Patients who previously received a dose of ZVL and/or those who have had prior episodes of herpes zoster infection should still be vaccinated with RZV (Answer A is incorrect). A two-dose RZV series is recommended regardless of previous vaccination with ZVL (Answer B is incorrect). A two-dose RZV series is recommended 2–6 months apart (Answer C is incorrect; Answer D is correct).

86
Q

J.B. is a 3-year-old boy who presents to the ED with his parents. He has severe neck pain, phonophobia, and photophobia. He has no history of any medical conditions or known drug allergies. He is behind on all of his vaccinations because his parents are concerned that they cause autism. A lumbar puncture shows glucose 44 mg/dL (peripheral 98 mg/dL), protein 184 mg/dL, and WBC 500/mm3. The CSF culture is growing Streptococcus pneumoniae, and sensitivities are pending. Which empiric antibiotic regimen is best for J.B. at this time?

A	Penicillin G intravenously for 7 days.
B	Ceftriaxone intravenously for 10 days.
C	Ceftriaxone intravenously plus vancomycin intravenously for 10 days (narrow therapy, if possible).
D	Ceftriaxone intravenously plus vancomycin intravenously plus ampicillin intravenously for 7 days (narrow therapy, if possible).
A

D - Using penicillin alone is inappropriate therapy because the S. pneumoniae could be resistant. There must be results from culture and sensitivity testing before using penicillin alone. In addition, pneumococcal meningitis must be treated for 10–14 days (Answer A is incorrect). Using ceftriaxone alone is inappropriate therapy because the S. pneumoniae could be highly resistant to penicillin and therefore be resistant to ceftriaxone as well. Again, results from culture and sensitivity testing are necessary (Answer B is incorrect). Using ceftriaxone and vancomycin together is the best empiric regimen (Answer C is correct). Vancomycin is included in case the organism is highly resistant to penicillin (and therefore ceftriaxone too). Vancomycin can be discontinued later once culture and sensitivity results show no high-level penicillin resistance. Treatment duration in Answer C is also appropriate. Adding ampicillin to the regimen is not necessary in patients older than 2 years because Listeria is no longer a concern in these older children (Answer D is incorrect).

87
Q

A randomized, phase III noninferiority trial evaluated a new factor Xa inhibitor (terixaban) against warfarin in patients with atrial fibrillation. Rates of the primary outcome (stroke and systemic embolism) were 1.79% in the warfarin group and 1.59% in the terixaban group (RR 0.89; 95% CI, 0.72–1.24). The predefined noninferiority margin was set at 1.58, expressed as RR. Which conclusion is best, given the results provided?

A	Terixaban is noninferior to warfarin.
B	Terixaban is inferior to warfarin.
C	Terixaban is superior to warfarin.
D	The results are inconclusive.
A

A - Given the noninferiority study design, terixaban is considered noninferior to warfarin according to the results because the upper limit of the 95% CI does not cross the noninferiority margin, making Answer A correct and Answer B incorrect. Given the results provided, superiority cannot be proven because further statistical testing is warranted (Answer C is incorrect). Answer D is incorrect because noninferiority has been demonstrated, according to the results presented.

88
Q

Which scenario would be most appropriate to report to MedWatch?

A	Anaphylaxis immediately after administration of the influenza vaccine.
B	Bleeding in a major trauma patient receiving a new oral anticoagulant.
C	Respiratory failure after a patient received a 10-fold increased sedative dose for a procedure.
D	Similar product labeling and packaging of intravenous hydromorphone and morphine.
A

D - MedWatch is the FDA’s medical product safety reporting program. Bleeding as a result of a major trauma while on a new oral anticoagulant is an expected adverse event because of the trauma and would not be the best option to report to MedWatch (Answer B is incorrect). A medication dosing error resulting in patient harm should be reported to the institution through the designated reporting system (Answer C is incorrect). Vaccine adverse events should be reported through the Vaccine Adverse Event Reporting System (VAERS) (Answer A is incorrect). Product labels or packaging that looks alike and may mistakenly be interchanged resulting in patient harm should be reported to MedWatch (Answer D is correct).

89
Q

Question 91
Which is the best reason that smart infusion pumps often incorporate “guardrails”?

A	Reduce risk of error because of an incorrectly programmed pump.
B	Restrict infusions to those within the hospital formulary.
C	Prevent a nurse from titrating infusions too rapidly.
D	Notify a pharmacist when an infusion goes outside the designated range.
A

A - Smart infusion pumps are used to improve patient safety and minimize the risk of error because of an incorrectly programmed pump, which could lead to a severe overdose. When the infusion rate is set outside the designated guardrail range, it will alert the user to confirm or reenter the correct rate (Answer A is correct). A user can select a generic entry to administer a drug not in the medication library (Answer B is incorrect). Smart pumps help facilitate titrating infusions, and guardrails alert and prevent a user from administering a medication beyond the designated range. However, this will not prevent a user from titrating too rapidly (Answer C is incorrect) and will not notify a pharmacist when the infusion rate is outside the range (Answer D is incorrect).

90
Q

K.L. is a 52-year-old postmenopausal woman with a history of hypertension, gastroesophageal reflux disease, and breast cancer at age 46 that resulted in a bilateral mastectomy. As part of her therapy, she received tamoxifen for 5 years; however, she completed therapy 1 year ago and is in full remission. She has no other history of surgical procedures. She is experiencing hot flashes that interfere with her mood. Which is the best treatment for her hot flashes?

A	Paroxetine 7.5-mg tablets.
B	Ospemifene 60-mg tablets.
C	17β-Estradiol 0.025-mg transdermal patch.
D	Estradiol 1 mg and progesterone 100-mg capsules.
A

A - The patient has symptoms of menopause, including hot flashes, which are affecting her quality of life. Because of the hot flashes, systemic therapy is warranted. The patient has a history of breast cancer, making estrogen use a contraindication for therapy (Answers C and D are incorrect). Ospemifene only addresses vaginal symptoms (Answer B is incorrect). Answer A is the best choice because it is a nonhormonal therapy. She no longer takes tamoxifen, so there is no concern for a drug interaction with paroxetine at this time, making Answer A correct.

91
Q

Results from a large phase III trial of a new diabetes medication found that A1C was reduced by 0.3% with treatment compared with placebo. The reported p value is less than 0.05. A patient has a current A1C value of 14%. Which explanation would be best to provide to this patient?

A	The medication is statistically better than placebo and should be started.
B	The medication is not statistically better than placebo and should not be started.
C	The medication is clinically better than placebo and should be started.
D	The medication is not clinically better than placebo and should not be started.
A

D - The p value of less than 0.05 indicates that a statistically significant difference was found between the medication and placebo (Answer B is incorrect). A reduction in A1C of 0.3% is not clinically significant for most people, particularly for someone with a current A1C of 14% (Answer D is correct; Answer C is incorrect). Although the medication is statistically better than placebo, it should not be started in this patient because of the patient’s current A1C value (Answer A is incorrect).

92
Q

A 65-year-old man with a diagnosis of stage IV metastatic pancreatic cancer returns to the clinic today. At diagnosis, his performance status was normal, and he was initiated on the regimen FOLFIRINOX (oxaliplatin, irinotecan, leucovorin, and fluorouracil). The patient arrives for a follow-up today before starting cycle 3 of chemotherapy. The patient states that he is feeling more fatigued than usual and can no longer be as active as he would like to be. His current laboratory values are as follows: Na 147 mEq/L, K 3.8 mEq/L, Cl 102 mEq/L, HCO3 24 mmol/L, BUN 31 mg/dL, SCr 1.18 mg/dL, glucose 108 mg/dL, WBC 4.6 × 103 cells/m3, Hgb 9.2 g/dL, Hct 27.6%, Plt 180,000/mm3, and absolute neutrophil count 2400 cells/mm3. Which statement regarding this patient is most accurate?

A	He is not eligible to receive darbepoetin because his Hgb concentration is above 9 g/dL.
B	He is being treated in the palliative setting for metastatic cancer and is therefore not eligible to receive darbepoetin.
C	He is symptomatic, and chemotherapy should be held until symptoms have resolved.
D	He is symptomatic, and treatment with darbepoetin alfa 500 mcg every 3 weeks should be considered.
A

D - Because the patient has metastatic disease that is incurable, he is being treated in the palliative setting. Erythropoiesis-stimulating agents such as darbepoetin alfa are appropriate for patients with chemotherapy-induced anemia when the Hgb concentration has decreased to below 10 g/dL and the patient is still receiving chemotherapy (Answer D is correct; Answers A and B are incorrect). Although the patient is symptomatic and reports some fatigue, his laboratory values are within normal treatment values, and he should continue with treatment without delay. In addition to initiating an erythropoiesis-stimulating agent, dose reduction of the chemotherapy could be considered for continued or worsening fatigue, but treatment should not be held (Answer C is incorrect).

93
Q

You are evaluating a study to assess a new formulation of pancreatic enzymes for patients with chronic pancreatitis. You have enrolled patients who have symptoms of abdominal pain and bowel habits related to chronic pancreatitis and who are not currently being treated for their symptoms. At the start of the study, the patients completed questionnaires regarding their pancreatitis symptoms over the past month. The subjects then received open-label treatment with the new pancreatic enzyme for 1 month. At the end of the study, subjects completed the same questionnaires regarding their pancreatitis symptoms during the trial. Which type of bias is of most concern regarding this study design?

A	Channeling.
B	Selection.
C	Publication.
D	Recall.
A

D - The subjects in this trial were asked to recall their symptoms during usual conditions (at the start of the study) and after knowingly receiving therapy with the open-label study drug at the end of the study. This study design is subject to recall bias because the outcomes rely on the subjects’ retrospective assessment of their symptoms (Answer D is correct). Recall bias may be mitigated by selecting objective measures for outcomes of interest and assigning patients to blinded treatment, when possible. Channeling bias results from assigning patients with more severe symptoms to a given study arm with the goal of providing them with the most effective treatment (Answer A is incorrect). Randomization of study subjects may minimize channeling bias. Selection bias occurs when specific patients are recruited and chosen to participate in a trial (Answer B is incorrect). Specific inclusion and exclusion criteria may minimize selection bias. Publication bias is because of the lack of publication of negative or neutral study results, leading to a skewed representation of data in the literature (Answer C is incorrect).

94
Q

A 42-year-old woman wants to stop smoking. She has a 20 pack-year history and currently smokes ½ pack/day. She has her first cigarette with her coffee about an hour after awakening. Her medical history includes type I bipolar disorder, hypertension, and asthma. Her current medications include divalproex sodium, quetiapine, amlodipine, ethinyl estradiol/etonogestrel (NuvaRing), budesonide/formoterol, and albuterol hydrofluoroalkane (used once weekly). Trials of venlafaxine and trazodone each previously induced a manic episode. She tried nicotine lozenges but did not like them because they gave her terrible heartburn when she chewed and swallowed them. Which would be best for her tobacco cessation?

A	Bupropion.
B	Nicotine inhaler plus nicotine 14-mg patch.
C	Nicotine 21-mg patch.
D	Varenicline.
A

D - This patient was likely using the nicotine lozenge incorrectly, given her adverse effects. She would be a reasonable candidate for nicotine replacement therapy, but the options listed are not correct for her. Because the patient smokes ½ pack/day, the correct starting dose for the nicotine patch is 14 mg (Answer C is incorrect). Although nicotine replacement is more effective when the patch is offered with a breakthrough formulation for cravings (e.g., the gum, lozenge, or inhaler), the nicotine inhaler is not indicated for patients with a history of asthma (Answer B is incorrect). Antidepressants have triggered manic episodes in this patient, so bupropion would not be a good choice (Answer A is incorrect). Varenicline is appropriate in this patient. The EAGLES trial showed that the incidence of neuropsychiatric symptoms with varenicline was low, even in patients with a history of psychiatric conditions (Answer D is correct).

95
Q

A 72-year-old man with nonischemic cardiomyopathy (ejection fraction 25%–30%) and atrial fibrillation presents to the ED with shortness of breath and difficulty breathing. He has gained 9 kg (20 lb) in the past 3 weeks, despite strict adherence to both dietary restrictions and medications. His vital signs include blood pressure 125/85 mm Hg, heart rate 87 beats/minute, respiratory rate 24 breaths/minute, and SaO2 95% on 3 L of nasal cannula. Physical examination reveals a 12-cm jugular venous distention, regular rhythm and rate, crackles bilaterally, and 3+ bilateral lower-extremity edema. He states that his β-blocker dose was increased last month for better rate control. He received furosemide 40 mg intravenously × 1 with a moderate response in urinary output. Pertinent laboratory values include K+ 4.5 mEq/L, BUN 38 mg/dL, SCr 1.5 mg/dL (baseline 1.2 mg/dL), and INR 2.7. Cardiac enzymes are within normal limits × 2 sets. His home medications include lisinopril 40 mg once daily, metoprolol XL 100 mg once daily, and furosemide 40 mg twice daily. Which is the best medication change at this time?

A	Add nitroglycerin 10 mcg/minute.
B	Initiate a furosemide infusion at 40 mg/hour.
C	Decrease metoprolol to 50 mg once daily.
D	Discontinue metoprolol.
A

C - Titration of β-blocker therapy carries a risk of causing decompensation, particularly when done too quickly or when initiated in patients who are not euvolemic. This patient’s volume status at his last office visit, when his β-blocker was increased, is unknown; however, this should be a suspected cause of his presenting symptoms. If initial diuretic therapy does not rapidly improve his symptoms, the patient’s β-blocker can be reduced to the previous stable dose (Answer C is correct). Nitroglycerin would rapidly control symptoms if this patient had acute pulmonary edema or severe hypertension, but given his SaO2 and response to initial diuresis, it may not be needed at this time (Answer A is incorrect). Loop diuretics can be given as an intermittent bolus or a continuous infusion. Dosing of a continuous infusion should be initiated at 0.1 mg/kg/hour (10 mg/hour in this patient) and doubled every 4–8 hours, according to symptom control, to a maximum of 0.4 mg/kg/hour (40 mg/hour in this patient). Answer B is incorrect because furosemide should not be initiated at the maximum dose. β-Blockers should be held only in cases of hemodynamic instability (Answer D is incorrect).

96
Q

You are answering a drug information question and providing published literature to support your recommendation. One of the studies you used comes from a lesser-known journal, and a physician asks you the journal’s impact factor. You look it up and find that it is 1.54. Which is the best interpretation of that impact factor?

A	The journal has, on average, 1.54 articles indexed on PubMed from each issue.
B	Articles from the journal are cited 1.54 times more often than articles from the average medical journal.
C	Individual articles from the journal are cited in other articles, on average, 1.54 times in a particular year.
D	An article from the journal is cited, on average, in 1.54 therapeutic guideline publications in a particular year.
A

C - A journal’s impact factor provides information on the importance of that particular journal compared with other journals. It is a measure of the frequency with which the average article in a journal has been cited in a particular year (Answer C is correct). Most all articles from medical journals are indexed in PubMed (Answer A is incorrect). Answers B and D are both incorrect because they are not the appropriate definition of a journal’s impact factor.

97
Q

A clinical trial is conducted comparing ponesimod with teriflunomide in multiple sclerosis. The primary outcome is the annualized relapse rate. Ponesimod is found to result in fewer relapses, and the relative risk reduction is 0.69 (95% CI, 0.54–0.9; p<0.001). Which is the best interpretation of these results?

A	There were 69% more relapses in the teriflunomide group.
B	There were 31% more relapses in the ponesimod group.
C	There could be anywhere from 10% to 46% fewer relapses with ponesimod.
D	The results are not clinically significant because the 95% CI does not include 1.0.
A

C - A relative risk of 0.69 means that there were 31% fewer relapses with ponesimod in this clinical trial (Answers A and B are incorrect). Because the 95% CI is 0.54–0.9, it means that if this trial were to be repeated or if we were to use these drugs in clinical practice, we could predict that the actual decrease in annualized relapses with ponesimod would range from 10% to 46% (Answer C is correct). Statistically significant results do not necessarily mean the differences are clinically significant; that would require a judgment according to the actual number of decreased relapses found in the trial (Answer D is incorrect).

98
Q

R.H. is a 28-year-old woman with a 6-year history of migraine headaches. She takes sumatriptan at the start of her headaches, but with only minimal relief. All of her headaches occur around her menstrual period. Her headaches have increased such that she misses most of a week of work when they occur. She has never taken a prophylactic migraine medication. Which prophylactic medication would be best to initiate for R.H.?

A	Frovatriptan daily during her period.
B	Topiramate as needed.
C	Valproate daily.
D	Erenumab-aooe monthly.
A

A - Because she is consistently missing several days of work, prophylaxis for her headaches is needed. Topiramate (Answer B) would be effective; however, it must be given daily and has teratogenic effects. Therefore, topiramate might not be ideal for her, given her reproductive potential. Valproate (Answer C) is effective but also has teratogenic effects. Erenumab-aooe (Answer D) is a newer drug that is highly effective; however, it is expensive, and she has not taken a prophylactic medication previously. Frovatriptan (Answer A) can be given daily only during her period and has been shown effective when used in this way.

99
Q

Which best describes the approach for providing patient education that includes asking patients to state in their own words what you have explained?

A	Teach-back method.
B	Shared decision-making.
C	Show-me method.
D	Action planning
A

A - The teach-back method is a way to check for patient understanding by asking patients to state in their own words what they need to know or do for their health. It confirms that you have explained things in a way that your patient understands (Answer A is correct). Shared decision-making is the process of deciding about treatment plans that involves both the provider and the patient (Answer B is incorrect). The show-me method is similar to the teach-back method except it involves demonstration of correct use of a product such as an inhaler (Answer C is incorrect). Action planning outlines one or more steps a patient can take to attain a health goal (Answer D is incorrect).

100
Q

A.B. is a 76-year-old man with new-onset seizures. He loses consciousness during these spells, smacks his lips, and has purposeless movements of his right arm and hand. The spells last 1–2 minutes, and he is confused for several minutes afterward. Spells occur once or twice a week. He has mild dementia (Mini-Mental State Examination score 27/30), diabetes, hypertension, and mild congestive heart failure. His physician would like to initiate an antiepileptic drug (AED). Which would be best for A.B.?

A	Phenobarbital.
B	Levetiracetam.
C	Ethosuximide.
D	Phenytoin.
A

B - Older adults have the highest incidence of epilepsy, which presents unique problems when selecting a medication, including comorbid diseases and multiple medications. When selecting a new AED, potential harmful adverse effects and drug interactions should be minimized. Phenobarbital (Answer A) and phenytoin (Answer D) have several adverse effects, including sedation, ataxia, dizziness, and impaired cognition, making them suboptimal for older adults. Ethosuximide (Answer C) is indicated only for generalized nonmotor seizures (absence), a type of seizure that is inconsistent with the description provided. Levetiracetam (Answer B) has no drug interactions and fewer adverse effects than phenobarbital or phenytoin. Levetiracetam is indicated for focal seizures with loss of awareness (complex partial seizures).

101
Q

B.W. is a 62-year-old woman who presented to the hospital with jaundice and abdominal pain. She is given a diagnosis of a pancreatic mass, and a Whipple procedure is performed. She is now 3 days after surgery, in the ICU, and recently extubated. Her temperature increases to 101.3ºF (38.5ºC), WBC is 11 × 103 cells/mm3, and blood, urine, and sputum specimens are sent for culture. She is initiated on broad-spectrum antibiotics. The urine culture grows Candida albicans. B.W. has a urinary catheter but no urinary symptoms. Which is the best recommendation for treating this fungal infection?

A	Remove the urinary catheter, but there is no need for antifungal therapy.
B	Initiate amphotericin B bladder irrigation for 3 days.
C	Remove the urinary catheter, and initiate fluconazole 400 mg orally daily for 14 days.
D	Remove the urinary catheter, and initiate micafungin 100 mg intravenously daily for 7 days.
A

A - Because this patient is asymptomatic, there is no need to treat the positive Candida urine culture. Only if the patient had neutropenia or would be undergoing a urologic procedure would treatment be necessary. However, the urinary catheter should be removed to reduce that risk factor for candiduria (Answer A is correct). Amphotericin B bladder irrigations can be an effective and less-toxic treatment for candiduria than systemic amphotericin B administration, but recurrences are common and antifungal treatment in this case is unnecessary (Answer B is incorrect). Fluconazole would be appropriate if treatment were necessary, and the 14-day duration would be appropriate. But again, because she is asymptomatic, treatment is unnecessary (Answer C is incorrect). Micafungin does not achieve high-enough urinary tract concentrations to be effective for Candida UTIs (although there are case reports of its effectiveness for patients with resistant organisms or those who cannot tolerate azoles). Moreover, the treatment duration is too short, and ultimately antifungal treatment is unnecessary (Answer D is incorrect).

102
Q

K.B. is a 69-year-old White woman (height 64 inches, weight 64 kg) who denies alcohol and tobacco use. She walks 60 minutes a day and has a history of a deep venous thrombosis at age 65. She takes warfarin 2.5 mg orally daily. Her bone mineral density T-score is -2.3 at the hip and -2.1 at the spine. Her father had a hip fracture at age 71. Her FRAX score (10-year fracture probability) is 22%, and her 10-year probability of hip fracture is 5.5%. Which treatment recommendation is best for K.B.?

A	Alendronate 70 mg orally weekly plus calcium carbonate 1200 mg orally with vitamin D 600 international units orally daily.
B	Calcium carbonate 1200 mg orally daily plus vitamin D 800 international units orally daily.
C	Conjugated estrogens/bazedoxifene 0.45 mg/20 mg orally daily plus vitamin D 600 international units orally daily.
D	Denosumab 60 mg subcutaneously every 6 months plus calcium carbonate 1200 mg orally daily.
A

A - This patient has osteopenia because her T-scores are above -2.5 but below -1.0. Her FRAX score for 10-year major fracture probability is greater than 20%, and her probability of hip fracture is greater than 3%, indicating that she needs prescription therapy at this time, in addition to calcium and vitamin D supplementation (Answer B is incorrect). The drug of choice is a bisphosphonate plus vitamin D and calcium supplementation (Answer A is correct). The patient is younger than 71; thus, according to the Institute of Medicine, 600 international units of vitamin D would be appropriate. Conjugated estrogens/bazedoxifene is an option for the prevention of osteoporosis, but the patient has a history of a deep venous thrombosis, which is a contraindication for use of an estrogen product (Answer C is incorrect). Denosumab can be considered as a first-line agent in women with severe osteoporosis, but it is generally not used first line, particularly for a patient such as this, who does not have severe bone loss at this time (Answer D is incorrect).

103
Q

A study is designed to investigate the development of an event in a group of individuals during a year. In the results, the authors report that 100 events occurred per 20,000 person-years. Which measure best describes what the estimate represents?

A Risk ratio (RR).
B Incidence rate.
C Odds ratio (OR).
D Prevalence.

A

B - Incidence rate is a measure of the rate of developing a disease (Answer B is correct; Answer D is incorrect). Incidence rate is estimated by dividing the number of individuals who develop a disease by the total time these individuals were at risk of developing a disease. Prevalence is a measure of the number of individuals who have a condition/disease at any given time (Answer D is incorrect). Both RR and OR estimate the extent of association between exposure and disease (Answers A and C are incorrect).

104
Q

A 5-year-old girl with a history of behavioral issues was recently given a diagnosis of attention-deficit/hyperactivity disorder (ADHD) and initiated on behavioral treatment. Her symptoms include a lack of attention to detail, difficulty listening, trouble organizing her activities, being easily distracted, being forgetful, being fidgety, talking excessively, having frequent outbursts in class, and frequently losing her personal belongings. These behavioral issues were noted at home by her parents and at preschool by several of her teachers. Her family and school have been adherent to behavioral treatments, but improvement has been minimal. Her family and teachers worry this is impairing her ability to progress in her learning at school, and her parents would like to discuss starting her on a medication. Which is the best recommendation for managing the patient’s ADHD?

A	Behavioral therapy alone should be continued because the patient is too young to warrant pharmacologic treatment.
B	Initiate methylphenidate IR (Ritalin) twice daily.
C	Initiate amphetamine IR twice daily.
D	Initiate atomoxetine once daily.
A

B - For children 4–5 years of age, the American Academy of Pediatrics recommends initial management with behavioral treatment alone. If behavioral treatment fails to improve symptoms and there is moderate to severe disturbance in the child’s function, medications can be considered. In this case, the patient has not improved and is having impaired ability to learn at school, so medication can be considered (Answer A is incorrect). The recommended first-line agent for children 4–5 years of age is methylphenidate because of a larger body of evidence supporting its use at this age (Answer B is correct; Answers C and D are incorrect). Had this patient been 6 years or older, amphetamines would have been a reasonable alternative first-line agent. Non-stimulant agents, such as atomoxetine, are FDA approved for the treatment of ADHD; however, they are generally only considered after a trial of stimulant agents or if other special considerations warrant earlier use (e.g., comorbid tic disorder).

105
Q

You are speaking with a patient about whether she should start taking aspirin for primary prevention of myocardial infarction and stroke. Given her age, you tell her that the level of evidence to support her taking a daily aspirin is “C.” Which is the best interpretation of this recommendation?

A	The evidence is good that taking an aspirin daily will cause risks that outweigh the potential benefits.
B	The evidence is good that taking an aspirin daily will provide benefits that outweigh the potential risks.
C	The evidence is fair that taking an aspirin daily will provide benefits that outweigh the potential risks.
D	There may be benefits of taking a daily aspirin, but the risks are almost equally balanced with the benefits.
A

D - In evidence-based medicine, level A evidence states that there are good data to support a benefit over the potential risk (Answer B is incorrect). Level B evidence states that the benefits of a therapy outweigh the risk, but the evidence is only fair (Answer C is incorrect). Level C evidence states that the benefits and risks to a therapy are balanced and that the individual will usually need to decide whether to use the therapy (Answer D is correct). Level D evidence states that the data support that the risk of a therapy is greater than the potential benefit (Answer A is incorrect).

106
Q

A 55-year-old woman with type I bipolar disorder, type 2 diabetes, and gastroesophageal reflux disease presents to the ED with palpitations and dizziness. Her husband found her passed out on the floor and subsequently called 911. On presentation, her blood pressure is 96/55 mm Hg and heart rate is 180 beats/minute. An ECG is positive for a QTc of 520 milliseconds. Her mood disorder has been stabilized on ziprasidone and fluoxetine. She also takes empagliflozin, insulin glargine, metformin, and dexlansoprazole. She began taking azithromycin 3 days ago for a sinus infection. Which pair of drugs most likely interacted with azithromycin to produce the current clinical picture?

A	Fluoxetine and ziprasidone.
B	Fluoxetine and empagliflozin.
C	Dexlansoprazole and ziprasidone.
D	Empagliflozin and dexlansoprazole.
A

A - This patient has torsades de pointes secondary to a drug interaction. Azithromycin prolongs the QTc interval and causes torsades de pointes. Two other medications she takes, fluoxetine and ziprasidone, can also prolong the QTc interval. The combination of the three drugs is probably responsible for this patient’s current clinical picture (Answer A is correct). Neither dexlansoprazole nor empagliflozin prolongs the QTc interval (Answers B, C, and D are incorrect).

107
Q

A 21-year-old man presents with concerns of burning on urination and penile discharge for the past 3 days. His nucleic acid amplification test is positive for gonococcus and chlamydia. Which is the best recommendation for this patient?

A	Doxycycline 100 mg orally twice daily for 7 days.
B	Benzathine penicillin G 2.4 million units intramuscularly as a single dose and azithromycin 1 g orally once.
C	Cefixime 400 mg orally as a single dose.
D	Ceftriaxone 250 mg intramuscularly as a single dose and azithromycin 1 g orally once.
A

D - The patient’s laboratory tests were positive for gonococcus and chlamydia, so he will require treatment for both conditions that includes ceftriaxone 250 mg intramuscularly to cover gonococcus and azithromycin 1 g orally as a single dose to cover chlamydia (Answer D is correct). Doxycycline is appropriate for chlamydia but not gonococcus (Answer A is incorrect). Benzathine penicillin G is for the treatment of syphilis (Answer B is incorrect). Cefixime could be used in place of ceftriaxone if ceftriaxone is not an option; treatment of chlamydia with azithromycin would still be required (Answer C is incorrect).

108
Q

A 77-year-old woman presents to the clinic today for a follow-up and treatment for multiple myeloma. She currently receives treatment with carfilzomib, lenalidomide, and dexamethasone. The patient’s daughter, who accompanies her to the clinic today, states that her mother has been acting slightly confused lately. The patient’s laboratory values are Na 136 mEq/L, K 4.2 mEq/L, Cl 96 mEq/L, HCO3 29 mmol/L, BUN 21 mg/dL, SCr 0.8 mg/dL, glucose 122 mg/dL, calcium 12.4 mg/dL, and albumin 2.3 g/dL. Which is most appropriate to initiate at this time?

A	Calcitonin 200 units spray into one nostril daily.
B	Allopurinol 300 mg orally daily.
C	Zoledronic acid 4 mg intravenously once.
D	Hydration with normal saline at 200 mL/hour and monitor for worsening symptoms.
A

C - The patient has hypercalcemia of malignancy related to the multiple myeloma. Her corrected calcium concentration is currently 13.76 mg/dL, and she has signs of confusion. This would place her in the moderate-risk category because her corrected calcium is less than 14 mg/dL, and pharmacotherapy is needed together with hydration. Bisphosphonates such as zoledronic acid are standard initial therapy (Answer C is correct). Calcitonin is an appropriate option for patients with severe disease or patients who are symptomatic, but the route indicated for hypercalcemia of malignancy is subcutaneous, not intranasal. Intranasal calcitonin is only indicated for osteoporosis (Answer A is incorrect). Allopurinol is not indicated for the treatment of hypercalcemia of malignancy and is instead indicated for prophylaxis of tumor lysis syndrome (Answer B is incorrect). Although hydration with normal saline is necessary, it is not appropriate for her without the addition of pharmacotherapy with a bisphosphonate because she currently has moderate hypercalcemia of malignancy (Answer D is incorrect).

109
Q

A physician is looking for a tertiary resource providing information on the effect of sodium-glucose cotransporter 2 inhibitors on cardiovascular outcomes in patients with type 2 diabetes. Which resource is most appropriate to recommend?

A	A cohort study.
B	A review article summarizing several clinical trials.
C	An original research article describing a randomized controlled trial.
D	PubMed.
A

B - A tertiary resource is a summary or compilation of primary and secondary resources that provides an overview of a topic of interest. A review article summarizing several clinical trials is an example of a tertiary resource, making Answer B correct. Answers A and C are incorrect because a cohort study and a randomized controlled trial are examples of primary resources. PubMed is a secondary resource database used to identify primary literature, making Answer D incorrect.

110
Q

A 60-year-old man with hypertension and chronic kidney disease (CKD) with an estimated glomerular filtration rate (eGFR) of 25 mL/minute/1.73 m2 is seen in the outpatient clinic for a follow-up. His blood pressure is adequately controlled. Repeat laboratory studies are similar to previous measurements with serum sodium 142 mEq/L, serum potassium 4.2 mEq/L, CO2 22 mEq/L, BUN 34 mg/dL, SCr 3.2 mg/dL, serum phosphorus 6.2 mg/dL, serum calcium 9.6 mg/dL, and serum albumin 3.0 g/dL. His intact parathyroid hormone is 200 pg/mL. Which therapy is best to minimize this patient’s risk of CKD–mineral and bone disorder?

A	Sevelamer hydrochloride 800 mg orally three times daily with meals.
B	Aluminum hydroxide 300 mg orally three times daily with meals.
C	Cinacalcet 30 mg orally once daily with a meal.
D	Calcium carbonate 1250 mg orally three times daily with meals.
A

A - For patients with CKD category G4 (eGFR 15–29 mL/minute/1.73 m2), the goals for serum phosphorus and serum calcium are to be within the normal range (serum phosphorus 2.7–4.6 mg/dL; serum calcium 8.5–10.5 mg/dL). Persistently elevated phosphate concentrations warrant the initiation of phosphate binder therapy. This patient’s corrected calcium is in the high-normal range [measured calcium + (0.8)(4 – serum albumin) = 9.6 mg/dL + (0.8)(4 – 3.0) = 9.6 + 0.8 = 10.4 mg/dL], so it would be best to avoid a calcium-containing phosphate binder (Answer D is incorrect). Sevelamer can be used as phosphate binder, and the dose listed is an appropriate starting dose (Answer A is correct). Although aluminum hydroxide is an effective phosphate binder, it should be avoided, when possible, because of the risk of aluminum accumulation and toxicity (Answer B is incorrect). In contrast, cinacalcet is a calcimimetic used to treat hyperparathyroidism but should not be used for patients before correction of hyperphosphatemia (Answer C is incorrect).

111
Q

B.L. is a 68-year-old female patient who presents to the ED with new-onset atrial fibrillation. She describes symptoms of dizziness, dyspnea, and increased fatigue over the past week when the “fluttering in her chest began.” She has a history of dyslipidemia, hypertension (HTN), heart failure (HF) with reduced ejection fraction, and osteoarthritis. Her home medications include atorvastatin 20 mg daily, lisinopril 20 mg daily, metoprolol succinate 100 mg daily, acetaminophen 1000 mg three or four times daily, spironolactone 25 mg once daily, and furosemide 40 mg once daily. Her laboratory values are within normal limits, and her CrCl is 40 mL/minute/1.73 m2. Her current vital signs are heart rate 78 beats/minute and blood pressure 122/76 mm Hg. Which antithrombotic strategy would be best for B.L.?

A	Dabigatran 75 mg twice daily.
B	Aspirin 162 mg daily.
C	Rivaroxaban 20 mg daily.
D	Apixaban 5 mg twice daily.
A

D - B.L.’s CHA2DS2-VASc score is 4 (1 point each for age, HTN, HF, and female sex), and anticoagulant therapy is warranted to reduce the risk of stroke or systemic embolism (Answer B is incorrect). Of the anticoagulants listed, only apixaban is dosed correctly for this patient’s renal function, making Answer D correct. Dabigatran could be considered at a dose of 150 mg twice daily (Answer A), and rivaroxaban could be considered at a dose of 15 mg daily (Answer C), depending on renal function.

112
Q

A 38-year-old woman was treated 6 months ago with ranitidine for a suspected duodenal ulcer. Today, she describes recurrent epigastric tenderness during the day, which is usually relieved by antacids such as calcium carbonate. She also describes being awakened during the night with “gnawing” stomach pain. She states that she has been under a lot of stress at work lately. She reports a history of hives with penicillin and aspirin. She takes ibuprofen as needed for headaches and menstrual cramps. A urea breath test is positive for Helicobacter pylori. She is prescribed lansoprazole 30 mg orally twice daily. Which 14-day regimen is most appropriate to initiate together with lansoprazole for this patient?

A	Lansoprazole 30 mg twice daily is sufficient for her peptic ulcer disease.
B	Administer clarithromycin 500 mg twice daily and amoxicillin 1 g twice daily.
C	Administer clarithromycin 500 mg twice daily and metronidazole 500 mg twice daily.
D	Administer bismuth subsalicylate 525 mg four times daily, metronidazole 500 mg three times daily, and tetracycline 500 mg four times daily.
A

C - H. pylori is a known carcinogen; therefore, if patients test positive, they must be treated with appropriate antimicrobial therapy to eradicate the H. pylori (Answer A). Clarithromycin in combination with amoxicillin is an appropriate regimen; however, this patient reports an allergy to penicillin, making amoxicillin inappropriate (Answer B). Clarithromycin combined with metronidazole is an alternative option that can be used in the setting of penicillin allergy (Answer C). Quadruple therapy with bismuth subsalicylate, metronidazole, and tetracycline is recommended as first-line therapy or salvage therapy in patients who have not responded to a first-line triple regimen. However, this patient also reports an aspirin allergy, which is a contraindication to the use of bismuth subsalicylate (Answer D). If quadruple therapy is desired, a combination product containing tetracycline, metronidazole, and bismuth subcitrate salt rather than subsalicylate salt could be used.

113
Q

A 44-year-old woman has a BMI of 34 kg/m2 and a diagnosis of obesity. After lifestyle modifications were implemented, her weight decreased slightly but not sufficiently. Thus, despite her best efforts, liraglutide was initiated. Her liraglutide dose was titrated at weekly intervals, and she currently takes 1.8 mg once daily. She tried increasing the dose twice to 3 mg once daily but could not tolerate the nausea and vomiting at that dose; however, she states that the current dose is tolerable. Which recommendation is most appropriate for this patient?

A	Increase her physical activity and further improve her diet.
B	Increase liraglutide to 3 mg once daily.
C	Continue liraglutide at the current dose.
D	Discontinue liraglutide and select an alternative agent.
A

D - This patient cannot tolerate the therapeutic target dose of liraglutide 3 mg once daily. Hence, according to the current recommendations, this agent should be discontinued and an alternative agent selected (Answer D is correct). Answer A is incorrect because the patient has tried and been adherent to lifestyle changes, but they were ineffective alone. Answer B is incorrect because she has twice tried a dosage increase and could not tolerate it; a third attempt is not warranted. Answer C is incorrect because the only dose currently recommended for chronic use is 3 mg once daily, and if not tolerated, a lower dose is not recommended.

114
Q

A 45-year-old man presents to the ED with a severe headache and blood pressure 227/124 mm Hg. On examination, the physician reports bilateral retinal hemorrhaging. A chest radiograph is significant for early pulmonary edema. Laboratory results include SCr 2.4 mg/dL. His medical history is significant for steroid-dependent asthma and gout. Which regimen is most appropriate to manage this patient’s blood pressure?

A	Esmolol 25 mcg/kg/minute intravenously.
B	Nicardipine 5 mg/hour intravenously.
C	Labetalol 0.5 mg/minute intravenously.
D	Enalaprilat 2.5 mg intravenously every 6 hours.
A

B - Intravenous esmolol and labetalol should be avoided in a patient with steroid-dependent asthma because of their β-blocking properties (Answers A and C are incorrect). Enalaprilat is not the best choice for a patient with significant renal impairment (SCr 2.4 mg/dL); also, the onset and duration of enalaprilat do not allow rapid dose adjustments compared with nicardipine (Answer D is incorrect). Nicardipine, a calcium channel blocker, can be titrated easily and is the best choice for this patient, given his contraindication to β-blockade and impaired renal function (Answer B is correct). Care should be taken not to decrease the mean arterial pressure by more than 25% in the first hour.

115
Q

A.G. is a 52-year-old Hispanic woman with a medical history of hypertension. She has never smoked and reports rare alcohol consumption. Her current medications include enalapril 10 mg twice daily and amlodipine 5 mg daily. Her clinic vital signs include blood pressure 128/76 mm Hg and heart rate 66 beats/minute. She presents to the clinic today for her annual evaluation. Her laboratory values include a low-density lipoprotein cholesterol (LDL-C) of 238 mg/dL; all other laboratory values are within normal limits. A.G. returns to the clinic 3 months after starting statin therapy. She reports adherence to her diet and lifestyle modifications. Today, her LDL-C is 146 mg/dL. Which additional pharmacotherapy, if any, would best be recommended for A.G.?

A	No additional therapy is warranted.
B	Increase atorvastatin to 120 mg daily.
C	Add ezetimibe 10 mg daily.
D	Add evolocumab 140 mg subcutaneously every 2 weeks.
A

C - This patient’s LDL-C did not decrease by at least 50% and is still above 100 mg/dL, so additional therapy is warranted (Answer A is incorrect). Atorvastatin 80 mg is the maximum recommended dose, making Answer B incorrect. Ezetimibe (Answer C) is an appropriate add-on therapy for patients with severe hypercholesterolemia receiving maximally tolerated statin therapies. In the current dyslipidemia guidelines, evolocumab is not recommended until ezetimibe has already been initiated, making Answer D incorrect.

116
Q

A 69-year-old patient with chronic kidney disease (CKD) stage 5 on hemodialysis was initiated on calcium acetate 667 mg, 2 capsules three times daily with meals. During the past 2 months of therapy, serum phosphate has changed from 6.9 mg/dL to 5.2 mg/dL. Other current laboratory values include serum calcium 8.5 mg/dL and serum albumin 3.2 g/dL. Currently, the patient’s 25-hydroxyvitamin D concentration is 12 ng/mL and parathyroid hormone (PTH) concentration is 85 pg/mL, which is increased from 55 pg/mL 3 months ago. Which is best for managing this patient’s risk of CKD–mineral and bone disorder (CKD-MBD)?

A	Add sevelamer 1200 mg by mouth three times daily with meals.
B	Add ergocalciferol 50,000 units by mouth once weekly.
C	Add calcitriol 0.25 mcg by mouth once daily.
D	Increase calcium acetate to 3 capsules three times daily with meals.
A

B - Chronic kidney disease–mineral and bone disorder can develop from hyperphosphatemia, hypocalcemia, vitamin D deficiency, and hyperparathyroidism. This patient’s hyperphosphatemia has improved with the addition of calcium acetate to within the target serum phosphate range (3.5–5.5 mg/dL) for a patient with CKD stage 5. Therefore, adding another phosphate binder such as sevelamer would be unnecessary (Answer A is incorrect). Similarly, increasing calcium acetate would not be appropriate (Answer D is incorrect). Ergocalciferol should be added for this patient, who is deficient in vitamin D with a 25-hydroxyvitamin D concentration less than 30 ng/mL. The 2017 KDIGO CKD-MBD guidelines recommend that vitamin D deficiency and insufficiency be corrected using treatment strategies for the general population. Because the patient’s disease is in the mildly deficient range (5–15 ng/mL), weekly oral doses would be appropriate (Answer B is correct). An active form of vitamin D would only be added if the patient’s PTH concentration were severely and progressively elevated, despite control of phosphate and adequate vitamin D concentrations. The patient’s calcium concentration would also need to be low enough, less than 9.5 mg/dL, to safely initiate calcitriol (Answer C is incorrect).

117
Q

Question 119
An 85-year-old woman is hospitalized for hip replacement surgery after a fall with fracture. Her medical history is significant for hypothyroidism, osteoarthritis, and hyperlipidemia. Her current medications include simvastatin 20 mg daily, risedronate 35 mg once weekly, vitamin D 1000 international units daily, calcium 650 mg twice daily, levothyroxine 75 mcg daily, and oxycodone/acetaminophen 5/325 mg 1 or 2 tablets every 4 hours as needed for moderate or severe pain. While preparing for hospital discharge, the patient reports a new onset of “losing her water” the day before and again overnight to the pharmacist who is counseling her on her discharge medications. Which intervention would be most appropriate for this patient at this time?

A	Urinalysis.
B	Pelvic floor muscle exercises.
C	Tolterodine 2 mg daily.
D	Duloxetine 20 mg daily.
A

A - New symptoms of urinary incontinence in an older adult should be thoroughly evaluated to determine whether there is a reversible cause; infection, or the “I” in the mnemonic DRIP (drugs, delirium; restricted mobility/retention; inflammation/infection/impaction [fecal]; polyuria/prostatism), may be causing this woman’s new symptom. Urinalysis would be most appropriate for this reversible cause of incontinence (Answer A is correct). Tolterodine is used in urge incontinence that does not respond to an adequate pelvic floor muscle trial (Answer C is incorrect). Duloxetine has been used off-label for stress incontinence (Answer D is incorrect). Pelvic floor muscle exercises or Kegel exercises should be first-line therapy for stress, urge, or mixed incontinence in women (Answer B is incorrect).

118
Q

You are asked to provide an inservice presentation on the appropriate use of antibiotics in the treatment of UTIs. During the presentation, you want to assess the effectiveness of the education you are providing. Which option will best assess whether your audience can ultimately use antibiotics appropriately in clinical practice?

A	Ask verbal questions of the entire group and receive answers from a few of the audience members.
B	Have some audience members tell you the muddiest points of the presentation and then provide clarification.
C	Provide clinically applicable cases for the audience to discuss in groups and provide their recommendations to the entire audience.
D	Create an electronic poll with multiple-choice questions that audience members can respond to individually, followed by a review of the results.
A

C - All of these educational assessment options are potentially valid and appropriate. However, if you want to assess whether your audience can use the information provided in clinical practice, providing clinically applicable cases for discussion will best mimic how the audience members will use the information clinically (Answer C is correct). Asking verbal questions with only a few audience members interacting will not allow you to assess learning for the entire group (Answer A is incorrect). Having audience members ask about the muddiest points may clarify a few complicated points, but it will primarily assess what participants do not understand, rather than what they do understand (Answer B is incorrect). An electronic poll may provide feedback from more audience members (if they are motivated to respond), but multiple-choice questions are not as effective in helping the educator assess whether the information provided can be translated to clinical practice (Answer D is incorrect).

119
Q

The internal medicine team you round with would like to know whether a different drug has better efficacy than the standard of care medication used for treating heart failure with preserved ejection fraction. Data from which study design would best answer this question?

A	Cross-sectional study.
B	Case-controlled study.
C	Case series.
D	Cohort study.
A

D - Although randomized controlled trials are typically preferred to determine differences between drugs, they are not always available. Case series include only a small number of patients with similar experience; therefore, they are only hypothesis generating. Case series are one of the lowest levels of evidence in the hierarchy of clinical study design, making Answer C incorrect. Cross-sectional studies, also known as prevalence studies, identify the prevalence or characteristics of a condition in a group. Because this is not related to the clinical question being asked, Answer A is incorrect. Answer B is incorrect because case-controlled studies are designed to identify the association between an exposure and a condition. Of the observational study designs, data from cohort studies provide the highest level of data. Cohort trials provide the ability to evaluate patients, in a nonrandomized fashion, receiving a different or new drug compared with patients not receiving the agent but typically the standard of care. Therefore, Answer D is correct.

120
Q

Which source is most appropriate to consider as secondary literature?

A	Review article on the treatment of hepatorenal syndrome.
B	Chapter on anticoagulation in a pharmacotherapy textbook.
C	Observational study comparing the effectiveness of two insulin regimens.
D	Randomized controlled trial assessing a new analgesic to standard of care.
A

A - Question 122
Which source is most appropriate to consider as secondary literature?

A	Review article on the treatment of hepatorenal syndrome.
B	Chapter on anticoagulation in a pharmacotherapy textbook.
C	Observational study comparing the effectiveness of two insulin regimens.
D	Randomized controlled trial assessing a new analgesic to standard of care. Show/Hide Answer and Explanation Correct Answer: A Explanation: Original research published in peer-reviewed journals is considered primary literature (Answers C and D are incorrect). Textbooks are condensed summaries of material and considered tertiary literature (Answer B is incorrect). Review articles that summarize and synthesize primary literature are considered secondary literature (Answer A is correct).
121
Q

A 77-year-old African American man (weight 75 kg) presents to the ED with a 6-hour history of chest pressure (10/10 on a pain scale). His ECG reveals ST-segment depression in inferior leads. His medical history is significant for hypertension, dyslipidemia, and chronic kidney disease. Pertinent laboratory results are troponin 154 ng/L, SCr 4.5 mg/dL, and eCrCl 15 mL/minute/1.73 m2. The patient has been given aspirin 325 mg × 1; a nitroglycerin drip, initiated at 5 mcg/minute, will be titrated to chest pain relief and blood pressure. The cardiology team plans to take this patient to the catheterization laboratory for percutaneous coronary intervention (PCI).

Question 123
Which anticoagulation strategy is most appropriate to initiate in this patient?

A Intravenous heparin 4000-unit intravenous bolus, followed by a 900-unit/hour continuous infusion.
B Enoxaparin 60 mg subcutaneously every 12 hours.
C Fondaparinux 2.5 mg subcutaneously daily.
D Bivalirudin 45-mg bolus, followed by a 105-mg/hour infusion.

A

A - The non–ST-segment elevation acute coronary syndrome (NSTE-ACS) guidelines recommend the use of one anticoagulant during an acute event. Enoxaparin, fondaparinux, unfractionated heparin, and bivalirudin are all recommended as class I agents for NSTE-ACS. However, fondaparinux becomes less desirable when a patient is to be treated invasively during PCI. Increased risk of catheter-related thrombosis has been associated with fondaparinux use in the catheterization laboratory. The NSTE-ACS guidelines advise an additional anticoagulant if fondaparinux was chosen as an initial anticoagulant when the patient underwent intervention, whereas the PCI guidelines give fondaparinux a class III or harmful recommendation (Answer C is incorrect). Of the remaining options, unfractionated heparin dosed as a 60-unit/kg bolus with a 4000-unit maximum and a 12-unit/kg/hour infusion (maximum starting infusion of 1000 units/hour) is preferred because of its rapid clearance (Answer A is correct). Both enoxaparin (Answer B) and bivalirudin (Answer D) would need to be dose adjusted, given this patient’s CrCl of 15 mL/minute/1.73 m2. Dosages listed in Answers B and D are for patients with a normal CrCl (Answers B and D are incorrect).

122
Q

A 77-year-old African American man (weight 75 kg) presents to the ED with a 6-hour history of chest pressure (10/10 on a pain scale). His ECG reveals ST-segment depression in inferior leads. His medical history is significant for hypertension, dyslipidemia, and chronic kidney disease. Pertinent laboratory results are troponin 154 ng/L, SCr 4.5 mg/dL, and eCrCl 15 mL/minute/1.73 m2. The patient has been given aspirin 325 mg × 1; a nitroglycerin drip, initiated at 5 mcg/minute, will be titrated to chest pain relief and blood pressure. The cardiology team plans to take this patient to the catheterization laboratory for percutaneous coronary intervention (PCI).

Question 124
The patient is prepared for discharge. Which discharge regimen is most supported by the clinical evidence to reduce future cardiovascular events?

A	Nitroglycerin as needed for chest pain.
B	Spironolactone 25 mg daily.
C	Metoprolol for at least 3 years.
D	Aspirin for only 6 months.
A

C - After acute coronary syndrome (ACS), several medications will reduce the likelihood of future cardiovascular events. Of the options given, however, only metoprolol given for at least 3 years will reduce the future risk of ACS. β-Blockers limit infarct size, reduce remodeling after ACS, reduce the risk of sudden cardiac death, and reduce the risk of a subsequent myocardial infarction (MI). Data analyses suggest that the benefit of β-blockers is out to 3 years (Answer C is correct). Nitroglycerin does not reduce the onset of cardiovascular events but will help reduce angina in an ischemic attack (Answer A is incorrect). An aldosterone antagonist is most beneficial after an MI when the ejection fraction decreases below 35% (Answer B is incorrect because the patient’s ejection fraction and blood pressure are unknown). Answer D is incorrect because aspirin should be continued indefinitely for secondary prevention.

123
Q

A 77-year-old African American man (weight 75 kg) presents to the ED with a 6-hour history of chest pressure (10/10 on a pain scale). His ECG reveals ST-segment depression in inferior leads. His medical history is significant for hypertension, dyslipidemia, and chronic kidney disease. Pertinent laboratory results are troponin 154 ng/L, SCr 4.5 mg/dL, and eCrCl 15 mL/minute/1.73 m2. The patient has been given aspirin 325 mg × 1; a nitroglycerin drip, initiated at 5 mcg/minute, will be titrated to chest pain relief and blood pressure. The cardiology team plans to take this patient to the catheterization laboratory for percutaneous coronary intervention (PCI).

Question 125
The patient undergoes PCI and receives a drug-eluting stent to his right coronary artery. After appropriate antiplatelet loading doses, which is the best antiplatelet strategy at discharge for this patient?

A	Aspirin 325 mg indefinitely plus prasugrel 10 mg daily for 6 months.
B	Aspirin 325 mg indefinitely plus ticagrelor 90 mg twice daily for 6 months.
C	Aspirin 81 mg indefinitely plus clopidogrel 75 mg daily for 12 months.
D	Aspirin 81 mg indefinitely plus prasugrel 10 mg for 12 months
A

C - The choice of P2Y12 receptor antagonist in acute coronary syndrome (ACS) depends on patient presentation and contraindications, as well as on whether PCI is involved. The 2014 NSTE-ACS guidelines give a class I recommendation for clopidogrel, ticagrelor, and prasugrel in patients with ACS undergoing PCI. This patient is 77 years old, and the age-related caution with using prasugrel in this age group is to generally avoid the drug unless the patient has diabetes or an ST-segment elevation myocardial infarction (Answers A and D are incorrect). Although the loading dose of aspirin is generally 162–325 mg in the acute setting, the current guidelines state that lower doses of aspirin (i.e., 81 mg) are preferred to higher doses (i.e., 325 mg) at the time of discharge (Answer B is incorrect). Furthermore, ticagrelor (Answer B) has a boxed warning regarding reduced effectiveness when aspirin doses higher than 100 mg daily are used concomitantly. Finally, after drug-eluting stent placement in an ACS setting, the time recommended for dual antiplatelet therapy is at least 12 months (Answer C is correct).

124
Q

G.J. is a 51-year-old postmenopausal woman with concerns of dyspareunia (vaginal pain upon sexual intercourse) and vaginal dryness. She has been using OTC vaginal lubricants, which have not helped. She has a history of hypertension but is otherwise healthy with no surgical history. She currently takes metoprolol 50 mg orally daily and hydrochlorothiazide 25 mg orally daily. Which treatment is best to recommend for G.J.?

A	Conjugated estrogens 0.625 mg and medroxyprogesterone acetate 2.5-mg tablets (Prempro).
B	17β-Estradiol 0.025-mg transdermal patch (Climara).
C	Ospemifene 60-mg tablets (Osphena).
D	Paroxetine 7.5-mg tablets (Brisdelle).
A

C - This patient is experiencing local vaginal symptoms that are affecting her quality of life. She has a history of hypertension, which makes use of an estrogen-containing product less favorable; also, her symptoms are local and do not require systemic treatment (Answers A and B are incorrect). She does have a uterus; thus, if she did use an estrogen product, Answer B would be inappropriate because she would need a progestin in addition to the estrogen patch. Ospemifene is indicated for genitourinary symptoms associated with vaginal atrophy and would be appropriate in this patient (Answer C is correct). Paroxetine is used for vasomotor symptoms, not vaginal atrophy, and would not be an option in this patient (Answer D is incorrect).

125
Q

A new direct-acting antiviral, horteprevir, is being studied for the treatment of hepatitis C genotype 3. The trial is designed as a double-blind, active-controlled trial, with the treatment group receiving horteprevir 200 mg once daily in combination with ledipasvir and sofosbuvir for 8 weeks. The control group is receiving ledipasvir and sofosbuvir for a total of 8 weeks. The primary outcome is achievement of sustained virologic response 12 weeks after treatment. Which statistical test would best determine whether a statistically significant difference in the primary outcome exists between the two groups?

A	Student t-test.
B	Analysis of variance.
C	Chi-square.
D	Mann-Whitney U.
A

C - The primary outcome of whether virologic cure is achieved in either group constitutes a nominal outcome in which cure is either achieved or not achieved. Of the statistical tests listed, only the chi-square test (Answer C) is appropriate for use with two or more groups of nominal data. The Student t-test (Answer A) is a parametric test used to evaluate the difference between two independent groups of continuous, normally or near-normally distributed data. The analysis of variance (Answer B) is also a parametric test used to evaluate continuous, normally or near-normally distributed data from three or more independent groups. The Mann-Whitney U (Answer D) is a nonparametric test used to evaluate the difference between ordinal data and non-normally distributed continuous data derived from two independent groups.

126
Q

A 56-year-old man with a medical history of diabetes and hypertension controlled with metformin 1000 mg orally twice daily and lisinopril 20 mg orally once daily presents to the clinic with concerns of severe pain, redness, and swelling in the big toe of his right foot that have been present for 72 hours. He is given a diagnosis of an acute gouty attack for the second time in the past 12 months. No tophi are present. His serum uric acid concentration is 7.2 mg/dL, and his renal function is within normal limits.

Question 128
After 3 months, he returns to the clinic. His initial gouty attack resolved with the therapy that was recommended at the last visit. Today, he is given a diagnosis of another acute gouty attack in his right ankle and knee that began 48 hours ago. He rates the severity of this attack as a 3 on a 0-to-10 visual analog scale (with 10 being the worst pain). His serum uric acid concentration is 10.2 mg/dL. No tophi are currently present. He does not like having to take pills each day and asks about an injection to help with his acute gouty attack. Which is the most appropriate response regarding the use of intra-articular corticosteroids?

A	Contraindicated because of comorbidities.
B	Contraindicated because of duration since onset of symptoms.
C	Indicated because of joints involved.
D	Indicated because of severity of the attack.
A

C - Anti-inflammatory treatment in the form of an intra-articular corticosteroid injection can be considered in this patient. This acute gouty attack only involves two large joints, making it appropriate (Answer C is correct). Although the patient does have diabetes, intra-articular corticosteroid injections have only been shown to have short-term effects on hyperglycemia that are usually less than 1 week in duration. The patient should be instructed to more closely monitor his blood glucose concentrations, but the presence of diabetes is not a contraindication for intra-articular corticosteroid use (Answer A is incorrect). Attack severity and duration since onset of symptoms are not generally considerations in choosing oral anti-inflammatories versus intra-articular corticosteroids (Answers B and D are incorrect).

127
Q

A 56-year-old man with a medical history of diabetes and hypertension controlled with metformin 1000 mg orally twice daily and lisinopril 20 mg orally once daily presents to the clinic with concerns of severe pain, redness, and swelling in the big toe of his right foot that have been present for 72 hours. He is given a diagnosis of an acute gouty attack for the second time in the past 12 months. No tophi are present. His serum uric acid concentration is 7.2 mg/dL, and his renal function is within normal limits.

Question 129
Assuming this patient will be receiving acute anti-inflammatory therapy, which regimen is most appropriate for him today?

A	Allopurinol 100 mg once daily.
B	Febuxostat 80 mg once daily.
C	Lesinurad 200 mg once daily.
D	Probenecid 250 mg once daily.
A

A - Urate-lowering therapy (ULT) is currently indicated in this patient because he is experiencing his second acute gouty attack this year. Allopurinol and febuxostat are xanthine oxidase inhibitors (XOIs) that are appropriate as first-line ULT (Answer A is correct). However, the starting dose of febuxostat is 40 mg once daily (Answer B is incorrect). Lesinurad should only be used in combination with an XOI and is not appropriate as initial ULT. Answer D is incorrect because probenecid is a uricosuric, which is an alternative ULT if an XOI is contraindicated or cannot be tolerated.

128
Q

J.W. is a 32-year-old man who has been HIV positive for the past 6 years. Two years ago, he was given a diagnosis of cytomegalovirus (CMV) retinitis and treated accordingly. At the time of the CMV diagnosis, his CD4 count was 42 cells/mm3. In addition to treatment for CMV, he was initiated on prophylaxis for Pneumocystis pneumonia (PCP) and Mycobacterium avium (MAC) as well as antiretroviral therapy (ART). For the past 12 months, his CD4 count has been over 450 cells/mm3. Which is the best plan for prophylaxis of J.W.’s opportunistic infections?

A	Continue prophylaxis for CMV, PCP, and MAC because, at one time, J.W. had a low CD4 count.
B	Discontinue PCP and MAC prophylaxis, but continue CMV prophylaxis because he once had a diagnosis of the disease.
C	Discontinue CMV and MAC prophylaxis, but continue PCP prophylaxis because he is still at risk with a CD4 count less than 500 cells/mm3.
D	Discontinue CMV, PCP, and MAC prophylaxis because his immune system has adequately recovered with ART.
A

D - Because J.W.’s CD4 count is back above 200 cells/mm3 (and probably has been for at least 3 months), prophylaxis can be discontinued for CMV, PCP, and MAC (Answer D is correct). The CMV secondary prophylaxis can be discontinued if the CD4 count is greater than 100 cells/mm3 for 3–6 months (Answer B is incorrect). Primary prophylaxis for PCP and MAC can be discontinued if the CD4 count is greater than 200 cells/mm3 and 100 cells/mm3, respectively, for 3 months (Answers A and C are incorrect).

129
Q

A therapeutic intervention is touted as providing tertiary prevention against cardiovascular disease. Which is the best definition of this type of preventive health strategy?

A	Detecting a disease in its early stages, or subclinical form, often through screenings, to prevent/treat the disease before it progresses.
B	Reducing risk factors for a disease in an entire population through a focus on social and environmental conditions.
C	Using measures to prevent a disease from ever occurring in a susceptible individual.
D	Reducing negative clinical outcomes or long-term sequelae of a disease once it has been diagnosed.
A

D - Primordial prevention involves reducing risk factors in an entire population through social and environmental conditions (Answer B is incorrect). Primary prevention is using measures to prevent a disease from ever occurring, either in an individual or a population (Answer C is incorrect). Secondary prevention is detecting a disease in its early or subclinical phase and preventing it from progressing (Answer A is incorrect). Tertiary prevention is the practice of trying to reduce negative clinical outcomes or long-term sequelae of a disease once it has been diagnosed (Answer D is correct).

130
Q

A patient with severe chronic obstructive pulmonary disease reports increased anxiety and fatigue because of worsening dyspnea. Which type of cost is this best categorized as?

A	Direct medical.
B	Direct nonmedical.
C	Indirect.
D	Intangible.
A

D - Answer D is correct because increased anxiety and fatigue as the result of an illness are considered intangible costs and are challenging to measure. Direct medical costs are directly associated with the illness, such as hospitalizations, physician visits, or medications (Answer A is incorrect). Direct nonmedical costs are related to the illness, but are not medical in nature. An example of a direct nonmedical cost includes money spent on commuting to and from physician visits (Answer B is incorrect). Indirect costs are defined as costs as a result of productivity, including work absenteeism (Answer C is incorrect).

131
Q

In a recent clinical trial, the authors reported the relative risk reduction as 20% (95% CI, -5% to 45%). Which option best describes this finding?

A	Result is not statistically significant (p>0.05).
B	Result is not statistically significant (p>0.20).
C	With the results of this study, the authors have not excluded a 50% decrease in risk.
D	Result is statistically significant (p<0.05).
A

The 95% CI includes zero; therefore, the result of this study is not statistically significant because the true value of the difference between the risks could include zero. Answer A is correct because the 95% CI is provided, which refers to a p value of 0.05, not 0.20 (Answer B is incorrect). With the results of this study, the authors have excluded a 50% increase or decrease in risk (-50% and 50% are not contained in the CI) (Answer C is incorrect). The result is not statistically significant (Answer D is incorrect).

132
Q

A 6-month-old girl arrives at the ED with reports of seizure-like activity at home. She attends day care, and there have been reports of other children with hand-foot-and-mouth syndrome this past week. This morning, she had a temperature of 104ºF (40ºC) and, within the next hour, had abnormal movements of concern for seizure that lasted around 2 minutes. Her parents report that their older daughter, who is now 6 years old, had two febrile seizures in the past. Which is the best medical management for this patient for the prevention of recurrent febrile seizures?

A	Administer ibuprofen at the onset of future fevers.
B	No medical management is recommended.
C	Administer valproic acid for at least 6 months after this febrile seizure.
D	Administer oral diazepam at the onset of future fevers.
A

B - Simple febrile seizures are generally considered benign and have an excellent prognosis. Although the risk of recurrence is greatest for infants younger than 12 months, medical management is not generally recommended (Answer B is correct). Phenobarbital long-term therapy, primidone long-term therapy, valproic acid long-term therapy, and diazepam intermittent therapy have been shown to reduce the recurrence of simple febrile seizures; however, the risk of drug toxicities outweighs the benefits. Thus, they are not recommended by the American Academy of Pediatrics (Answers C and D are incorrect). Although oral diazepam intermittent therapy can be considered when parental anxiety is severe, it is not routinely recommended. Antipyretics, such as ibuprofen and acetaminophen, have not been shown to be effective in reducing the recurrence of febrile seizures; however, they can be used to reduce discomfort in patients (Answer A is incorrect).

133
Q

During lactation, a person is prescribed doxycycline 100 mg twice daily for a possible tick bite. They want to know whether it is safe to continue feeding their 3-month-old infant, or if they will need to supplement with formula while taking the antibiotic. Which place is best to search for information about the safety of doxycycline during lactation?

A	Google.
B	LactMed.
C	MommyMeds.com.
D	PubMed.
A

B - Google is not an appropriate place to search for reputable medical information, making Answer A incorrect. LactMed is the most appropriate place to search. LactMed is a database containing information about drug use during lactation and is provided by the NIH, making Answer B correct. MommyMeds.com is a consumer-focused website that contains information about some medications during lactation and may be appropriate in some situations; however, it is not the best choice of the options provided (Answer C is incorrect). PubMed is used to identify primary literature such as clinical trials and is not the best place to identify information about medication use in lactation (Answer D is incorrect).

134
Q

A research assistant for a clinical trial incorrectly records the time that a series of blood samples were obtained for 10% of trial participants. Which most accurately depicts the type of bias that has occurred?

A	Selection.
B	Recall.
C	Observational.
D	Interviewer.
A

C - Observational bias (sometimes called information bias) is an error in the recording of individual factors of a study, such as inaccurate recording of the timing of a blood sample – making Answer C correct. Selection bias occurs when subjects in the intervention and control groups differ in one or more characteristics that affect a study outcome (Answer A is incorrect). Recall bias occurs because of differences in the accuracy or completeness of the recollection of past events or experiences (Answer B is incorrect). Interview bias occurs when interviews are not conducted in a uniform manner (Answer D is incorrect).

135
Q

S.L. is a 58-year-old man who presents to his physician with symptoms that are determined to be erectile dysfunction. He has a history of type 2 diabetes. His medications include metformin 1000 mg orally twice daily. His total testosterone concentration is 905 ng/dL (reference range 270–1070 ng/dL), and his free testosterone is 20 ng/dL (reference range 9–30 ng/dL). His physician decides on a pharmacologic therapy plan for S.L.’s erectile dysfunction. Which recommendation would be best for this patient?

A	Alprostadil 500 mcg urethral suppository; use 5–10 minutes before intercourse.
B	Testosterone patch 2 mg/day transdermal system applied daily.
C	Sildenafil 50-mg oral tablet; use 1 hour before intercourse.
D	Sertraline 20-mg oral tablet; take 1 tablet orally daily.
A

C - This patient has no contraindications to the first-line therapy of phosphodiesterase type 5 (PDE-5) inhibitors. Alprostadil is not the first-line choice for erectile dysfunction (Answer A is incorrect). The patient’s testosterone concentrations are within the normal range; thus, he does not need testosterone supplementation (Answer B is incorrect). The PDE-5 inhibitors are first line for the treatment of erectile dysfunction, making sildenafil the best option for this patient (Answer C is correct). Sertraline is used for premature ejaculation and would not be appropriate for this patient (Answer D is incorrect).

136
Q

A 61-year-old white woman has a medical history of hypertension, type 2 diabetes, chronic stable angina, and gout. Her current medications include aspirin 81 mg daily, metformin 850 mg twice daily, atorvastatin 40 mg daily, metoprolol tartrate 100 mg twice daily, and colchicine 0.6 mg twice daily. She presents to the clinic today with continued chest pain when she gardens or walks her dogs. The pain is predictable and subsides after a few minutes of rest. She has no other concerns. Her clinic vital signs include blood pressure 136/76 mm Hg and heart rate 59 beats/minute. Her laboratory results are all within normal limits, including a serum creatinine of 1.1 mg/dL. Which medication adjustments, if any, would be best at this visit?

A	Change metoprolol tartrate to metoprolol succinate 200 mg daily.
B	Add amlodipine 5 mg daily.
C	Add ranolazine 500 mg twice daily.
D	No medication changes are warranted
A

B - This patient has symptoms of stable ischemic heart disease when she exerts herself and therefore warrants medication changes to reduce the frequency of chest pain (Answer D is incorrect). There is no need to change metoprolol tartrate to succinate for stable angina; also, the doses are equipotent, so no additional benefit would likely be gained (Answer A is incorrect). Adding a calcium channel blocker to a background of β-blockers is recommended to reduce anginal symptoms (Answer B is correct). Ranolazine is useful in patients who cannot tolerate further reductions in blood pressure or who are nonresponsive to β-blockers, calcium channel blockers, or long-acting nitrates. However, this patient might benefit from further blood pressure reduction (Answer C is incorrect).

137
Q

A U.S. pharmaceutical manufacturer is conducting a postmarketing study of a drug approved within the past 5 months. The manufacturer wants to determine whether this drug causes elevated blood pressure. Which design represents the best approach for studying the temporal relationship between the drug and elevated blood pressure?

A	Randomized controlled trial.
B	Case-control study.
C	Prospective cohort study.
D	Cross-sectional study.
A

C - To study temporal effects, only a randomized controlled trial (Answer A) or a prospective cohort study (Answer C) would be appropriate (Answer B is incorrect). Ethically, however, a randomized controlled trial (Answer A) would not be reasonable because of the negative outcome (elevated blood pressure) that would need to be investigated over a prolonged period (Answer C is correct). A cross-sectional study would only provide a snapshot in time and might not detect the outcome of interest (Answer D).

138
Q

A 5-year-old girl with no significant medical history presents for a pediatrician appointment in December. Her family has had some financial difficulty, so they have been unable to reliably keep housing or transportation. The patient’s immunizations were up to date until she was 3 years of age, but she has not received any immunizations since then because of a lack of follow-up care. Which immunizations would be best to recommend in the clinic today?

A	Hepatitis B, PCV13, DTaP, IPV, influenza vaccine, MMR.
B	DTaP, IPV, influenza vaccine, MMR, varicella.
C	Tdap, PCV13, IPV, influenza vaccine, MMR, varicella, HPV, hepatitis A.
D	DTaP, Hib, PCV13, IPV, influenza vaccine, MMR, varicella, hepatitis A.
A

B - The CDC provides an in-depth explanation of vaccine schedules, including recommendations for catch-up vaccinations and immunizations in special populations. The following vaccination series would have already been completed before 3 years of age, and thus, this patient would no longer require further immunization: hepatitis B vaccine (three-dose series at birth, 1–2 months, and 6 months), Hib (two-dose series at 2 and 4 months or three-dose series at 2, 4, and 6 months with a booster dose at 12–15 months), hepatitis A vaccine (two-dose series at 12 and 18–24 months), and PCV13 (four-dose series at 2, 4, 6, and 12–15 months) (Answers A and D are incorrect). The following vaccinations are not initiated until after 6 years of age: Tdap (starting age 11 years or 7 years if behind schedule for DTaP) and HPV (starting age 11 years) (Answer C is incorrect). Thus, the patient is due for DTaP, IPV, influenza, MMR, and varicella, according to the CDC schedule (Answer B is correct).

139
Q

A continuing education program provided at your institution is designed to improve adverse drug reaction reporting. During this program, you provide appropriate terminology to help clarify commonly confused terms. Which best depicts the case in which harm to the patient definitely occurs?

A	Adverse drug reaction.
B	Medication error.
C	Adverse effect.
D	Potential adverse drug event.
A

A - By definition, an adverse drug reaction is the only option when patient harm definitely occurs. The definition of an adverse drug reaction is harm directly caused by a drug at normal doses (Answer A is correct). An adverse drug reaction can be either expected or unexpected. A medication error (inappropriate use of a drug that may or may not result in harm) or adverse effect (usually a predictable or dose-dependent effect of a drug that is not the principal effect for which the drug was chosen; the adverse effect may be desirable, undesirable, or inconsequential) may cause harm but not all the time (Answers B and C are incorrect). A potential adverse drug event is defined as circumstances that could result in harm by the use of a drug but that did not harm the patient (Answer D is incorrect).

140
Q

A 23-year-old man (BMI 34 kg/m2) is brought to your hospital by ambulance after jumping from the balcony of an apartment building. He states he was following the instructions of the voices he heard that told him he was worthless and should kill himself. He has a 3-year history of schizophrenia. He previously developed acute dystonias on haloperidol and refuses to take it or similar medications. He did not respond to a therapeutic trial of ziprasidone. For the past 6 months, he has been receiving a paliperidone long-acting injectable every month (last dose was 3 weeks ago). His medical history is noncontributory. He normally lives with his parents, who are his legal guardians and want him to come live with them on discharge. Which medication would be best?

A	Brexpiprazole.
B	Clozapine.
C	Fluphenazine.
D	Risperidone.
A

B - This patient has treatment-resistant schizophrenia, as evidenced by his lack of response to two therapeutic trials of antipsychotics (aripiprazole and paliperidone). Treatment of choice for treatment-resistant schizophrenia is clozapine (Answer B is correct). Clozapine is also indicated in cases involving suicidal ideation or behavior. Fluphenazine, like haloperidol, is a high-potency first-generation antipsychotic that may have adverse effects similar to haloperidol (Answer C is incorrect). Risperidone is not likely a good choice for this patient because he did not respond to its active metabolite, paliperidone (Answer D is incorrect). Brexpiprazole is similar to aripiprazole, another agent to which the patient did not respond (Answer A is incorrect).

141
Q

High-intensity statins have been shown to have a 10-year number needed to treat (NNT10) of 20 for atherosclerotic cardiovascular disease (ASCVD) events. Which best interprets the NNT10?

A	20 people need to be treated for 10 years to prevent one ASCVD event.
B	20 people need to be treated for 1 year to prevent one ASCVD event.
C	20 people need to be treated for 10 years to prevent 10 ASCVD events.
D	20 people need to be treated for 1 year to prevent 10 ASCVD events.
A

A - The number needed to treat identifies how many people need to be treated over a time interval to prevent one event from occurring (Answers C and D are incorrect). This question is asking about NNT10, meaning that it is the number of patients needing to be treated for 10 years to prevent an event (Answers B and D are incorrect). Answer A is correct.

142
Q

A 72-year-old female patient being treated for stage IV ovarian cancer with metastasis to the liver is being followed in the pain management clinic. Her current pain regimen includes morphine sustained release 100 mg orally every 12 hours and oxycodone/acetaminophen 10 mg/325 mg 1 tablet orally every 4 hours as needed for pain. Today, in the clinic, the patient states her current pain rating is 5/10. She feels that lately, she has been having increased muscle spasms in her arms and back. Which is the most appropriate recommendation at this time?

A	Increase morphine sustained release to 120 mg orally every 12 hours and oxycodone/acetaminophen 10 mg/325 mg 2 tablets orally every 4 hours as needed for pain.
B	Continue current pain regimen and add baclofen 5 mg orally three times a day.
C	Discontinue morphine sustained release, initiate fentanyl transdermal patch 75 mcg/hour (applied every 72 hours), and continue oxycodone acetaminophen 10 mg/325 mg as breakthrough.
D	Continue current regimen with no changes. Her current pain score is 5/10, so her regimen need not be adjusted at this time.
A

B - The patient currently has muscle spasms. Opioids would not likely offer much benefit in treating muscle spasms; thus, increasing the dose of sustained-release morphine or changing to another opioid is not recommended (Answers A and C are incorrect). Baclofen is an adjunctive pain option that can treat muscle spasms while the patient continues on her current pain regimen (Answer B is correct). The patient should receive intervention for muscle spasms, regardless of the level of the pain she reports at the clinic today (Answer D is incorrect).

143
Q

If a drug following linear pharmacokinetics has an increase in its volume of distribution, which is the best conclusion?

A	Time to steady state will be increased.
B	Concentration at steady state will be increased.
C	Amount of a loading dose will need to be decreased.
D	Clearance of the drug will be decreased.
A

A - When the volume of distribution is increased, the elimination rate constant is decreased according to the equation k = Cl/Vd. When the elimination rate constant is decreased, the half-life is increased according to the equation t1/2 = 0.693/k. If the half-life is increased, the time to steady state is increased (Answer A is correct). The concentration at steady state is only a function of the dose and clearance, not of the volume of distribution (Answer B is incorrect). Because the amount of a loading dose is directly related to a drug’s volume of distribution, an increase in the volume of distribution would result in the need for a larger, not smaller, loading dose (Answer C is incorrect). Finally, clearance and volume of distribution are independent of each other; thus, an increase in the volume of distribution would not affect the clearance of the drug (Answer D is incorrect).

144
Q

A 19-month-old girl is brought by her father to her pediatrician after having visited yesterday. She initially presented with fever, poor oral intake, and irritability, and she was constantly sticking her finger in her left ear. Her diagnosis at that time was unilateral acute otitis media without otorrhea. The pediatrician had elected to delay antibiotic prescribing; however, today, her father feels like she is crying even more, she still has a fever, and she still will not eat. Her father reports an allergy to penicillin, which he states gave her diarrhea. Which would be the best recommendation for treating this patient?

A	Administer cefdinir 14 mg/kg/day oral solution for 10 days.
B	Administer amoxicillin 90 mg/kg/day for 10 days.
C	Continue to watch and delay antibiotic prescribing for another 24 hours.
D	Administer clindamycin 30 mg/kg/day for 10 days
A

B - Delayed antibiotic prescribing is an acceptable option for children older than 6 months. For children between 6 months and 2 years of age, this approach is recommended only if the otitis media is unilateral, the symptoms are mild, or the diagnosis is uncertain. For patients in whom delayed antibiotic prescribing is initiated, an antibiotic should be prescribed if symptoms worsen or if there is no improvement in 48–72 hours. In this case, the symptoms appear to be worse, so further treatment should not be delayed (Answer C is incorrect). Cefdinir or clindamycin monotherapy or with a third-generation cephalosporin may be considered in patients for whom first-line treatment regimens have failed, but this patient has not yet been initiated on antibiotics (Answers A and D are incorrect). Cefdinir would have been a reasonable option, however, had the patient had a true allergy to penicillin (diarrhea would be considered an intolerance and not a true allergy). The American Academy of Pediatrics recommends high-dose amoxicillin (80–100 mg/kg/day) as first-line therapy for acute otitis media (Answer B is correct).

145
Q

R.T. is a 42-year-old African American woman with a medical history of type 2 diabetes and migraine headaches. She has never smoked and reports rare alcohol consumption. Her current medications are metformin and canagliflozin. Her vital signs in the clinic include blood pressure 122/78 mm Hg and heart rate 66 beats/minute. Her lipid panel includes LDL 96 mg/dL, HDL 52 mg/dL, TC 168 mg/dL, and TG 98 mg/dL. All other laboratory values are within normal limits. She presents to the clinic today for her annual evaluation. You have been asked to determine what, if any, statin would be best to initiate for R.T. You calculate her 10-year atherosclerotic cardiovascular disease (ASCVD) risk to be 3.2%. Which is the most appropriate recommendation for R.T., according to the ACC/AHA recommendations for statin use for ASCVD prevention?

A	No statin therapy is warranted.
B	Initiate simvastatin 20 mg at bedtime.
C	Initiate atorvastatin 80 mg daily.
D	Initiate rosuvastatin 20 mg daily.
A

B - Because this patient has type 2 diabetes and is 40–75 years of age with an LDL less than 190 mg/dL, statin therapy is warranted, making Answer A incorrect. For patients 40–75 years of age with diabetes, ASCVD risk factors should be considered when making decisions about statin intensity. This patient does not have multiple ASCVD risk factors, so high-intensity statins are not required (Answers C and D are incorrect). Answer B represents a moderate-intensity statin and is the best choice for this patient.

146
Q

A woman with Cushing syndrome remains symptomatic despite surgery to remove the suspected pituitary adenoma. Her serum cortisol remains elevated, and she has menstrual irregularities (amenorrhea) and weight gain around her abdomen. She also states that her serum glucose concentrations have increased over the past month. Which agent would most help with her symptoms?

A	Ketoconazole.
B	Nadolol.
C	Propylthiouracil.
D	Pasireotide.
A

A - Patients who remain symptomatic after surgical intervention for Cushing syndrome may require pharmacotherapy to reduce cortisol concentrations and improve symptoms. Answer A, ketoconazole, can improve symptoms and reduce cortisol concentrations. Answer B, nadolol, is not used for Cushing syndrome but is effective for symptoms associated with hyperthyroidism. Answer C, propylthiouracil, is incorrect because it is also used for hyperthyroidism, not Cushing syndrome. Answer D is incorrect. Although pasireotide can be used for Cushing syndrome, the patient’s worsening glycemic control would make it less appropriate than mitotane.

147
Q

A 63-year-old patient presents to your institution after missing several dialysis sessions. Pertinent medical history includes end-stage renal disease, hypertension, and diabetes. When a cardiac monitor is placed on the patient, peaked T waves are appreciated. Urgent laboratory tests are obtained that show a potassium concentration of 6.8 mEq/L. Which is the best step in the initial treatment of this patient?

A	Insulin 10 units intravenously plus 50 g of dextrose 50% in water intravenously.
B	Sodium bicarbonate 50 mEq intravenously.
C	Albuterol 20 mg nebulized.
D	Calcium gluconate 1 g intravenously.
A

D - This patient presents with hyperkalemia with ECG changes, as evidenced by the peaked T waves. Before potassium is shifted intracellularly or eliminated, the cardiac membrane should be stabilized by calcium gluconate (Answer D). Answers A–C are all options to treat hyperkalemia but should be given after calcium gluconate.

148
Q

L.J. is a 36-year-old man who comes to the ambulatory care clinic with a 3-day history of fever, chills, pleuritic chest pain, malaise, and cough productive of sputum. In the clinic, his temperature is 102.1ºF (38.9ºC). His chest radiograph reveals consolidation in the right lower lobe. He is given a diagnosis of community-acquired pneumonia. He has had no antibiotics for more than 10 years and has no significant health history. The macrolide resistance rate in your community is less than 15%. Which would be the best empiric therapy for L.J.?

A	Azithromycin 500 mg, then 250 mg orally daily.
B	Cefuroxime axetil 250 mg orally twice daily.
C	Levofloxacin 750 mg orally daily.
D	Trimethoprim/sulfamethoxazole DS orally twice daily.
A

A - Cefuroxime has good activity against Haemophilus influenzae and Moraxella catarrhalis but none against atypical organisms (Mycoplasma, Chlamydia, Legionella). Moreover, its activity against Staphylococcus pneumoniae may be limited (if L.J. lives in an area with extensive drug resistance) (Answer B is incorrect). Although levofloxacin has excellent activity against atypical organisms, H. influenzae, M. catarrhalis, and S. pneumoniae (even drug-resistant S. pneumoniae), azithromycin or doxycycline is the best initial choice because L.J. has had no antibiotics recently and has no comorbidities (Answer A is correct; Answer C is incorrect).Trimethoprim/sulfamethoxazole does not have activity against atypical organisms but is active against H. influenzae and M. catarrhalis. Its activity against S. pneumoniae may be limited (if L.J. lives in an area with extensive drug-resistant S. pneumoniae) (Answer D is incorrect).

149
Q

T.D. is a 35-year-old woman originally from Laos who presents to the clinic for a purified protein derivative (PPD) test because a family member was recently given a diagnosis of tuberculosis (TB). The PPD is positive, but because she had the bacille Calmette-Guérin vaccine as a child, a T-SPOT TB (interferon-gamma release assay) test is also ordered, which is also positive. She has no symptoms of TB at this time (e.g., negative chest radiograph, no cough, no fever, no weight loss, no night sweats). If she does not want to take antibiotics for 6 months, which is the best option for T.D.?

A	Administer rifapentine 900 mg plus isoniazid 900 mg weekly for 3 months (with directly observed therapy).
B	Administer rifampin 600 mg daily for 2 months.
C	Administer isoniazid 300 mg daily plus rifampin 600 mg daily for 1 month (with directly observed therapy).
D	There are no regimens less than 6 months, so she should receive isoniazid 300 mg daily for 6 months.
A

A - The standard regimen for latent TB is isoniazid 300 mg daily for 6–9 months (preferably 9 months). This could be an option for this patient, but she does not want to take medications for 6 months, and there are, in fact, other options of either 3 or 4 months’ duration (Answer D is incorrect). Rifampin can be given alone, but the duration is 4 months, not 2 (Answer B is incorrect). No regimen is recommended for only 1 month (Answer C is incorrect). Rifapentine plus isoniazid is recommended on a weekly basis for 3 months (Answer A is correct).

150
Q

A 63-year-old male patient with recently diagnosed diffuse large B-cell lymphoma is currently being treated with R-CHOP (rituximab, cyclophosphamide, doxorubicin, vincristine, and prednisone) every 21 days. The patient is doing well on treatment and is in the clinic today for a follow-up after cycle 4 of chemotherapy. However, the patient is feeling fatigued. His laboratory values are Na 143 mEq/L, K 4.1 mEq/L, Cl 98 mEq/L, HCO3 26 mmol/L, BUN 18 mg/dL, SCr 1.1 mg/dL, glucose 103 mg/dL, WBC 785 cells/mm3, Hgb 11.4 g/dL, Hct 43.2 g/dL, Plt 130,000/ mm3, neutrophils 30%, and bands 10%. His Multinational Association of Supportive Care in Cancer (MASCC) index score is 16. His vital signs are oral temperature 102ºF, blood pressure 88/67 mm Hg, heart rate 97 beats/minute, and respiratory rate 21 breaths/minute. Which is the most appropriate treatment course for this patient?

A	He has low-risk febrile neutropenia and should be initiated on oral ciprofloxacin and amoxicillin/clavulanate in the outpatient setting.
B	He has high-risk febrile neutropenia and should be admitted to the hospital and initiated on vancomycin and amphotericin B.
C	He is at risk of developing febrile neutropenia and should continue to be monitored closely in the outpatient setting.
D	He has high-risk febrile neutropenia and should be admitted to the hospital and initiated on intravenous piperacillin/tazobactam.
A

D - The patient has a fever and is severely neutropenic with an absolute neutrophil count (ANC) of 314 cells/mm3. This would be considered high-risk neutropenia, given the patient’s low ANC concentration (Answers A and C are incorrect). According to the patient’s low MASCC index score, indicating increased severity risk, as well as the current guidelines and recommendations, he should be admitted to the hospital and initiated on a broad-spectrum single-agent antibiotic. Vancomycin is not typically recommended first line unless the patient has specific risk factors, and antifungal agents are not recommended unless the patient does not respond to initial antibiotic therapy (Answer B is incorrect). The antibiotics of choice are broad spectrum with antipseudomonal activity, including cefepime, carbapenems, and piperacillin/tazobactam (Answer D is correct).

151
Q

After completion of a clinical trial comparing apixaban with rivaroxaban for deep venous thrombosis/pulmonary embolism treatment, the study authors found that apixaban was statistically better at preventing recurrences (p=0.01). Before study initiation, the authors calculated they would need to enroll 3500 subjects in order for their study to have 80% power, but they ended up enrolling only 3100 subjects. Which is the best interpretation of the trial’s results?

A	The study must have found a larger difference in recurrence rates than initially expected, and the power was adequate to show a statistically significant difference.
B	The study results cannot be used to confidently predict that apixaban is better than rivaroxaban because of lack of power (not meeting the sample size calculation).
C	Because the number of enrolled patients was less than originally calculated, the power is lower than 80%, but the results can still confidently be assumed to be correct.
D	Because the number of enrolled patients was less than originally calculated, there may not be an actual difference between the two drugs.
A

A - Sample size is calculated before study initiation according to an assumed α error (p value), β error (1 – power), expected difference, and variability in the data. If the sample size is not met in a trial, the beta error will go up (power will go down) if everything else stays the same. Because this trial showed a statistically significant difference with fewer subjects than originally calculated, the difference the authors ultimately found must be greater than they originally expected (Answer A is correct). The study obviously had adequate power because it was able to show a difference (Answers B, C, and D are incorrect).

152
Q

C.C. is a 71-year-old white woman (height 62 inches, weight 54 kg) who denies alcohol and tobacco use. She has a history of hypertension and a fall that resulted in a wrist fracture at age 55. She does not eat many dairy products because of lactose intolerance. Her current medications include amlodipine 5 mg orally daily and furosemide 20 mg orally daily. Her bone mineral density T-score is -2.4 at the hip and -2.2 at the spine. Her FRAX score (10-year fracture probability) is 21%, and her 10-year probability of hip fracture is 5.4%. Which is best for C.C.’s osteoporosis?

A	Begin calcium carbonate 1200 mg orally daily, vitamin D 800 international units orally daily, and 30 minutes of exercise 5 days a week.
B	Begin 30 minutes of exercise 5 days a week and continue to monitor C.C.’s bone mineral density to initiate treatment if T-scores drop below -2.5.
C	Begin alendronate 70 mg orally weekly, calcium carbonate 1200 mg orally daily, vitamin D 800 international units orally daily, and 30 minutes of exercise 5 days a week.
D	Begin teriparatide 20 mcg subcutaneously daily, calcium carbonate 1200 mg orally daily, vitamin D 800 international units orally daily, and 30 minutes of exercise 5 days a week.
A

C - Given the patient’s FRAX score of 21% for a 10-year fracture probability and 5.4% for a 10-year hip fracture probability, together with her low bone mass (T-scores of -2.4 at the hip and -2.2 at the spine), pharmacologic treatment is indicated in addition to calcium, vitamin D, and exercise. Therefore, Answers A and B are incorrect. Given that the patient does not eat dairy products, she likely needs supplemental calcium. Bisphosphonates are first-line treatment, together with calcium, vitamin D, and weight-bearing exercise, making Answer C correct. Teriparatide is not first line for osteoporosis, does not prevent hip fractures, and is best for vertebral fractures, making Answer D incorrect.

153
Q

A pharmacy resident performed a medication use evaluation on a new agent for opioid-induced constipation. This agent was added to the hospital formulary 6 months ago without restriction criteria for use. Which metric would best evaluate and quantify a cost savings opportunity?

A	Total use over the 6-month period.
B	Quantity administered with incorrect doses.
C	Incidence of significant adverse drug events.
D	Quantity of avoidable doses.
A

D - Total use over the study period would display the total cost of use but not help identify a cost savings opportunity (Answer A is incorrect). The quantity administered with incorrect doses would show an opportunity for quality improvement but not significantly affect cost savings (Answer B is incorrect). The incidence of significant adverse drug events is an important metric to address safety improvement opportunities (Answer C is incorrect). Quantity and subsequent cost of avoidable doses would best reflect a cost savings opportunity (Answer D is correct).

154
Q

A 67-year-old man with a history of narcolepsy has difficulty falling asleep. He has established a regular bedtime for himself and practices good sleep hygiene. Despite this, it takes him 1–2 hours to fall asleep. Once he falls asleep, he is able to stay asleep for 8 hours. He has not tried any pharmacologic treatment. Which medication would be most appropriate for treating his insomnia?

A	Doxepin.
B	Ramelteon.
C	Suvorexant.
D	Temazepam.
A

B - This patient has difficulties with sleep latency. Although temazepam’s onset and duration of action would be correct for him, benzodiazepines are not recommended in older adults. This is particularly true for this patient, who has not tried anything for his insomnia (Answer D is incorrect). Doxepin can help decrease sleep latency, but it also has a long half-life that can lead to a hangover effect in the morning. This might further increase the patient’s challenges when he awakens in the morning (Answer A is incorrect). Suvorexant decreases sleep latency, which would help this patient, but it also improves sleep latency, which is not an issue in this patient, and might again increase his difficulty in getting out of bed in the morning. Suvorexant is also contraindicated in patients with narcolepsy (Answer C is incorrect). Ramelteon is a melatonin analog that helps decrease sleep latency and regulate the sleep cycle with minimal hangover or CNS adverse effects, making it the best option in this patient (Answer B is correct).

155
Q

A solid organ transplant pharmacy specialist is working to implement an effective patient counseling program on transplant immunosuppression. Which would best maximize the short-term effectiveness of the education?

A	Include educational handouts with pictures and verbiage tailored to a third-grade reading level.
B	Perform monthly follow-up calls with a clinical pharmacy specialist to ensure adherence.
C	Monitor prescription fill history quarterly to assess adherence.
D	Incorporate active learning and an assessment during each counseling session.
A

D - Conducting routine follow-ups and/or assessing fill history would not affect the short-term effectiveness of the education (Answers B and C are incorrect). Creating an engaging environment with education assessment would best increase effectiveness (Answer D is correct). Although handouts are beneficial, patients may still not understand the information, and the pharmacist may not gauge understanding without an assessment (Answer A is incorrect).

156
Q

T.G. is a 23-year-old woman with a history of cerebral palsy and a ventriculoperitoneal shunt who often receives antibiotics for pneumonia because she has a tracheostomy and is chronically mechanically ventilated. She has had significant, watery diarrhea for the past 3 days. The stool tests positive for Clostridioides difficile toxin. Her WBC is 13.2 × 103 cells/mm3, and her SCr and albumin are normal. She is not hypotensive or in shock and does not have an ileus. She was treated with oral vancomycin 125 mg four times daily for C. difficile for the first time 5 weeks ago. Which is the best antibiotic regimen for T.G.?

A	Fidaxomicin 200 mg orally twice daily for 10 days.
B	Vancomycin 500 mg orally four times daily for 10 days.
C	Rifaximin 400 mg orally three times daily for 14 days.
D	Metronidazole 500 mg intravenously every 8 hours for 14 days.
A

A - A first recurrence of C. difficile infection after treatment with vancomycin should be treated with either fidaxomicin or vancomycin. Because vancomycin was used first, fidaxomicin can be dosed normally for 10 days (Answer A is correct). If vancomycin is used, pulsed/prolonged dosing should be used. The dose is 125 mg four times daily for 10–14 days, two times daily for a week, once daily for a week, and then every 2 or 3 days for 2–8 weeks. A higher dose of vancomycin for only 10 days is not recommended (Answer B is incorrect). Rifaximin is recommended for the second or more recurrent infection (Answer C is incorrect). Metronidazole alone should not be used for C. difficile infections, especially recurrent infections (Answer D is incorrect).

157
Q

A 66-year-old woman with type 2 diabetes and rheumatoid arthritis is interested in using an intermittently scanned continuous glucose monitoring (isCGM) device. She currently takes insulin glargine 34 units once daily at bedtime and insulin aspart (10 units before breakfast, 12 units before lunch, and 12 units before dinner). She reports a fear of hypoglycemia because of recent events of blood glucose values decreasing below 70 mg/dL without presenting with symptoms. She self-monitors blood glucose with fingersticks four times daily; however, she reports that the frequency of testing is overwhelming because of manual dexterity issues. She presents today with an A1C of 8.9%. Which statement regarding CGM is most appropriate to discuss with the patient?

A	Studies have not shown the benefit of CGM in improving glycemic control in patients with type 2 diabetes.
B	There is a good correlation between the CGM metric, percent time in range (%TIR), and A1C.
C	Use of intermittent CGM negates the need for performing fingerstick testing.
D	Unlike real-time CGM devices, isCGM devices do not offer alerts and alarms for hypoglycemia.
A

B - Studies have shown that CGM improves glycemic control in patients with type 2 diabetes taking multiple daily insulin injections as well as in patients taking basal insulin alone (Answer A is incorrect). Use of CGM provides metrics such as %TIR, which has been found to correlate well with A1C and may allow for the transition of using this metric in assessing glycemic control and future risk of complications (Answer B is correct). Fingerstick confirmatory testing may still be needed in certain situations while using isCGM, including when hypoglycemia is present, glucose is rapidly changing, symptoms of high or low blood glucose are present, symptoms do not match system readings, and inaccurate readings are suspected (Answer C is incorrect). An isCGM system released in 2020 includes Bluetooth connectivity that enables high and low glucose alerts (Answer D is incorrect).

158
Q

The Surviving Sepsis Campaign uses the Grading of Recommendations Assessment, Development and Evaluation (GRADE) system to determine strength of recommendations and quality of evidence. The guidelines recommend norepinephrine as the first-choice vasopressor (strong recommendation, moderate quality of evidence). Which interpretation best reflects this designation?

A	Evidence consists of several observational studies with risk of bias.
B	Additional research is unlikely to change confidence in the estimate of clinical effect.
C	Evidence consists of randomized trials with some inconsistency and imprecision.
D	Policy-makers and stakeholders may have difficulty establishing a consensus on using this recommendation.
A

C - According to the GRADE system, a recommendation is designated as high quality of evidence when additional research is unlikely to change confidence in the estimate of clinical effect (Answer B is incorrect). Observational studies are initially designated as low quality of evidence, and a risk of bias may further lower this rating (Answer A is incorrect). Weak recommendations often lead to difficulty establishing a consensus on using the recommendation (Answer D is incorrect). Randomized trials are initially designated as high-quality evidence; however, they may be lowered to moderate quality of evidence with inconsistency and/or imprecision (Answer C is correct).

159
Q

L.P. is a 76-year-old man recently hospitalized for a heart failure exacerbation with significant fluid overload and severely restricted mobility. His other medical conditions include hypertension, coronary artery disease with a myocardial infarction 8 years ago, dyslipidemia, osteoarthritis, and gastroesophageal reflux disease. He is taking appropriate medications with some adjustment on admission. The team is discussing the potential need for extended prophylaxis for venous thromboembolism (VTE) diseases. Which would be most appropriate for L.P.?

A	Enoxaparin 40 mg subcutaneously daily for 35 days.
B	Rivaroxaban 10 mg daily for 35 days.
C	Unfractionated heparin 5000 units subcutaneously twice daily for 10 days.
D	Apixaban 2.5 mg twice daily for 35 days
A

B - This patient being hospitalized for acute medical illness (heart failure) and severe immobility would have been a patient in the APEX or MAGELLAN trials with betrixaban for 35–42 days or rivaroxaban for 31–39 days, respectively, compared with enoxaparin 40 mg subcutaneously for 10 days. In both trials, patients receiving betrixaban or rivaroxaban had a significant reduction in VTE events without an increase in major bleeding. Unfortunately, betrixaban is no longer commercially available. The ability to achieve no difference in major bleeding with rivaroxaban means avoiding patients with bronchiectasis, a gastroduodenal ulcer in the past 3 months, any bleeding in the past 3 months, and hospitalization for treatment of cancer and patients on dual antiplatelet therapy. Because this patient meets these criteria, Answer B is correct. Although extended prophylaxis with enoxaparin significantly reduced VTE, it significantly increased major bleeding. Therefore, Answer A is incorrect. Answer C (unfractionated heparin 5000 units twice daily) would also be incorrect because the duration is shorter, and there is question about the efficacy of unfractionated heparin 5000 twice daily, especially in higher-risk medically ill patients such as those with heart failure or stroke. Apixaban (Answer D) is incorrect because extended prophylaxis in medically ill patients with apixaban did not reduce VTE but still increased major bleeding in the ADOPT trial.

160
Q

Which statement is most accurate regarding patient safety organizations (PSOs)?

A	The creation of PSOs was authorized under the Data Security Act of 2015.
B	The Patient Safety Organization Privacy Protection Center (PSOPPC) ensures patient safety events submitted to the network are non-identifiable.
C	The National Committee for Quality Assurance coordinates the endorsement of the Common Formats developed by the Agency for Healthcare Research and Quality (AHRQ).
D	Use of Common Formats is required by PSOs for Centers for Medicare & Medicaid Services conditions of participation (CoPs).
A

B - The Patient Safety and Quality Improvement Act of 2005 (also known as the Patient Safety Act [PSA]) authorized the creation of PSOs in response to reports detailing the high cost of medical errors and high incidence of resultant adverse events, making Answer A incorrect. The PSA requires the AHRQ to administer a Network of Patient Safety Databases to assess nationwide, de-identified patient safety events. This legislation authorized the creation of PSOs to securely collect, aggregate, and analyze data to identify and reduce risks of harm. The AHRQ also created the PSOPPC to provide technical assistance to PSOs to ensure that patient safety events submitted to its databases were non-identifiable, making Answer B correct. Data are submitted by PSOs through Common Formats, developed by the AHRQ, and reviewed by the National Quality Forum to provide a systematic process for reporting adverse events, near misses, and unsafe conditions, making Answer C incorrect. Although data are submitted using Common Formats, their use is not required for CoPs in Quality Assessment and Performance Improvement surveys, making Answer D incorrect.

161
Q

In a study you are evaluating, a drug’s impact on the forced expiratory volume in 1 second (FEV1) is being assessed. Fifty-five subjects were studied at baseline and after 4 months of therapy. Which is the best statistical test to use for this comparison?

A	Two-sample t-test.
B	Paired t-test.
C	Chi-square test.
D	Wilcoxon rank sum test.
A

B - This question addresses (1) whether the FEV1 data are continuous and appropriate for a parametric analysis and (2) which type of study design was used for the data being compared. Answer B is correct. (1) The FEV1 data are continuous and (2) the study design is a paired design; thus, the comparison is a matched or paired comparison in the same study subjects between baseline and after 4 months of therapy. Therefore, Answers A, C, and D are incorrect because none describes a matched or paired analysis. Answer A describes a parametric test used for comparing values in two unrelated groups of individuals; Answer D describes a nonparametric test used for comparing values in two unrelated groups of individuals; and Answer C describes a test for comparing nominal data between two groups, typically reported as percentages.

162
Q

A 22-year-old woman presents for a follow-up from an ED visit 1 month ago, where she was given a new diagnosis of asthma and treated for an exacerbation with oral corticosteroids and an albuterol metered dose inhaler 2 puffs four times a day as needed for shortness of breath. Over the past month, she has been experiencing symptoms of coughing and wheezing an average of four times per week during the day, and she wakes up twice per week in the middle of the night with symptoms. She feels that only her most strenuous activities are limited by her symptoms. Spirometry today reveals a forced expiratory volume in 1 second (FEV1) of 83% of predicted (pre-bronchodilator).

Question 164
Given her asthma control at this visit, which is the preferred regimen?

A	Budesonide/formoterol 80/4.5 mcg 2 puffs twice daily and 1 puff as needed for shortness of breath.
B	Budesonide/formoterol 160/4.5 mcg 2 puffs twice daily and 1 puff as needed for shortness of breath.
C	Mometasone 110 mcg 1 puff daily in addition to albuterol MDI 2 puffs as needed for shortness of breath.
D	Mometasone 220 mcg 2 puffs daily in addition to albuterol MDI 2 puffs as needed for shortness of breath.
A

A - Because her asthma is classified as “moderate persistent,” it is recommended that she begin treatment in step 3 of the asthma treatment recommendations. For patients 12 years and older, the recommended initial treatment for step 3 is daily and as-needed combination low-dose inhaled corticosteroid (ICS)-formoterol in a single inhaler, also known as single maintenance and reliever therapy (SMART), making Answer A correct. Answer B is incorrect because the dose of 160/4.5 mcg 2 puffs twice daily is considered a medium-dose combination. Daily medium-dose ICS as monotherapy is considered an appropriate alternative therapy; however, a dose of 110 mcg daily is considered low dose (Answer C is incorrect),and a dose of 220 mcg 2 puffs daily is considered high dose (Answer D is incorrect).

163
Q

A 22-year-old woman presents for a follow-up from an ED visit 1 month ago, where she was given a new diagnosis of asthma and treated for an exacerbation with oral corticosteroids and an albuterol metered dose inhaler 2 puffs four times a day as needed for shortness of breath. Over the past month, she has been experiencing symptoms of coughing and wheezing an average of four times per week during the day, and she wakes up twice per week in the middle of the night with symptoms. She feels that only her most strenuous activities are limited by her symptoms. Spirometry today reveals a forced expiratory volume in 1 second (FEV1) of 83% of predicted (pre-bronchodilator).

Question 165
Which best classifies her current asthma severity?

A	Intermittent.
B	Mild persistent.
C	Moderate persistent.
D	Severe persistent.
A

C - In assessing this patient’s asthma, her frequency of daytime symptoms, interference with normal activity, and FEV1 fall under the “mild persistent” category. However, her frequency of nighttime awakenings with symptoms falls under the “moderate persistent” category. In classifying asthma severity or control, the most severe category is used on the basis of patient symptoms, making Answer C correct and Answers A, B, and D incorrect.

164
Q

In addition to standards of care with blood pressure and cholesterol management, which medication is most likely to improve a patient’s hyperglycemia and reduce cardiovascular events in a patient with established cardiovascular disease?

A	Glimepiride.
B	Saxagliptin.
C	Exenatide.
D	Empagliflozin
A

D - Empagliflozin is the only agent listed that has been shown in clinical trials to reduce cardiovascular events and improve blood glucose concentrations (Answer D is correct). Sulfonylureas, DPP-4 (dipeptidyl peptidase-4) inhibitors, and the GLP-1 (glucagon-like peptide-1) agonist exenatide have not been shown to reduce cardiovascular events (Answers A, B, and C are incorrect). Current ADA guidelines recommend considering an agent clinically studied to reduce cardiovascular outcomes, in this case empagliflozin, for patients with uncontrolled diabetes and established cardiovascular disease.

165
Q

A 72-year-old woman with chronic kidney disease (CKD) stage 5 on hemodialysis has been treated with stable doses of darbepoetin alfa subcutaneously once weekly for the past 12 months, together with weekly doses of intravenous iron. Three months ago, her Hgb was 10.5 g/dL and Hct was 31%. Her current Hgb is 9.2 g/dL and Hct is 27%. Her other current laboratory values include MCV 84 fL/cell, serum iron 30 mcg/dL, total iron-binding capacity (TIBC) 248 mcg/dL, and serum ferritin 140 ng/mL. Which is best for managing this patient’s anemia of CKD?

A	Increase weekly dose of darbepoetin alfa.
B	Increase darbepoetin alfa to two times weekly.
C	Increase dose of intravenous iron.
D	Continue current therapy.
A

C - Previous goals of therapy for anemia were to achieve an Hgb concentration of 11–12 g/dL. These goals have been revised with a less-clear lower-end goal but with trying to prevent the need for transfusions. This patient’s Hgb and Hct have decreased despite stable doses of darbepoetin, so continuing her current therapy would not be appropriate (Answer D is incorrect). This patient’s iron studies show that she is iron deficient. Her transferrin saturation is low, less than 30% [serum iron/TIBC × 100 = (30 mcg/dL/248 mcg/dL) × 100 = 12%], and serum ferritin is less than 500 ng/mL, so increasing the dose of iron is necessary (Answer C is correct). Neither increasing the dose nor increasing the frequency of administration of darbepoetin would improve this patient’s anemia until the iron stores are repleted (Answers A and B are incorrect).

166
Q

The physician you are working with would like to add clopidogrel 75 mg daily to a patient’s aspirin 81 mg daily regimen for management of the patient’s peripheral arterial disease (PAD). You mention that, according to the ACC/AHA guidelines, the use of dual antiplatelet therapy in patients with PAD has a level of recommendation of IIb. Which is the most accurate interpretation of this class (strength) of recommendation?

A	No benefit – not recommended.
B	Weak – may be reasonable.
C	Moderate – can be useful.
D	Strong – beneficial.
A

B - The ACC/AHA class of recommendations describes the strength of the recommendation and the anticipated magnitude and certainty of benefit. The highest class of recommendation is class I, which is a strong recommendation known to be beneficial, making Answer D incorrect. Next is a class IIa recommendation, which is a moderate recommendation that can be useful, making Answer C incorrect. A class IIb recommendation is a weak recommendation, which may be reasonable, making Answer B correct. Finally, there are class III recommendations, which may have no benefit or be harmful. Therefore, Answer A is incorrect.

167
Q

A 3-month-old girl born at 29 weeks 4 days’ gestation is being discharged from the neonatal ICU in January. She was intubated initially at birth, was extubated on day of life 9, and needed supplemental oxygen for the first 24 days of life. She lives with her parents and 2-year-old sister. Both she and her sister will attend day care. Her sister was admitted to the hospital last year with a viral bronchiolitis. Which recommendation regarding the use of palivizumab for preventing respiratory syncytial virus (RSV) is best for this patient?

A	Palivizumab is not indicated for this patient.
B	Palivizumab should be administered monthly for five doses because of her history of prematurity.
C	Palivizumab should be administered monthly for three doses because she has a sibling younger than 5 years and will be attending day care.
D	Palivizumab should be administered monthly for three doses because she was intubated at birth and required supplemental oxygen.
A

A - Before the 2014 publication of the AAP’s updated guidelines, infants who were born before 31 weeks 6 days’ gestation who were 6 months or younger at the beginning of RSV season were considered at high risk of severe infection. The 2014 guidelines changed the prematurity-based criteria for routine palivizumab prophylaxis to include only infants born before 29 weeks’ gestation who are younger than 12 months at the beginning of RSV season; thus, this patient no longer qualifies on the basis of prematurity alone (Answer B is incorrect). In addition, because the patient is being discharged in January, there are only about 3 months remaining in the RSV season; thus, five doses would be too many. Patients may receive a maximum of five doses throughout an RSV season. Risk factors such as day care attendance and school-aged siblings are no longer considered when determining whether prophylaxis is warranted (Answer C is incorrect). The guidelines stated that premature infants born before 32 weeks’ gestation who required supplemental oxygen for more than 28 days were indicated for palivizumab prophylaxis; however, the patient in this case only required supplemental oxygen for 24 days. Thus, she does not meet the criteria for chronic lung disease (Answer D is incorrect). Therefore, palivizumab is not indicated (Answer A is correct).

168
Q

For which patient is it most appropriate to recommend use of a continuous glucose monitoring (CGM) system as part of type 2 diabetes management?

A	On multiple oral agents; refuses to perform fingerstick blood glucose monitoring.
B	On multiple daily insulin injections; A1C is 6.9%.
C	On multiple oral agents; A1C is 10%.
D	On multiple daily insulin injections; experiences frequent low blood glucose readings.
A

C - Although the use of CGM systems significantly decreases the need for fingerstick monitoring, a fingerstick check is sometimes still required as part of the calibration process or to confirm a blood glucose reading (Answer A is incorrect). In adults with type 2 diabetes, evidence is lacking to show an improvement in hypoglycemic episodes with the use of CGM (Answer D is incorrect). Patients receiving both multiple daily insulin injections and oral medications have been shown to have significant reductions in their A1C values with the use of CGM. Therefore, the patient described in Answer C is most likely to benefit from use of CGM. The patient in Answer B already has an A1C that is at goal and likely does not need additional therapy optimization with CGM use.

169
Q

Which best describes an advantage of a tertiary literature source?

A	Provides the ability to obtain current and detailed information on a topic.
B	Is usually written by experts in the given field.
C	Requires an in-depth knowledge of study design and biostatistics.
D	Provides quick access to the primary literature.
A

B - Primary, secondary, and tertiary literature sources have many advantages and disadvantages. Although the primary literature is current and detailed, tertiary sources are typically delayed by at least 1 year and are more general in scope. The primary literature also requires an in-depth knowledge of study design and biostatistics, which is not necessary with the general scope of the tertiary literature. Therefore, Answers A and C are incorrect. Answer D is an advantage of secondary literature sources, but not tertiary sources. Tertiary literature, such as textbooks and chapters, is written by experts in their respective fields because of their extensive experience and ability to summarize a topic well, making Answer B correct.

170
Q

A 78-year-old woman was admitted to a long-term care facility for rehabilitation following hospitalization after having several mechanical falls at home. Her medical history is significant for hypertension, hypothyroidism, Alzheimer disease, hyperlipidemia, and osteoarthritis. Her medication regimen includes metoprolol succinate 50 mg daily, levothyroxine 75 mcg daily, atorvastatin 10 mg daily, pantoprazole 40 mg once daily, and donepezil 10 mg daily. Her blood pressure is 126/80 mm Hg and heart rate is 70 beats/minute. Basic metabolic panel results are all within reference ranges: 25-hydroxyvitamin D concentration 20 ng/mL, thyroid-stimulating hormone 1.89 mU/L, TC 180 mg/dL, LDL 140 mg/dL, HDL 35 mg/dL, and TG 176 mg/dL. Her Mini-Mental State Examination score is 16/30, and her Geriatric Depression Scale score is 2/15. According to the Beers Criteria, which recommendation would be most appropriate for this patient?

A	Discontinue metoprolol because of the risk of orthostatic hypotension.
B	Decrease atorvastatin dosing to every other day to avoid rhabdomyolysis.
C	Decrease the dose of donepezil because of the potential GI adverse effects.
D	Discontinue pantoprazole because of the risk of fractures.
A

D - The Beers Criteria specify several medication classes to be used with caution in older adults. In this patient’s case, discontinuing pantoprazole (Answer D) would be the most appropriate option; inappropriate use of proton pump inhibitors may increase the risk of Clostridioides difficile diarrhea and bone loss and fractures; also, this patient has no clear indication for their use. Her blood pressure and heart rate are well controlled; thus, discontinuing metoprolol would be inappropriate (Answer A is incorrect), and changing atorvastatin to every-other-day dosing or reducing the donepezil dose would currently be unnecessary as well (Answers B and C are incorrect).

171
Q

S.J. is a 66-year-old African American man (height 70 inches, weight 86 kg) who was recently admitted to the hospital with an ischemic stroke. His medical history includes atrial fibrillation and hypertension. His current medications include metoprolol tartrate 50 mg twice daily, apixaban 5 mg twice daily, and atorvastatin 40 mg daily. He is following up in the clinic today for management of his hypertension regimen. His vital signs include blood pressure 142/84 mm Hg (average of two measures using proper technique) and heart rate 76 beats/minute. His laboratory values are all within normal limits, including Na 136 mEq/L, K 5.0 mEq/L, and SCr 1.1 mg/dL. He has no current concerns. Which is the best recommendation for management of S.J.’s hypertension?

A	Discontinue metoprolol and add lisinopril.
B	Discontinue metoprolol and add amlodipine.
C	Continue metoprolol and add chlorthalidone.
D	Continue metoprolol and add losartan.
A

C - S.J. is currently receiving one antihypertensive medication (metoprolol), but his blood pressure is still not at goal (less than 130/80 mm Hg). S.J. has a history of atrial fibrillation and likely requires metoprolol for heart rate control, so discontinuing this medication would be inappropriate, making Answers A and B incorrect. The AHA/ACC guidelines recommend thiazide diuretics, angiotensin-converting enzyme inhibitors, or angiotensin receptor blockers (ARBs) in the management of hypertension in patients with a history of stroke. Because S.J. is African American, he is more likely to respond to chlorthalidone than losartan. In addition, the patient’s K is 5.0 mEq/L, which could make an ARB less appealing. The AHA/ACC guidelines recommend including calcium channel blockers or thiazide diuretics in the antihypertensive regimen for African American patients, making Answer C correct and Answer D incorrect.

172
Q

A 48-year-old woman presents to your institution with concerns for dizziness. She was recently initiated on hydrochlorothiazide, and her sodium concentration is 118 mEq/L. Which is the most appropriate sodium goal (in mEq/L) for this patient at 24 hours?

A	120.
B	128.
C	135.
D	140.
A

B - With a sodium concentration of 118 mEq/L and concerns for dizziness, this patient has symptoms of hyponatremia. In addition, she was recently initiated on hydrochlorothiazide. This patient’s hyponatremia can safely be corrected by an increase of up to 12 mEq/L in 24 hours. Increasing to 120 mEq/L (Answer A) would not be a sufficient correction, given the patient’s symptoms. Answers C and D would both increase the patient’s sodium greater than the 12-mEq/L threshold. Answer B is the best choice because it allows a 10-mEq/L increase.

173
Q

You are asked to provide an inservice presentation to a group of ICU nurses on the calcium channel blocker clevidipine. When putting the presentation together, you contemplate how best to focus the 10 minutes you have been allotted. To save on time and tailor your presentation to the audience, which information is best to delete from the inservice?

A	Clinical differences between clevidipine and nicardipine.
B	Clevidipine’s place in therapy as noted in the 2019 stroke early management guidelines.
C	Potential medication errors because of clevidipine’s visual similarities to propofol.
D	How to titrate clevidipine to reach goal blood pressures.
A

B - With limited time, it is important to tailor your presentation to the needs of your audience, in this case ICU nurses. Obviously, these nurses need to know how to titrate clevidipine effectively to reach goal blood pressures because that is their main role in the administration process (Answer D is incorrect). Because clevidipine and nicardipine differ pharmacokinetically (especially related to half-life), it is important for nurses to know this so that they can predict what will happen as they titrate clevidipine (Answer A is incorrect). Because ICU nurses are the ones who would potentially make a medication error by choosing a look-alike drug instead of what is ordered, reviewing the visual similarities and differences between clevidipine and propofol would be important (Answer C is incorrect). Although potentially important for the sake of completeness, spending extra time reviewing the stroke guidelines would probably not be good use of the limited time available (Answer B is correct).